You are on page 1of 191

Seat No -

Total number of questions : 60

10878_Control System I
Time : 1hr
Max Marks : 50
N.B

1) All questions are Multiple Choice Questions having single correct option.

2) Attempt any 50 questions out of 60.

3) Use of calculator is allowed.

4) Each question carries 1 Mark.

5) Specially abled students are allowed 20 minutes extra for examination.

6) Do not use pencils to darken answer.

7) Use only black/blue ball point pen to darken the appropriate circle.

8) No change will be allowed once the answer is marked on OMR Sheet.

9) Rough work shall not be done on OMR sheet or on question paper.

10) Darken ONLY ONE CIRCLE for each answer.

Q.no 1. From bode plot ......... coefficient cannot be determined

A : Different error

B : Static error

C : Both A and B

D : None of the above

Q.no 2. The bode plot is a plot relating log w with magnitude in decible and........

A : Phase angle

B : 90 degree

C : 180 degree

D : 45 degree

Q.no 3. In PID controller, 'K / S' indicates

A : Proportional Controller

B : Integral Controller

C : Differential Controller

D : Feedback Controller

Q.no 4. If number of poles are greater than number of zeroes then the system is known as

A : Stable system

B : Unstable system

C : Minimum phase system


D : Non-minimum phase system

Q.no 5. Generally compensating networks affects

A : Unaltered the performance of the system

B : Completely change out put of the system

C : Time domain performance parameters of the system

D : Frequency domain performance parameters of the system

Q.no 6. In polar plots, if a pole is added at the origin, what would be the value of the magnitude
at w = 0

A : zero

B : unity

C : infinity

D : unpredictable

Q.no 7. In a parallel combination, the direction of flow of signals through blocks in parallel must
resemble to the main

A : Forward

B : Feedback

C : Opposite

D : Diagonal

Q.no 8. PD controller affects system performance

A : System error will get amplified

B : System will become less oscillatory

C : Order of the system will increase by one

D : Settling time reduces

Q.no 9. Consider the following statement regarding Routh Hurwitz criterion:

A : It gives absolute stability

B : It gives gain and phase margin

C : It gives the number of roots lying in RHS of the s-plane

D : It gives gain, phase margin and number of roots lying in RHS of the s-plane

Q.no 10. If the initial conditions for a system are inherently zero, what does it physically mean?

A : The system is at rest but stores energy

B : The system is working but does not store energy

C : The system is at rest or no energy is stored in any of its part

D : The system is working with zero reference input

Q.no 11. PI controller affects system performance


A : System error will get amplified

B : System will become less oscillatory

C : Order of the system will increase by one

D : Settling time reduces

Q.no 12. If rise time is also taken into consideration it should be to the settling
time.

A : Inconsistent

B : Consistent

C : Perfect

D : Unique

Q.no 13. Synchro error detector has combination of

A : Synchro transmitter and Synchro receiver

B : Synchro control transformer and Synchro receiver

C : Synchro transmitter and Synchro control transformer

D : None of the above

Q.no 14. The necessary condition for the stability of the linear system is that all the coefficients of
characteristic equation 1+G(s)H(s) = 0, be real and have the :

A : Positive sign

B : Negative sign

C : Same sign

D : Both positive and negative

Q.no 15. The frequency at which the phase of the system acquires is known as 'Phase
crossover frequency'

A : 90°

B : -90°

C : 180°

D : -180°

Q.no 16. Armature controlled dc servomotor is represented on the basis of block diagram as

A : Closed loop type

B : Open loop type

C : Hybrid type

D : None of the above

Q.no 17. Equivalent analogy for current in mechanical translational system is

A : man

B : acceleration
C : velocity

D : displacement

Q.no 18. One of the disadvantage of block diagram reduction technique is

A : time consuming

B : For more number of blocks, simplication is difficult

C : both 1 and 2

D : none of these

Q.no 19. Field controlled dc servomotor is represented on the basis of block diagram as

A : Closed loop type

B : Open loop type

C : Hybrid type

D : None of the above

Q.no 20. Routh Hurwitz criterion cannot be applied when the characteristic equation of the
system containing coefficient’s which is/are

A : Exponential function of s

B : Sinusoidal function of s

C : Complex

D : Exponential and sinusoidal function of s and complex

Q.no 21. In frequency domain, response varies with respect to

A : time

B : frequency

C : time, frequency

D : constant

Q.no 22. At which frequency does the magnitude of the system becomes zero dB?

A : Resonant frequency

B : Cut-off frequency

C : Gain crossover frequency

D : Phase crossover frequency

Q.no 23. For relative stability of the system which of the following is sufficient?

A : Gain margin

B : Phase margin

C : Both (a) and (b)

D : None of the above


Q.no 24. Which one of the following is the most likely reason for large overshoot in a control
system?

A : High gain in a system

B : Presence of dead time delay in a system

C : High positive correcting torque

D : High retarding torque

Q.no 25. The nature of transient response is revealed by

A : Sine wave

B : Cos wave

C : Tan wave

D : Test signals

Q.no 26. Equivalent analogy for flux linkages in mechanical rotational system is

A : angular velocity

B : angular displacement

C : torque

D : moment of inertia

Q.no 27. Nyquist criterion determines stability of

A : Open loop system

B : close loop system

C : Open loop as well as close loop system

D : None of these

Q.no 28. Routh Hurwitz criterion gives:

A : Number of roots in the right half of the s-plane

B : Value of the roots

C : Number of roots in the left half of the s-plane

D : Number of roots in the top half of the s-plane

Q.no 29. Rotor of the ac servomotor is of the nature

A : Drag cup aluminium rotor

B : Squirrel cage rotor

C : Double layer type

D : Projected pole type

Q.no 30. Constuction wise AC servomotor is a

A : Single phase induction motor

B : Two phase induction motor


C : Three phase induction motor

D : Synchronous motor

Q.no 31. First order system is defined as :

A : Number of poles at origin

B : Order of the differential equation

C : Total number of poles of equation

D : Total number of poles and order of equation

Q.no 32. A transfer function has two zeroes at infinity. Then the relation between the numerator
(N) and the denominator degree (M) of the transfer function is:

A : N=M+2

B : N=M-2

C : N=M+1

D : N=M-1

Q.no 33. Ziegler-Nichol method is used for

A : To optimise frequency response

B : To tune PID Controller

C : To design system components

D : To change system input

Q.no 34. When the number of poles is equal to the number of zeroes, how many branches of root
locus tends towards infinity?

A:1

B:2

C:0

D : Equal to number of zeroes

Q.no 35. The main objective of drawing root locus plot is :

A : To obtain a clear picture about the open loop poles and zeroes of the system

B : To obtain a clear picture about the transient response of feedback system for various values of open
loop gain K

C : To determine sufficient condition for the value of ‘K’ that will make the feedback system unstable

D : Both b and c

Q.no 36. Root locus is used to calculate:

A : Marginal stability

B : Absolute stability

C : Conditional stability

D : Relative stability
Q.no 37. Which one of the following are correct? The root locus is the path of the roots of the
characteristic equation traced out in the s-plane?

A : As the input of the system is changed

B : As the output of the system is changed

C : As a system parameter is changed

D : As the sensitivity is changed

Q.no 38. PID Controller of electronic circuits has ..............as major component

A : Transister

B : Regulator

C : Op-Amp

D : Triggering circuit

Q.no 39. For open loop poles on imaginary axis Nyquist contour is selcted

A : Complete right half s plane

B : complete right half s plane bypassing poles on imaginary axis

C : complete left half s plane

D : complete left half s plane bypassing imaginary poles

Q.no 40. A close loop control system is stable if the Nyquist plot of the corresponding open loop
transfer function

A : encircles the s plane point (-1+j0) in the counterclockwise direction as many times as the number of
right half s plane poles

B : encircles the s plane point (0-j1) in the clockwise direction as many times as the number of right
half s plane poles

C : encircles the s plane point (-1+j0) in the counterclockwise direction as many times as the number of
left half s plane poles

D : encircles the s plane point (-1+j0) in the clockwise direction as many times as the number of right
half s plane poles

Q.no 41. In polar plots, what does each and every point represent w.r.t magnitude and angle

A : Vector

B : Scalar

C : Phasor

D : Differentiator

Q.no 42. Ramp input :

A : Denotes constant velocity

B : Value increases linearly with time

C : It denotes constant velocity and varies linearly with time

D : It varies exponentially with time

Q.no 43. PID controller affects


A : Frequency domain response of the system

B : Time domain response of the system

C : Time damain and frequency domain response of the system at a time

D : Neither Time damain nor frequency domain response of the system

Q.no 44. What is the number of the root locus segments which do not terminate on zeroes?

A : The number of poles

B : The number of zeroes

C : The difference between the number of poles and zeroes

D : The sum of the number of poles and the number of the zeroes

Q.no 45. Which unit is adopted for phase measurement in Bode plots?

A : Degree

B : Decimal

C : Decibel

D : Deviation

Q.no 46. Consider the following statements regarding root loci:

A : All root loci start from the respective poles of G(s) H(s)

B : All root loci end at the respective zeros of G(s) H(s) or go to infinity

C : The root loci are symmetrical about the imaginary axis of the s-plane

D : All root loci start and end from the respective poles of G(s) H(s) or go to infinity

Q.no 47. Due to Phase Lead network, system's

A : Phase margin increases

B : Phase margin decreses

C : No change in phase margin

D : Can't comment

Q.no 48. The peak percentage overshoot of the closed loop system is :

A : 5 Percent

B : 10 Percent

C : 16.3 Percent

D : 1.63 Percent

Q.no 49. In frequency reponse of the system if input is having frequency f output frequency will
be

A:f

B : 2f

C:0
D : None of these

Q.no 50. Which principle specifies the relationship between enclosure of poles & zeros by s-plane
contour and the encirclement of origin by q(s) plane contour

A : Argument

B : Agrrement

C : Assesment

D : Assortment

Q.no 51. Which unit is adopted for magnitude measurement in Bode plots?

A : Degree

B : Decimal

C : Decibel

D : Deviation

Q.no 52. Due to phase lag network, system's

A : Bandwidth increases

B : Bandwidth decreses

C : bandwith remain unchanged

D : Can't comment

Q.no 53. The polar plot of a system transfer function passing through point (-1,0) its gain margin
is

A : 1 dB

B : Zero

C : infinity

D : 10 degree

Q.no 54. If the unity feedback system is given by the open loop transfer function G(s) = ks2 / [(1
+ 0.3s) (1+ 0.05s)], what would be the initial slope of magnitude plot?

A : 20 dB/decade

B : 40 dB/decade

C : 60 dB/decade

D : Unpredictable

Q.no 55. Banwidth is defined as frequency at which magnitude M(jw) equals to

A : 30dB

B : 3 dB

C : 1 dB

D : 0.67 dB

Q.no 56. Consider the assertions related to block diagram. Which among them represents the
precise condition? A. Block diagram is used for analysis & design of control system. B. Block
diagram also provides the information regarding the physical construction of the system.

A : A is true, B is false

B : A is false, B is true

C : Both A & B are false

D : Both A & B are true

Q.no 57. If a system is having damping ratio =0.7 with natural frequency wn=2rad/sec. What will
be the value of resonant frequency

A : 1.428

B : 12

C : 0.2828

D : 1.2828

Q.no 58. A system with transfer function 1/Ts+1, subjected to a step input takes to seconds to
reach 50% of step height. The value of t is :

A : 6.9 s

B : 10s

C : 14.4s

D : 20

Q.no 59. Cut off is the slope of log-magnitude curve

A : At the start of curve

B : At the end of curve

C : Near the cut off frequency

D : None of these

Q.no 60. Consider a feedback system with gain margin of about 30. At what point does Nyquist
plot crosses negative real axis

A : -3

B : -0.3

C : -30

D : -03

Q.no 1. What will be the nature of time response if the roots of the characteristic equation are
located on the s-plane imaginary axis?

A : Oscillations

B : Damped oscillations

C : No oscillations

D : Under damped oscillations

Q.no 2. If the roots of the have negative real parts then the response is

A : Stable
B : Unstable

C : Marginally stable

D : Bounded

Q.no 3. is a quantitative measure of how fast the transients die out in the system.

A : Absolutely stable

B : Conditionally stable

C : Unstable

D : Relative Stability

Q.no 4. Servomotor has Torque - Slip characteristic nature

A : Positive slope Torque Slip characteristic

B : Negative slope Torque Slip characteristic

C : Non linear slope Torque Slip characteristic

D : None of the above

Q.no 5. Linear mathematical model applies to :

A : Linear systems

B : Stable systems

C : Unstable systems

D : Non-linear systems

Q.no 6. An automatic toaster is a classfied as which type 0f control system.

A : open

B : closed

C : partially closed

D : any of the above

Q.no 7. Steady state refers to

A : Error at the steady state

B : Error at the transient state

C : Error at both state

D : Precision

Q.no 8. The disadvantages of the error constants are:

A : They do not give the information of the steady state error when the inputs are other than the three
basic types

B : Error constant fail to indicate the exact manner in which the error function change with time.

C : They do not give information of the steady state error and fail to indicate the exact manner in which
the error function change with time
D : They give information of the steady state error

Q.no 9. Consider the following, which statement is true:

A : A system is said to be stable if its output is bounded for any input

B : A system is said to be stable if all the roots of the characteristic equation lie on the left half of the s
plane.

C : A system is said to be stable if all the roots of the characteristic equation have negative real parts.

D : A second order system is always stable for finite values of open loop gain

Q.no 10. Which one is not frequency domain specification

A : Resonant peak

B : Resonant frequency

C : Bandwidth

D : rise time

Q.no 11. If a pole is located at origin, how does it get represented on the magnitude plot?

A : -10 log (ω) dB

B : -20 log (ω) dB

C : . -40 log (ω) dB

D : -60 log (ω) dB

Q.no 12. The bode plot is applicable to. .....network.

A : Maximum phase

B : Minimum phase

C : All phase

D : Zero phase

Q.no 13. In feedback control system, with positive value of feedback gain, the overall gain of
the system will

A : decrease

B : increase

C : be unaffected

D : any of the above

Q.no 14. According to the principle of log-scales, if the ratio between two points is same, then the
two points get equally.

A : United

B : Separated

C : Multiplexed

D : Mixed

Q.no 15. PD controller affects system performance


A : Increases overshoot of the system

B : Decreases overshoot of the system

C : Overshoot is unaffected

D : Can't say anything

Q.no 16. Component Synchro is

A : AC operated device

B : DC operated device

C : AC & DC operated device

D : No electrical signal required for it's operation

Q.no 17. Step signal is the signal whose values is :

A : 1 for all values greater than zero

B : Indeterminate 0 to 10

C : It is zero for time less than 1

D : It is 1 for time less than 0

Q.no 18. Proportional controller in the system has effect

A : System Error reduces to the zero

B : Amplify oscillations

C : Increases Settling time

D : Amplify error of the system

Q.no 19. For Nyquist contour the size of radius is

A:0

B:1

C : infinity

D : 10

Q.no 20. For the system 2/s+1, the approximate time taken for a step response to reach 98% of its
final value is:

A : 1s

B : 2s

C : 4s

D : 8s

Q.no 21. When deriving the transfer function of a linear element

A : Both initial conditions and loading are taken into account

B : Initial conditions are taken into account but the element is assumed to be not loaded

C : Initial conditions are assumed to be zero but loading is taken into account
D : Initial conditions are assumed to be zero and the element is assumed to be not loaded

Q.no 22. Which system conveniently see the impact of poles and zeros on phase and gain
margin?

A : Root locus

B : Nyquist plot

C : Routh-Hurwitz criterion

D : Bode plot

Q.no 23. Gain cross over frequency at which magnitude is

A : Zero

B : Infinity

C : Unity

D : constant

Q.no 24. How is the sinusoidal transfer function obtained from the system transfer function in
frequency domain?

A : Replacement of jw by s

B : Replacement of s by w

C : Replacement of s by jw

D : Replacement of w by s

Q.no 25. Generally servomotors are located in the .............. of the system

A : Feedback path

B : Error detector

C : Forward path

D : None of the above

Q.no 26. The order of the auxiliary polynomial is always:

A : Even

B : Odd

C : May be even or odd

D:0

Q.no 27. In control system, which of the following component will act as error detector

A : Op-Amp

B : Synchro

C : Potentiometer

D : All of above

Q.no 28. Spring constant in force-voltage analogy is analogous to


A : capacitance

B : reciprocal of capacitance

C : current

D : resistance

Q.no 29. The polar plot and nicholas plot can be had from bode plot and

A : Same angle

B : Vice versa

C : Different angle

D : Any of the above

Q.no 30. Which one of the following statements is not correct?

A : Root loci can be used for analyzing stability and transient performance

B : Root loci provide insight into system stability and performance

C : Shape of the root locus gives idea of type of controller needed to meet design specification

D : Root locus can be used to handle more than one variable at a time

Q.no 31. Number of roots of characteristic equation is equal to the number of

A : Branches

B : Root

C : Stem

D : Poles

Q.no 32. According to Nyquist stability criterion, where should be the position of all zeros of q(s)
corresponding to s-plane?

A : Random

B : on right half

C : on left half

D : at the center

Q.no 33. For Type 1 system what would be magnitude of system at w=0

A:0

B : Infinity

C:K

D : None of these

Q.no 34. While increasing the value of gain K, the system becomes

A : Less stable

B : More stable

C : Unstable
D : Absolute stable

Q.no 35. For which systems are the signal flow graphs applicable?

A : Causal

B : Invertible

C : Linear time invariant system

D : Dynamic

Q.no 36. Zero of the compensator is dominating, in case of

A : Lag Compensator

B : Lead Compensator

C : Lag - Lead Compensator

D : Under Compensator

Q.no 37. Gain margin of a first or second order system is

A:1

B : 100

C:0

D : Infinity

Q.no 38. Pole of the compensator is dominating, in case of

A : Lag Compensator

B : Lead Compensator

C : Lag - Lead Compensator

D : Under Compensator

Q.no 39. In a signal flow graph method, how is an overall transfer function of a system obtained?

A : Poisson’s equation

B : Block diagram reduction rules

C : Mason’s formula

D : Lagrange’s equation

Q.no 40. Which of the following statements are correct?

A : Root locus is for the negative feedback systems

B : Complementary root locus is for the positive feedback systems

C : Root locus is for the negative feedback and Complementary root locus is for the positive feedback
systems

D : Complementary root locus is for the negative feedback systems

Q.no 41. Transient response analysis is done for systems

A : Unstable
B : Stable

C : Conditionally stable

D : Marginally stable

Q.no 42. Laplace transform of unit impulse signal is :

A : A/s

B:A

C:1

D : 1/s

Q.no 43. The damping ratio and peak overshoot are measures of:

A : Relative stability

B : Speed of response

C : Steady state error

D : Absolute stability

Q.no 44. Gain margin expressed in decibels is

A : Positive if Kg greater than 1 and negative for Kg less than 1

B : Negative if Kg greater than 1 and negative for Kg less than 1

C : Always zero

D : Infinity for Kg equal to 1

Q.no 45. A car is running at a constant speed of 50 km/h, which of the following is the feedback
element for the driver ?

A : Clutch

B : Needle of the speedometer

C : Steering wheel

D : Eyes

Q.no 46. By adding a pole at s=0 polar plot of the system will shift

A : at higher and lower frequencies by 90 degree clockwise

B : higher frequencies shift by 90 degree cloclwise

C : higher frequencies by 180 degree clockwise

D : Not change at all

Q.no 47. The polar plot of system is

A : locus of the real and imaginary parts of Y(jω) in the polar plane

B : plot of real part

C : plot of imaginary part

D : Phase angle plot


Q.no 48. To find system’s response by means of convolution integral of the system
is used.

A : Sum

B : Difference

C : Exponential

D : Weighing

Q.no 49. Cut off frequency is the frequency at which magnitude of closed loop frequency
response is

A : 1 db below its zero frequency

B : 2 db below its zero frequency

C : 3 db below its zero frequency

D : 4 db below its zero frequency

Q.no 50. Which type servomotor is generally not used in the practice

A : Armature Controlled DC servomotor

B : AC Servomotor

C : Field Controlled DC Servomotor

D : All of above

Q.no 51. In connection with servomotor, which of the statement from the following is correction

A : Servomotor is high inertia motor

B : Servomotor has large diameter and small length rotor

C : Servomotor has small diameter and large length rotor

D : Servomotor is slow accelerating and decelerating motor

Q.no 52. If the gain of the system is reduced to a zero value, the roots of the system in the s-plane,

A : Coincide with zero

B : Move away from zero

C : Move away from poles

D : Coincide with the poles

Q.no 53. Generally location of Controllers and Compensators, in the control system is at

A : In the Input side

B : In the output side

C : In the feedback path

D : After error detector and before plat

Q.no 54. If the phase angle at gain crossover frequency is estimated to be -105°, what will be the
value of phase margin of the system?

A : 23°
B : 45°

C : 60°

D : 75°

Q.no 55. The bode plot is used to analyse which of the following?

A : Minimum phase network

B : Lag lead network

C : Maximum phase network

D : All phase network

Q.no 56. Transfer function of potentiometer is

A : Vo(S) / Vin(S)

B : Vin(S) / Vo(S)

C : Vo(S) / Θ(S)

D : Θ(S) / Vo(S)

Q.no 57. Assertion (A): Practical systems must be closed loop system. Reason (R): This is due to
the fact that closed loop systems are least affected by parameter variations, stable, higher
bandwidth, linear and more speed of response.

A : Both A and R are true and R is the correct explanation of A.

B : Both A and R are true but R is not the correct explanation of A.

C : A is true but R is false.

D : A is false but R is true.

Q.no 58. TF of the compensaor given bellow, A<B represent, if G(S)H(S) = (S+A) / (S+B)

A : Lag Compensator

B : Lead Compensator

C : Lag - Lead Compensator

D : Under Compensator

Q.no 59. Root locus of s(s+2)+K(s+4) =0 is a circle. What are the coordinates of the center of this
circle?

A : -2,0

B : -3,0

C : -4,0

D : -5,0

Q.no 60. The main requirement of two potentiometers when acted as error detector is

A : Two potentiometers should be identical

B : Two potentiometers may be different

C : Power rating must be same but potentiometer resistance may be different


D : No any specific condition

Q.no 1. In an octave frequency band, the ratio of f2 / f1 is equivalent to

A:2

B:4

C:8

D : 10

Q.no 2. Nyquist plot is

A : symmetrical around real axis

B : symmetrical around imaginary axis

C : not symmetrical

D : symmetrical about both axis

Q.no 3. Generally compensating networks affects

A : Unaltered the performance of the system

B : Completely change out put of the system

C : Time domain performance parameters of the system

D : Frequency domain performance parameters of the system

Q.no 4. Transfer founction, when the bode diagram is plotted should be of the form

A : (1+T)

B : (1+S)

C : (Ts)

D : (1+Ts)

Q.no 5. The input signals to control systems are not known fully ahead of time, the characteristics
of control system which suddenly strain a control system are:

A : Sudden shock

B : Indeterminate at zero

C : It is zero for time less than zero

D : 1 for all values greater than zero

Q.no 6. Phase cross over frequency at which phase is

A : 90

B:0

C : 180

D : -180

Q.no 7. If the constant 'k' is positive, then what would be its contribution on the phase plot?

A : 0°
B : 45°

C : 90°

D : 180°

Q.no 8. RLC network is an example of

A : open loop control system

B : closed loop control system

C : time variant system

D : time invariant system

Q.no 9. Which one of the following is the most likely reason for large overshoot in a control
system?

A : High gain in a system

B : Presence of dead time delay in a system

C : High positive correcting torque

D : High retarding torque

Q.no 10. In DC position controlled system, ................ is used as an error detector

A : Op - Amp

B : Synchro

C : Potntiometer

D : None of the above

Q.no 11. The type of transfer function used in Bode plot is

A : G(s)

B : G(j)

C : G(jw)

D : G(js)

Q.no 12. Bounded input and Bounded output stability notion concerns with :

A : A system under influence of input

B : A system not under influence of input

C : A system under influence of output

D : A system not under influence of output

Q.no 13. One of the disadvantage of block diagram reduction technique is

A : time consuming

B : For more number of blocks, simplication is difficult

C : both 1 and 2

D : none of these
Q.no 14. To reduce settling time, we have to prefer

A : Proportional Controller

B : Derivative Controller

C : Integral Controller

D : None of the above

Q.no 15. In Nyquist criterion roots of the characteristic equation are given by

A : Zeros of open loop transfer function

B : Zeros of closed loop transfer function

C : Poles of closed loop transfer function

D : Poles of open loop transfer function

Q.no 16. The critical value of gain for a system is 40 and gain margin is 6dB. The system is
operating at a gain of:

A : 20

B : 40

C : 80

D : 120

Q.no 17. Armature controlled dc servomotor is represented on the basis of block diagram as

A : Closed loop type

B : Open loop type

C : Hybrid type

D : None of the above

Q.no 18. Constuction wise AC servomotor is a

A : Single phase induction motor

B : Two phase induction motor

C : Three phase induction motor

D : Synchronous motor

Q.no 19. The overall transfer function from block diagram reduction for cascaded blocks is :

A : Sum of individual gain

B : Product of individual gain

C : Difference of individual gain

D : Division of individual gain

Q.no 20. Consider the following statement regarding Routh Hurwitz criterion:

A : It gives absolute stability

B : It gives gain and phase margin


C : It gives the number of roots lying in RHS of the s-plane

D : It gives gain, phase margin and number of roots lying in RHS of the s-plane

Q.no 21. The magnitude & phase relationship between input and the steady state
output is called as frequency domain.

A : Step

B : Sinusoidal

C : Ramp

D : Parabolic

Q.no 22. Asymptotic stability is concerned with:

A : A system under influence of input

B : A system not under influence of input

C : A system under influence of output

D : A system not under influence of output

Q.no 23. In connection with PI controller, which statement is correct

A : It increases order of the system by one

B : Order of the system is unchanged

C : It decreases order of the system by one

D : Can't say anything

Q.no 24. Equivalent analogy for flux linkages in mechanical rotational system is

A : angular velocity

B : angular displacement

C : torque

D : moment of inertia

Q.no 25. Gain margin is reciprocal of

A : Magitude

B : Phase

C : Magnitude and phase

D : None

Q.no 26. A linear time invariant system is stable if :

A : System in excited by the bounded input, the output is also bounded

B : In the absence of input output tends zero

C : Both a and b

D : System in excited by the bounded input, the output is not bounded

Q.no 27. Rotor of the ac servomotor is of the nature


A : Drag cup aluminium rotor

B : Squirrel cage rotor

C : Double layer type

D : Projected pole type

Q.no 28. In a signal flow graph, nodes are represented by small

A : Circles

B : Squares

C : Arrows

D : Pointers

Q.no 29. Nyquist criterion determines stability of

A : Open loop system

B : close loop system

C : Open loop as well as close loop system

D : None of these

Q.no 30. Synchro error detector has combination of

A : Synchro transmitter and Synchro receiver

B : Synchro control transformer and Synchro receiver

C : Synchro transmitter and Synchro control transformer

D : None of the above

Q.no 31. In polar plots, what does each and every point represent w.r.t magnitude and angle

A : Vector

B : Scalar

C : Phasor

D : Differentiator

Q.no 32. Which one of the following are correct? The root locus is the path of the roots of the
characteristic equation traced out in the s-plane?

A : As the input of the system is changed

B : As the output of the system is changed

C : As a system parameter is changed

D : As the sensitivity is changed

Q.no 33. Which unit is adopted for magnitude measurement in Bode plots?

A : Degree

B : Decimal

C : Decibel
D : Deviation

Q.no 34. If the unity feedback system is given by the open loop transfer function G(s) = ks2 / [(1
+ 0.3s) (1+ 0.05s)], what would be the initial slope of magnitude plot?

A : 20 dB/decade

B : 40 dB/decade

C : 60 dB/decade

D : Unpredictable

Q.no 35. Ramp input :

A : Denotes constant velocity

B : Value increases linearly with time

C : It denotes constant velocity and varies linearly with time

D : It varies exponentially with time

Q.no 36. For open loop poles on imaginary axis Nyquist contour is selcted

A : Complete right half s plane

B : complete right half s plane bypassing poles on imaginary axis

C : complete left half s plane

D : complete left half s plane bypassing imaginary poles

Q.no 37. Consider the following statements regarding root loci:

A : All root loci start from the respective poles of G(s) H(s)

B : All root loci end at the respective zeros of G(s) H(s) or go to infinity

C : The root loci are symmetrical about the imaginary axis of the s-plane

D : All root loci start and end from the respective poles of G(s) H(s) or go to infinity

Q.no 38. Root locus is used to calculate:

A : Marginal stability

B : Absolute stability

C : Conditional stability

D : Relative stability

Q.no 39. Assertion (A): It is observed that step function is first derivative of a ramp function and
impulse function is first derivative of a step function. Reason (R): From the derived time
response expression it is concluded that the output time response also follows the same sequence
as that of input functions.

A : Both A and R are true and R is the correct explanation of A

B : Both A and R are true but R is not correct explanation of A

C : Both A is True but R is false

D : Both A is False but R is true


Q.no 40. When the number of poles is equal to the number of zeroes, how many branches of root
locus tends towards infinity?

A:1

B:2

C:0

D : Equal to number of zeroes

Q.no 41. First order system is defined as :

A : Number of poles at origin

B : Order of the differential equation

C : Total number of poles of equation

D : Total number of poles and order of equation

Q.no 42. Due to Phase Lead network, system's

A : Phase margin increases

B : Phase margin decreses

C : No change in phase margin

D : Can't comment

Q.no 43. Due to phase lag network, system's

A : Bandwidth increases

B : Bandwidth decreses

C : bandwith remain unchanged

D : Can't comment

Q.no 44. Zeroes are defined as:

A : Roots of the denominator of the closed loop transfer function

B : Roots of the numerator of the closed loop transfer function

C : Parts of the numerator

D : Parts of the denominator

Q.no 45. PID Controller of electronic circuits has ..............as major component

A : Transister

B : Regulator

C : Op-Amp

D : Triggering circuit

Q.no 46. A close loop control system is stable if the Nyquist plot of the corresponding open loop
transfer function

A : encircles the s plane point (-1+j0) in the counterclockwise direction as many times as the number of
right half s plane poles
B : encircles the s plane point (0-j1) in the clockwise direction as many times as the number of right
half s plane poles

C : encircles the s plane point (-1+j0) in the counterclockwise direction as many times as the number of
left half s plane poles

D : encircles the s plane point (-1+j0) in the clockwise direction as many times as the number of right
half s plane poles

Q.no 47. PID controller affects

A : Frequency domain response of the system

B : Time domain response of the system

C : Time damain and frequency domain response of the system at a time

D : Neither Time damain nor frequency domain response of the system

Q.no 48. Banwidth is defined as frequency at which magnitude M(jw) equals to

A : 30dB

B : 3 dB

C : 1 dB

D : 0.67 dB

Q.no 49. In frequency reponse of the system if input is having frequency f output frequency will
be

A:f

B : 2f

C:0

D : None of these

Q.no 50. Bode analysis method can be applied

A : If transfer function has no poles and zeros on R.H. of s-plane

B : If transfer function has no poles on R.H. of s-plane

C : f transfer function has no zero on R.H. of s-plane

D : To all transfer functions

Q.no 51. What is the number of the root locus segments which do not terminate on zeroes?

A : The number of poles

B : The number of zeroes

C : The difference between the number of poles and zeroes

D : The sum of the number of poles and the number of the zeroes

Q.no 52. Lag & Lead electrical networks are generally constructed using

A : Using Resistances

B : Using Inductors

C : Using Capacitors
D : Using Resistance & Capacitors combinations

Q.no 53. The polar plot of a system transfer function passing through point (-1,0) its gain margin
is

A : 1 dB

B : Zero

C : infinity

D : 10 degree

Q.no 54. The main objective of drawing root locus plot is :

A : To obtain a clear picture about the open loop poles and zeroes of the system

B : To obtain a clear picture about the transient response of feedback system for various values of open
loop gain K

C : To determine sufficient condition for the value of ‘K’ that will make the feedback system unstable

D : Both b and c

Q.no 55. In frequency response, the resonance frequency is basically a measure of of


response.

A : Speed

B : distance

C : Angle

D : Curvature

Q.no 56. In a bode magnitude plot, which one of the following slopes would be exhibited at high
frequencies by a 4th order all-pole system?

A : -80dB/decade

B : -40 dB/decade

C : 40 dB/decade

D : 80 dB/decade

Q.no 57. If a Nyquist plot of G (jω) H (jω) for a closed loop system passes through (-2, j0) point in
GH plane, what would be the value of gain margin of the system in dB?

A : 0 dB

B : 2.20 dB

C : 4 dB

D : 6.02 dB

Q.no 58. If a system is having damping ratio =0.7 with natural frequency wn=2rad/sec. What will
be the value of resonant frequency

A : 1.428

B : 12

C : 0.2828

D : 1.2828
Q.no 59. TF of the compensaor given bellow, A>B represent, if G(S)H(S) = (S+A) / (S+B)

A : Lag Compensator

B : Lead Compensator

C : Lag - Lead Compensator

D : Under Compensator

Q.no 60. A system with transfer function 1/Ts+1, subjected to a step input takes to seconds to
reach 50% of step height. The value of t is :

A : 6.9 s

B : 10s

C : 14.4s

D : 20

Q.no 1. At which frequency does the magnitude of the system becomes zero dB?

A : Resonant frequency

B : Cut-off frequency

C : Gain crossover frequency

D : Phase crossover frequency

Q.no 2. PI controller affects system performance

A : System error will get amplified

B : System will become less oscillatory

C : Order of the system will increase by one

D : Settling time reduces

Q.no 3. In a parallel combination, the direction of flow of signals through blocks in parallel must
resemble to the main

A : Forward

B : Feedback

C : Opposite

D : Diagonal

Q.no 4. Equivalent analogy for torque in mechanical rotational system is

A : force

B : man

C : acceleration

D : displacement

Q.no 5. Consider the following, which statement is true:

A : A system is said to be stable if its output is bounded for any input


B : A system is said to be stable if all the roots of the characteristic equation lie on the left half of the s
plane.

C : A system is said to be stable if all the roots of the characteristic equation have negative real parts.

D : A second order system is always stable for finite values of open loop gain

Q.no 6. The capacitance, in force-current analogy, is analogous to

A : momentum

B : velocity

C : displacement

D : mass

Q.no 7. Equivalent analogy for current in mechanical translational system is

A : man

B : acceleration

C : velocity

D : displacement

Q.no 8. In frequency domain, response varies with respect to

A : time

B : frequency

C : time, frequency

D : constant

Q.no 9. Component Synchro is

A : AC operated device

B : DC operated device

C : AC & DC operated device

D : No electrical signal required for it's operation

Q.no 10. From bode plot ......... coefficient cannot be determined

A : Different error

B : Static error

C : Both A and B

D : None of the above

Q.no 11. In PID controller, 'K * S' indicates

A : Proportional Controller

B : Integral Controller

C : Differential Controller

D : Feedback Controller
Q.no 12. Routh Hurwitz criterion gives:

A : Number of roots in the right half of the s-plane

B : Value of the roots

C : Number of roots in the left half of the s-plane

D : Number of roots in the top half of the s-plane

Q.no 13. The disadvantages of the error constants are:

A : They do not give the information of the steady state error when the inputs are other than the three
basic types

B : Error constant fail to indicate the exact manner in which the error function change with time.

C : They do not give information of the steady state error and fail to indicate the exact manner in which
the error function change with time

D : They give information of the steady state error

Q.no 14. Which system conveniently see the impact of poles and zeros on phase and gain
margin?

A : Root locus

B : Nyquist plot

C : Routh-Hurwitz criterion

D : Bode plot

Q.no 15. Proportional controller in the system has effect

A : System Error reduces to the zero

B : Amplify oscillations

C : Increases Settling time

D : Amplify error of the system

Q.no 16. For relative stability of the system which of the following is sufficient?

A : Gain margin

B : Phase margin

C : Both (a) and (b)

D : None of the above

Q.no 17. In polar plots, if a pole is added at the origin, what would be the value of the magnitude
at w = 0

A : zero

B : unity

C : infinity

D : unpredictable

Q.no 18. If rise time is also taken into consideration it should be to the settling
time.
A : Inconsistent

B : Consistent

C : Perfect

D : Unique

Q.no 19. For making an unstable system stable:

A : Gain of the system should be increased

B : Gain of the system should be decreased

C : The number of zeroes to the loop transfer function should be increased

D : The number of poles to the loop transfer function should be increased

Q.no 20. PD controller affects system performance

A : System error will get amplified

B : System will become less oscillatory

C : Order of the system will increase by one

D : Settling time reduces

Q.no 21. When deriving the transfer function of a linear element

A : Both initial conditions and loading are taken into account

B : Initial conditions are taken into account but the element is assumed to be not loaded

C : Initial conditions are assumed to be zero but loading is taken into account

D : Initial conditions are assumed to be zero and the element is assumed to be not loaded

Q.no 22. Which one is not frequency domain specification

A : Resonant peak

B : Resonant frequency

C : Bandwidth

D : rise time

Q.no 23. Spring constant in force-voltage analogy is analogous to

A : capacitance

B : reciprocal of capacitance

C : current

D : resistance

Q.no 24. Routh Hurwitz criterion cannot be applied when the characteristic equation of the
system containing coefficient’s which is/are

A : Exponential function of s

B : Sinusoidal function of s

C : Complex
D : Exponential and sinusoidal function of s and complex

Q.no 25. According to the principle of log-scales, if the ratio between two points is same, then the
two points get equally.

A : United

B : Separated

C : Multiplexed

D : Mixed

Q.no 26. Frequency range of bode magnitude and phases are decided by :

A : The lowest and higher important frequencies of dominant factors of the OLTF

B : The lowest and highest important frequencies of all the factors of the open loop transfer function

C : Resonant frequencies of the second factors

D : None of the above

Q.no 27. Steady state refers to

A : Error at the steady state

B : Error at the transient state

C : Error at both state

D : Precision

Q.no 28. A ramp input applied to a unity feedback system results in 5% steady state error. The
type number and zero frequency gain of the system are respectively

A : 1 and 20

B : 0 and 20

C : 0 and 1/20

D : 1 and 1/20

Q.no 29. In PID controller, 'K / S' indicates

A : Proportional Controller

B : Integral Controller

C : Differential Controller

D : Feedback Controller

Q.no 30. The order of the auxiliary polynomial is always:

A : Even

B : Odd

C : May be even or odd

D:0

Q.no 31. In connection with servomotor, which of the statement from the following is correction
A : Servomotor is high inertia motor

B : Servomotor has large diameter and small length rotor

C : Servomotor has small diameter and large length rotor

D : Servomotor is slow accelerating and decelerating motor

Q.no 32. Zero of the compensator is dominating, in case of

A : Lag Compensator

B : Lead Compensator

C : Lag - Lead Compensator

D : Under Compensator

Q.no 33. Which principle specifies the relationship between enclosure of poles & zeros by s-plane
contour and the encirclement of origin by q(s) plane contour

A : Argument

B : Agrrement

C : Assesment

D : Assortment

Q.no 34. While increasing the value of gain K, the system becomes

A : Less stable

B : More stable

C : Unstable

D : Absolute stable

Q.no 35. According to Nyquist stability criterion, where should be the position of all zeros of q(s)
corresponding to s-plane?

A : Random

B : on right half

C : on left half

D : at the center

Q.no 36. For Type 1 system what would be magnitude of system at w=0

A:0

B : Infinity

C:K

D : None of these

Q.no 37. Pole of the compensator is dominating, in case of

A : Lag Compensator

B : Lead Compensator
C : Lag - Lead Compensator

D : Under Compensator

Q.no 38. Number of roots of characteristic equation is equal to the number of

A : Branches

B : Root

C : Stem

D : Poles

Q.no 39. Which unit is adopted for phase measurement in Bode plots?

A : Degree

B : Decimal

C : Decibel

D : Deviation

Q.no 40. To find system’s response by means of convolution integral of the system
is used.

A : Sum

B : Difference

C : Exponential

D : Weighing

Q.no 41. By adding a pole at s=0 polar plot of the system will shift

A : at higher and lower frequencies by 90 degree clockwise

B : higher frequencies shift by 90 degree cloclwise

C : higher frequencies by 180 degree clockwise

D : Not change at all

Q.no 42. The damping ratio and peak overshoot are measures of:

A : Relative stability

B : Speed of response

C : Steady state error

D : Absolute stability

Q.no 43. Generally location of Controllers and Compensators, in the control system is at

A : In the Input side

B : In the output side

C : In the feedback path

D : After error detector and before plat

Q.no 44. The bode plot is used to analyse which of the following?
A : Minimum phase network

B : Lag lead network

C : Maximum phase network

D : All phase network

Q.no 45. The polar plot of system is

A : locus of the real and imaginary parts of Y(jω) in the polar plane

B : plot of real part

C : plot of imaginary part

D : Phase angle plot

Q.no 46. If the gain of the system is reduced to a zero value, the roots of the system in the s-plane,

A : Coincide with zero

B : Move away from zero

C : Move away from poles

D : Coincide with the poles

Q.no 47. In a signal flow graph method, how is an overall transfer function of a system obtained?

A : Poisson’s equation

B : Block diagram reduction rules

C : Mason’s formula

D : Lagrange’s equation

Q.no 48. Gain margin of a first or second order system is

A:1

B : 100

C:0

D : Infinity

Q.no 49. If the phase angle at gain crossover frequency is estimated to be -105°, what will be the
value of phase margin of the system?

A : 23°

B : 45°

C : 60°

D : 75°

Q.no 50. Ziegler-Nichol method is used for

A : To optimise frequency response

B : To tune PID Controller

C : To design system components


D : To change system input

Q.no 51. A car is running at a constant speed of 50 km/h, which of the following is the feedback
element for the driver ?

A : Clutch

B : Needle of the speedometer

C : Steering wheel

D : Eyes

Q.no 52. Laplace transform of unit impulse signal is :

A : A/s

B:A

C:1

D : 1/s

Q.no 53. For which systems are the signal flow graphs applicable?

A : Causal

B : Invertible

C : Linear time invariant system

D : Dynamic

Q.no 54. Gain margin expressed in decibels is

A : Positive if Kg greater than 1 and negative for Kg less than 1

B : Negative if Kg greater than 1 and negative for Kg less than 1

C : Always zero

D : Infinity for Kg equal to 1

Q.no 55. A transfer function has two zeroes at infinity. Then the relation between the numerator
(N) and the denominator degree (M) of the transfer function is:

A : N=M+2

B : N=M-2

C : N=M+1

D : N=M-1

Q.no 56. Consider the assertions related to block diagram. Which among them represents the
precise condition? A. Block diagram is used for analysis & design of control system. B. Block
diagram also provides the information regarding the physical construction of the system.

A : A is true, B is false

B : A is false, B is true

C : Both A & B are false

D : Both A & B are true

Q.no 57. Transfer function of potentiometer is


A : Vo(S) / Vin(S)

B : Vin(S) / Vo(S)

C : Vo(S) / Θ(S)

D : Θ(S) / Vo(S)

Q.no 58. A system has poles at 0.01 Hz, 1 Hz and 80Hz, zeroes at 5Hz, 100Hz and 200Hz. The
approximate phase of the system response at 20 Hz is :

A : -90°

B : 0°

C : 90°

D : -180°

Q.no 59. Synchro has

A : One winding on the stator and one winding on rotor

B : Two phase winding on the stator and one winding on rotor

C : Three phase delta connected winding on the stator and one winding on rotor

D : Three phase star connected winding on stator and one winding on rotor

Q.no 60. TF of the compensaor given bellow, A<B represent, if G(S)H(S) = (S+A) / (S+B)

A : Lag Compensator

B : Lead Compensator

C : Lag - Lead Compensator

D : Under Compensator

Q.no 1. In control system, which of the following component will act as error detector

A : Op-Amp

B : Synchro

C : Potentiometer

D : All of above

Q.no 2. The input signals to control systems are not known fully ahead of time, the characteristics
of control system which suddenly strain a control system are:

A : Sudden shock

B : Indeterminate at zero

C : It is zero for time less than zero

D : 1 for all values greater than zero

Q.no 3. The polar plot and nicholas plot can be had from bode plot and

A : Same angle

B : Vice versa
C : Different angle

D : Any of the above

Q.no 4. Synchro error detector has combination of

A : Synchro transmitter and Synchro receiver

B : Synchro control transformer and Synchro receiver

C : Synchro transmitter and Synchro control transformer

D : None of the above

Q.no 5. Transfer founction, when the bode diagram is plotted should be of the form

A : (1+T)

B : (1+S)

C : (Ts)

D : (1+Ts)

Q.no 6. If the constant 'k' is positive, then what would be its contribution on the phase plot?

A : 0°

B : 45°

C : 90°

D : 180°

Q.no 7. Consider the following statement regarding Routh Hurwitz criterion:

A : It gives absolute stability

B : It gives gain and phase margin

C : It gives the number of roots lying in RHS of the s-plane

D : It gives gain, phase margin and number of roots lying in RHS of the s-plane

Q.no 8. Servomotor has Torque - Slip characteristic nature

A : Positive slope Torque Slip characteristic

B : Negative slope Torque Slip characteristic

C : Non linear slope Torque Slip characteristic

D : None of the above

Q.no 9. Armature controlled dc servomotor is represented on the basis of block diagram as

A : Closed loop type

B : Open loop type

C : Hybrid type

D : None of the above

Q.no 10. PD controller affects system performance


A : Increases overshoot of the system

B : Decreases overshoot of the system

C : Overshoot is unaffected

D : Can't say anything

Q.no 11. Equivalent analogy for flux linkages in mechanical rotational system is

A : angular velocity

B : angular displacement

C : torque

D : moment of inertia

Q.no 12. One of the disadvantage of block diagram reduction technique is

A : time consuming

B : For more number of blocks, simplication is difficult

C : both 1 and 2

D : none of these

Q.no 13. The bode plot is applicable to. .....network.

A : Maximum phase

B : Minimum phase

C : All phase

D : Zero phase

Q.no 14. Nyquist criterion determines stability of

A : Open loop system

B : close loop system

C : Open loop as well as close loop system

D : None of these

Q.no 15. In an octave frequency band, the ratio of f2 / f1 is equivalent to

A:2

B:4

C:8

D : 10

Q.no 16. Which one of the following is the most likely reason for large overshoot in a control
system?

A : High gain in a system

B : Presence of dead time delay in a system

C : High positive correcting torque


D : High retarding torque

Q.no 17. The magnitude & phase relationship between input and the steady state
output is called as frequency domain.

A : Step

B : Sinusoidal

C : Ramp

D : Parabolic

Q.no 18. Nyquist plot is

A : symmetrical around real axis

B : symmetrical around imaginary axis

C : not symmetrical

D : symmetrical about both axis

Q.no 19. Phase cross over frequency at which phase is

A : 90

B:0

C : 180

D : -180

Q.no 20. In Nyquist criterion roots of the characteristic equation are given by

A : Zeros of open loop transfer function

B : Zeros of closed loop transfer function

C : Poles of closed loop transfer function

D : Poles of open loop transfer function

Q.no 21. If the roots of the have negative real parts then the response is

A : Stable

B : Unstable

C : Marginally stable

D : Bounded

Q.no 22. RLC network is an example of

A : open loop control system

B : closed loop control system

C : time variant system

D : time invariant system

Q.no 23. Rotor of the ac servomotor is of the nature

A : Drag cup aluminium rotor


B : Squirrel cage rotor

C : Double layer type

D : Projected pole type

Q.no 24. Gain cross over frequency at which magnitude is

A : Zero

B : Infinity

C : Unity

D : constant

Q.no 25. A linear time invariant system is stable if :

A : System in excited by the bounded input, the output is also bounded

B : In the absence of input output tends zero

C : Both a and b

D : System in excited by the bounded input, the output is not bounded

Q.no 26. In feedback control system, with positive value of feedback gain, the overall gain of
the system will

A : decrease

B : increase

C : be unaffected

D : any of the above

Q.no 27. If the initial conditions for a system are inherently zero, what does it physically mean?

A : The system is at rest but stores energy

B : The system is working but does not store energy

C : The system is at rest or no energy is stored in any of its part

D : The system is working with zero reference input

Q.no 28. For Nyquist contour the size of radius is

A:0

B:1

C : infinity

D : 10

Q.no 29. Linear mathematical model applies to :

A : Linear systems

B : Stable systems

C : Unstable systems

D : Non-linear systems
Q.no 30. The nature of transient response is revealed by

A : Sine wave

B : Cos wave

C : Tan wave

D : Test signals

Q.no 31. Lag & Lead electrical networks are generally constructed using

A : Using Resistances

B : Using Inductors

C : Using Capacitors

D : Using Resistance & Capacitors combinations

Q.no 32. If the unity feedback system is given by the open loop transfer function G(s) = ks2 / [(1
+ 0.3s) (1+ 0.05s)], what would be the initial slope of magnitude plot?

A : 20 dB/decade

B : 40 dB/decade

C : 60 dB/decade

D : Unpredictable

Q.no 33. Which of the following statements are correct?

A : Root locus is for the negative feedback systems

B : Complementary root locus is for the positive feedback systems

C : Root locus is for the negative feedback and Complementary root locus is for the positive feedback
systems

D : Complementary root locus is for the negative feedback systems

Q.no 34. The peak percentage overshoot of the closed loop system is :

A : 5 Percent

B : 10 Percent

C : 16.3 Percent

D : 1.63 Percent

Q.no 35. Bode analysis method can be applied

A : If transfer function has no poles and zeros on R.H. of s-plane

B : If transfer function has no poles on R.H. of s-plane

C : f transfer function has no zero on R.H. of s-plane

D : To all transfer functions

Q.no 36. Root locus is used to calculate:

A : Marginal stability
B : Absolute stability

C : Conditional stability

D : Relative stability

Q.no 37. Which type servomotor is generally not used in the practice

A : Armature Controlled DC servomotor

B : AC Servomotor

C : Field Controlled DC Servomotor

D : All of above

Q.no 38. First order system is defined as :

A : Number of poles at origin

B : Order of the differential equation

C : Total number of poles of equation

D : Total number of poles and order of equation

Q.no 39. In frequency reponse of the system if input is having frequency f output frequency will
be

A:f

B : 2f

C:0

D : None of these

Q.no 40. Which unit is adopted for magnitude measurement in Bode plots?

A : Degree

B : Decimal

C : Decibel

D : Deviation

Q.no 41. For open loop poles on imaginary axis Nyquist contour is selcted

A : Complete right half s plane

B : complete right half s plane bypassing poles on imaginary axis

C : complete left half s plane

D : complete left half s plane bypassing imaginary poles

Q.no 42. Which one of the following are correct? The root locus is the path of the roots of the
characteristic equation traced out in the s-plane?

A : As the input of the system is changed

B : As the output of the system is changed

C : As a system parameter is changed


D : As the sensitivity is changed

Q.no 43. PID Controller of electronic circuits has ..............as major component

A : Transister

B : Regulator

C : Op-Amp

D : Triggering circuit

Q.no 44. A close loop control system is stable if the Nyquist plot of the corresponding open loop
transfer function

A : encircles the s plane point (-1+j0) in the counterclockwise direction as many times as the number of
right half s plane poles

B : encircles the s plane point (0-j1) in the clockwise direction as many times as the number of right
half s plane poles

C : encircles the s plane point (-1+j0) in the counterclockwise direction as many times as the number of
left half s plane poles

D : encircles the s plane point (-1+j0) in the clockwise direction as many times as the number of right
half s plane poles

Q.no 45. What is the number of the root locus segments which do not terminate on zeroes?

A : The number of poles

B : The number of zeroes

C : The difference between the number of poles and zeroes

D : The sum of the number of poles and the number of the zeroes

Q.no 46. Consider the following statements regarding root loci:

A : All root loci start from the respective poles of G(s) H(s)

B : All root loci end at the respective zeros of G(s) H(s) or go to infinity

C : The root loci are symmetrical about the imaginary axis of the s-plane

D : All root loci start and end from the respective poles of G(s) H(s) or go to infinity

Q.no 47. The main objective of drawing root locus plot is :

A : To obtain a clear picture about the open loop poles and zeroes of the system

B : To obtain a clear picture about the transient response of feedback system for various values of open
loop gain K

C : To determine sufficient condition for the value of ‘K’ that will make the feedback system unstable

D : Both b and c

Q.no 48. Transient response analysis is done for systems

A : Unstable

B : Stable

C : Conditionally stable

D : Marginally stable
Q.no 49. In polar plots, what does each and every point represent w.r.t magnitude and angle

A : Vector

B : Scalar

C : Phasor

D : Differentiator

Q.no 50. PID controller affects

A : Frequency domain response of the system

B : Time domain response of the system

C : Time damain and frequency domain response of the system at a time

D : Neither Time damain nor frequency domain response of the system

Q.no 51. Due to Phase Lead network, system's

A : Phase margin increases

B : Phase margin decreses

C : No change in phase margin

D : Can't comment

Q.no 52. Assertion (A): It is observed that step function is first derivative of a ramp function and
impulse function is first derivative of a step function. Reason (R): From the derived time
response expression it is concluded that the output time response also follows the same sequence
as that of input functions.

A : Both A and R are true and R is the correct explanation of A

B : Both A and R are true but R is not correct explanation of A

C : Both A is True but R is false

D : Both A is False but R is true

Q.no 53. Banwidth is defined as frequency at which magnitude M(jw) equals to

A : 30dB

B : 3 dB

C : 1 dB

D : 0.67 dB

Q.no 54. Cut off frequency is the frequency at which magnitude of closed loop frequency
response is

A : 1 db below its zero frequency

B : 2 db below its zero frequency

C : 3 db below its zero frequency

D : 4 db below its zero frequency

Q.no 55. Ramp input :

A : Denotes constant velocity


B : Value increases linearly with time

C : It denotes constant velocity and varies linearly with time

D : It varies exponentially with time

Q.no 56. If a system is having damping ratio =0.7 with natural frequency wn=2rad/sec. What will
be the value of resonant frequency

A : 1.428

B : 12

C : 0.2828

D : 1.2828

Q.no 57. If a Nyquist plot of G (jω) H (jω) for a closed loop system passes through (-2, j0) point in
GH plane, what would be the value of gain margin of the system in dB?

A : 0 dB

B : 2.20 dB

C : 4 dB

D : 6.02 dB

Q.no 58. A system with transfer function 1/Ts+1, subjected to a step input takes to seconds to
reach 50% of step height. The value of t is :

A : 6.9 s

B : 10s

C : 14.4s

D : 20

Q.no 59. Cut off is the slope of log-magnitude curve

A : At the start of curve

B : At the end of curve

C : Near the cut off frequency

D : None of these

Q.no 60. Root locus of s(s+2)+K(s+4) =0 is a circle. What are the coordinates of the center of this
circle?

A : -2,0

B : -3,0

C : -4,0

D : -5,0

Q.no 1. The bode plot is a plot relating log w with magnitude in decible and........

A : Phase angle

B : 90 degree

C : 180 degree
D : 45 degree

Q.no 2. The necessary condition for the stability of the linear system is that all the coefficients of
characteristic equation 1+G(s)H(s) = 0, be real and have the :

A : Positive sign

B : Negative sign

C : Same sign

D : Both positive and negative

Q.no 3. Bounded input and Bounded output stability notion concerns with :

A : A system under influence of input

B : A system not under influence of input

C : A system under influence of output

D : A system not under influence of output

Q.no 4. In a signal flow graph, nodes are represented by small

A : Circles

B : Squares

C : Arrows

D : Pointers

Q.no 5. If number of poles are greater than number of zeroes then the system is known as

A : Stable system

B : Unstable system

C : Minimum phase system

D : Non-minimum phase system

Q.no 6. In PID controller, 'K / S' indicates

A : Proportional Controller

B : Integral Controller

C : Differential Controller

D : Feedback Controller

Q.no 7. Generally compensating networks affects

A : Unaltered the performance of the system

B : Completely change out put of the system

C : Time domain performance parameters of the system

D : Frequency domain performance parameters of the system

Q.no 8. Equivalent analogy for current in mechanical translational system is

A : man
B : acceleration

C : velocity

D : displacement

Q.no 9. In frequency domain, response varies with respect to

A : time

B : frequency

C : time, frequency

D : constant

Q.no 10. In DC position controlled system, ................ is used as an error detector

A : Op - Amp

B : Synchro

C : Potntiometer

D : None of the above

Q.no 11. Steady state refers to

A : Error at the steady state

B : Error at the transient state

C : Error at both state

D : Precision

Q.no 12. The capacitance, in force-current analogy, is analogous to

A : momentum

B : velocity

C : displacement

D : mass

Q.no 13. A ramp input applied to a unity feedback system results in 5% steady state error. The
type number and zero frequency gain of the system are respectively

A : 1 and 20

B : 0 and 20

C : 0 and 1/20

D : 1 and 1/20

Q.no 14. How is the sinusoidal transfer function obtained from the system transfer function in
frequency domain?

A : Replacement of jw by s

B : Replacement of s by w

C : Replacement of s by jw
D : Replacement of w by s

Q.no 15. In PID controller, 'K * S' indicates

A : Proportional Controller

B : Integral Controller

C : Differential Controller

D : Feedback Controller

Q.no 16. Constuction wise AC servomotor is a

A : Single phase induction motor

B : Two phase induction motor

C : Three phase induction motor

D : Synchronous motor

Q.no 17. PI controller affects system performance

A : System error will get amplified

B : System will become less oscillatory

C : Order of the system will increase by one

D : Settling time reduces

Q.no 18. For making an unstable system stable:

A : Gain of the system should be increased

B : Gain of the system should be decreased

C : The number of zeroes to the loop transfer function should be increased

D : The number of poles to the loop transfer function should be increased

Q.no 19. To reduce settling time, we have to prefer

A : Proportional Controller

B : Derivative Controller

C : Integral Controller

D : None of the above

Q.no 20. is a quantitative measure of how fast the transients die out in the system.

A : Absolutely stable

B : Conditionally stable

C : Unstable

D : Relative Stability

Q.no 21. What will be the nature of time response if the roots of the characteristic equation are
located on the s-plane imaginary axis?

A : Oscillations
B : Damped oscillations

C : No oscillations

D : Under damped oscillations

Q.no 22. For relative stability of the system which of the following is sufficient?

A : Gain margin

B : Phase margin

C : Both (a) and (b)

D : None of the above

Q.no 23. Which one of the following statements is not correct?

A : Root loci can be used for analyzing stability and transient performance

B : Root loci provide insight into system stability and performance

C : Shape of the root locus gives idea of type of controller needed to meet design specification

D : Root locus can be used to handle more than one variable at a time

Q.no 24. In polar plots, if a pole is added at the origin, what would be the value of the magnitude
at w = 0

A : zero

B : unity

C : infinity

D : unpredictable

Q.no 25. Generally servomotors are located in the .............. of the system

A : Feedback path

B : Error detector

C : Forward path

D : None of the above

Q.no 26. Field controlled dc servomotor is represented on the basis of block diagram as

A : Closed loop type

B : Open loop type

C : Hybrid type

D : None of the above

Q.no 27. The critical value of gain for a system is 40 and gain margin is 6dB. The system is
operating at a gain of:

A : 20

B : 40

C : 80
D : 120

Q.no 28. Consider the following, which statement is true:

A : A system is said to be stable if its output is bounded for any input

B : A system is said to be stable if all the roots of the characteristic equation lie on the left half of the s
plane.

C : A system is said to be stable if all the roots of the characteristic equation have negative real parts.

D : A second order system is always stable for finite values of open loop gain

Q.no 29. The frequency at which the phase of the system acquires is known as 'Phase
crossover frequency'

A : 90°

B : -90°

C : 180°

D : -180°

Q.no 30. An automatic toaster is a classfied as which type 0f control system.

A : open

B : closed

C : partially closed

D : any of the above

Q.no 31. The damping ratio and peak overshoot are measures of:

A : Relative stability

B : Speed of response

C : Steady state error

D : Absolute stability

Q.no 32. Which principle specifies the relationship between enclosure of poles & zeros by s-plane
contour and the encirclement of origin by q(s) plane contour

A : Argument

B : Agrrement

C : Assesment

D : Assortment

Q.no 33. If the gain of the system is reduced to a zero value, the roots of the system in the s-plane,

A : Coincide with zero

B : Move away from zero

C : Move away from poles

D : Coincide with the poles

Q.no 34. Which unit is adopted for phase measurement in Bode plots?
A : Degree

B : Decimal

C : Decibel

D : Deviation

Q.no 35. Gain margin of a first or second order system is

A:1

B : 100

C:0

D : Infinity

Q.no 36. While increasing the value of gain K, the system becomes

A : Less stable

B : More stable

C : Unstable

D : Absolute stable

Q.no 37. According to Nyquist stability criterion, where should be the position of all zeros of q(s)
corresponding to s-plane?

A : Random

B : on right half

C : on left half

D : at the center

Q.no 38. In a signal flow graph method, how is an overall transfer function of a system obtained?

A : Poisson’s equation

B : Block diagram reduction rules

C : Mason’s formula

D : Lagrange’s equation

Q.no 39. The polar plot of a system transfer function passing through point (-1,0) its gain margin
is

A : 1 dB

B : Zero

C : infinity

D : 10 degree

Q.no 40. For Type 1 system what would be magnitude of system at w=0

A:0

B : Infinity
C:K

D : None of these

Q.no 41. If the phase angle at gain crossover frequency is estimated to be -105°, what will be the
value of phase margin of the system?

A : 23°

B : 45°

C : 60°

D : 75°

Q.no 42. Zeroes are defined as:

A : Roots of the denominator of the closed loop transfer function

B : Roots of the numerator of the closed loop transfer function

C : Parts of the numerator

D : Parts of the denominator

Q.no 43. To find system’s response by means of convolution integral of the system
is used.

A : Sum

B : Difference

C : Exponential

D : Weighing

Q.no 44. Generally location of Controllers and Compensators, in the control system is at

A : In the Input side

B : In the output side

C : In the feedback path

D : After error detector and before plat

Q.no 45. A transfer function has two zeroes at infinity. Then the relation between the numerator
(N) and the denominator degree (M) of the transfer function is:

A : N=M+2

B : N=M-2

C : N=M+1

D : N=M-1

Q.no 46. The bode plot is used to analyse which of the following?

A : Minimum phase network

B : Lag lead network

C : Maximum phase network

D : All phase network


Q.no 47. A car is running at a constant speed of 50 km/h, which of the following is the feedback
element for the driver ?

A : Clutch

B : Needle of the speedometer

C : Steering wheel

D : Eyes

Q.no 48. Laplace transform of unit impulse signal is :

A : A/s

B:A

C:1

D : 1/s

Q.no 49. When the number of poles is equal to the number of zeroes, how many branches of root
locus tends towards infinity?

A:1

B:2

C:0

D : Equal to number of zeroes

Q.no 50. By adding a pole at s=0 polar plot of the system will shift

A : at higher and lower frequencies by 90 degree clockwise

B : higher frequencies shift by 90 degree cloclwise

C : higher frequencies by 180 degree clockwise

D : Not change at all

Q.no 51. Due to phase lag network, system's

A : Bandwidth increases

B : Bandwidth decreses

C : bandwith remain unchanged

D : Can't comment

Q.no 52. For which systems are the signal flow graphs applicable?

A : Causal

B : Invertible

C : Linear time invariant system

D : Dynamic

Q.no 53. In connection with servomotor, which of the statement from the following is correction

A : Servomotor is high inertia motor


B : Servomotor has large diameter and small length rotor

C : Servomotor has small diameter and large length rotor

D : Servomotor is slow accelerating and decelerating motor

Q.no 54. Zero of the compensator is dominating, in case of

A : Lag Compensator

B : Lead Compensator

C : Lag - Lead Compensator

D : Under Compensator

Q.no 55. In frequency response, the resonance frequency is basically a measure of of


response.

A : Speed

B : distance

C : Angle

D : Curvature

Q.no 56. Assertion (A): Practical systems must be closed loop system. Reason (R): This is due to
the fact that closed loop systems are least affected by parameter variations, stable, higher
bandwidth, linear and more speed of response.

A : Both A and R are true and R is the correct explanation of A.

B : Both A and R are true but R is not the correct explanation of A.

C : A is true but R is false.

D : A is false but R is true.

Q.no 57. Consider the assertions related to block diagram. Which among them represents the
precise condition? A. Block diagram is used for analysis & design of control system. B. Block
diagram also provides the information regarding the physical construction of the system.

A : A is true, B is false

B : A is false, B is true

C : Both A & B are false

D : Both A & B are true

Q.no 58. Synchro has

A : One winding on the stator and one winding on rotor

B : Two phase winding on the stator and one winding on rotor

C : Three phase delta connected winding on the stator and one winding on rotor

D : Three phase star connected winding on stator and one winding on rotor

Q.no 59. TF of the compensaor given bellow, A<B represent, if G(S)H(S) = (S+A) / (S+B)

A : Lag Compensator

B : Lead Compensator
C : Lag - Lead Compensator

D : Under Compensator

Q.no 60. In a bode magnitude plot, which one of the following slopes would be exhibited at high
frequencies by a 4th order all-pole system?

A : -80dB/decade

B : -40 dB/decade

C : 40 dB/decade

D : 80 dB/decade

Q.no 1. Step signal is the signal whose values is :

A : 1 for all values greater than zero

B : Indeterminate 0 to 10

C : It is zero for time less than 1

D : It is 1 for time less than 0

Q.no 2. Frequency range of bode magnitude and phases are decided by :

A : The lowest and higher important frequencies of dominant factors of the OLTF

B : The lowest and highest important frequencies of all the factors of the open loop transfer function

C : Resonant frequencies of the second factors

D : None of the above

Q.no 3. For the system 2/s+1, the approximate time taken for a step response to reach 98% of its
final value is:

A : 1s

B : 2s

C : 4s

D : 8s

Q.no 4. Proportional controller in the system has effect

A : System Error reduces to the zero

B : Amplify oscillations

C : Increases Settling time

D : Amplify error of the system

Q.no 5. If a pole is located at origin, how does it get represented on the magnitude plot?

A : -10 log (ω) dB

B : -20 log (ω) dB

C : . -40 log (ω) dB

D : -60 log (ω) dB


Q.no 6. The bode plot is applicable to. ..... network.

A : Maximum phase

B : Minimum phase

C : All phase

D : Zero phase

Q.no 7. Equivalent analogy for torque in mechanical rotational system is

A : force

B : man

C : acceleration

D : displacement

Q.no 8. The overall transfer function from block diagram reduction for cascaded blocks is :

A : Sum of individual gain

B : Product of individual gain

C : Difference of individual gain

D : Division of individual gain

Q.no 9. Linear mathematical model applies to :

A : Linear systems

B : Stable systems

C : Unstable systems

D : Non-linear systems

Q.no 10. Rotor of the ac servomotor is of the nature

A : Drag cup aluminium rotor

B : Squirrel cage rotor

C : Double layer type

D : Projected pole type

Q.no 11. Synchro error detector has combination of

A : Synchro transmitter and Synchro receiver

B : Synchro control transformer and Synchro receiver

C : Synchro transmitter and Synchro control transformer

D : None of the above

Q.no 12. Which system conveniently see the impact of poles and zeros on phase and gain
margin?

A : Root locus

B : Nyquist plot
C : Routh-Hurwitz criterion

D : Bode plot

Q.no 13. When deriving the transfer function of a linear element

A : Both initial conditions and loading are taken into account

B : Initial conditions are taken into account but the element is assumed to be not loaded

C : Initial conditions are assumed to be zero but loading is taken into account

D : Initial conditions are assumed to be zero and the element is assumed to be not loaded

Q.no 14. The type of transfer function used in Bode plot is

A : G(s)

B : G(j)

C : G(jw)

D : G(js)

Q.no 15. Equivalent analogy for flux linkages in mechanical rotational system is

A : angular velocity

B : angular displacement

C : torque

D : moment of inertia

Q.no 16. Gain margin is reciprocal of

A : Magitude

B : Phase

C : Magnitude and phase

D : None

Q.no 17. Transfer founction, when the bode diagram is plotted should be of the form

A : (1+T)

B : (1+S)

C : (Ts)

D : (1+Ts)

Q.no 18. The order of the auxiliary polynomial is always:

A : Even

B : Odd

C : May be even or odd

D:0

Q.no 19. RLC network is an example of


A : open loop control system

B : closed loop control system

C : time variant system

D : time invariant system

Q.no 20. Routh Hurwitz criterion gives:

A : Number of roots in the right half of the s-plane

B : Value of the roots

C : Number of roots in the left half of the s-plane

D : Number of roots in the top half of the s-plane

Q.no 21. Asymptotic stability is concerned with:

A : A system under influence of input

B : A system not under influence of input

C : A system under influence of output

D : A system not under influence of output

Q.no 22. PD controller affects system performance

A : Increases overshoot of the system

B : Decreases overshoot of the system

C : Overshoot is unaffected

D : Can't say anything

Q.no 23. If the constant 'k' is positive, then what would be its contribution on the phase plot?

A : 0°

B : 45°

C : 90°

D : 180°

Q.no 24. A linear time invariant system is stable if :

A : System in excited by the bounded input, the output is also bounded

B : In the absence of input output tends zero

C : Both a and b

D : System in excited by the bounded input, the output is not bounded

Q.no 25. In control system, which of the following component will act as error detector

A : Op-Amp

B : Synchro

C : Potentiometer
D : All of above

Q.no 26. Gain cross over frequency at which magnitude is

A : Zero

B : Infinity

C : Unity

D : constant

Q.no 27. Servomotor has Torque - Slip characteristic nature

A : Positive slope Torque Slip characteristic

B : Negative slope Torque Slip characteristic

C : Non linear slope Torque Slip characteristic

D : None of the above

Q.no 28. In an octave frequency band, the ratio of f2 / f1 is equivalent to

A:2

B:4

C:8

D : 10

Q.no 29. Nyquist criterion determines stability of

A : Open loop system

B : close loop system

C : Open loop as well as close loop system

D : None of these

Q.no 30. If the roots of the have negative real parts then the response is

A : Stable

B : Unstable

C : Marginally stable

D : Bounded

Q.no 31. The polar plot of system is

A : locus of the real and imaginary parts of Y(jω) in the polar plane

B : plot of real part

C : plot of imaginary part

D : Phase angle plot

Q.no 32. Which type servomotor is generally not used in the practice

A : Armature Controlled DC servomotor


B : AC Servomotor

C : Field Controlled DC Servomotor

D : All of above

Q.no 33. Number of roots of characteristic equation is equal to the number of

A : Branches

B : Root

C : Stem

D : Poles

Q.no 34. In frequency reponse of the system if input is having frequency f output frequency will
be

A:f

B : 2f

C:0

D : None of these

Q.no 35. The main objective of drawing root locus plot is :

A : To obtain a clear picture about the open loop poles and zeroes of the system

B : To obtain a clear picture about the transient response of feedback system for various values of open
loop gain K

C : To determine sufficient condition for the value of ‘K’ that will make the feedback system unstable

D : Both b and c

Q.no 36. The peak percentage overshoot of the closed loop system is :

A : 5 Percent

B : 10 Percent

C : 16.3 Percent

D : 1.63 Percent

Q.no 37. Lag & Lead electrical networks are generally constructed using

A : Using Resistances

B : Using Inductors

C : Using Capacitors

D : Using Resistance & Capacitors combinations

Q.no 38. Transient response analysis is done for systems

A : Unstable

B : Stable

C : Conditionally stable
D : Marginally stable

Q.no 39. Pole of the compensator is dominating, in case of

A : Lag Compensator

B : Lead Compensator

C : Lag - Lead Compensator

D : Under Compensator

Q.no 40. If the unity feedback system is given by the open loop transfer function G(s) = ks2 / [(1
+ 0.3s) (1+ 0.05s)], what would be the initial slope of magnitude plot?

A : 20 dB/decade

B : 40 dB/decade

C : 60 dB/decade

D : Unpredictable

Q.no 41. Which of the following statements are correct?

A : Root locus is for the negative feedback systems

B : Complementary root locus is for the positive feedback systems

C : Root locus is for the negative feedback and Complementary root locus is for the positive feedback
systems

D : Complementary root locus is for the negative feedback systems

Q.no 42. Which one of the following are correct? The root locus is the path of the roots of the
characteristic equation traced out in the s-plane?

A : As the input of the system is changed

B : As the output of the system is changed

C : As a system parameter is changed

D : As the sensitivity is changed

Q.no 43. Ramp input :

A : Denotes constant velocity

B : Value increases linearly with time

C : It denotes constant velocity and varies linearly with time

D : It varies exponentially with time

Q.no 44. Due to Phase Lead network, system's

A : Phase margin increases

B : Phase margin decreses

C : No change in phase margin

D : Can't comment

Q.no 45. Cut off frequency is the frequency at which magnitude of closed loop frequency
response is
A : 1 db below its zero frequency

B : 2 db below its zero frequency

C : 3 db below its zero frequency

D : 4 db below its zero frequency

Q.no 46. Bode analysis method can be applied

A : If transfer function has no poles and zeros on R.H. of s-plane

B : If transfer function has no poles on R.H. of s-plane

C : f transfer function has no zero on R.H. of s-plane

D : To all transfer functions

Q.no 47. Banwidth is defined as frequency at which magnitude M(jw) equals to

A : 30dB

B : 3 dB

C : 1 dB

D : 0.67 dB

Q.no 48. What is the number of the root locus segments which do not terminate on zeroes?

A : The number of poles

B : The number of zeroes

C : The difference between the number of poles and zeroes

D : The sum of the number of poles and the number of the zeroes

Q.no 49. In polar plots, what does each and every point represent w.r.t magnitude and angle

A : Vector

B : Scalar

C : Phasor

D : Differentiator

Q.no 50. Root locus is used to calculate:

A : Marginal stability

B : Absolute stability

C : Conditional stability

D : Relative stability

Q.no 51. A close loop control system is stable if the Nyquist plot of the corresponding open loop
transfer function

A : encircles the s plane point (-1+j0) in the counterclockwise direction as many times as the number of
right half s plane poles

B : encircles the s plane point (0-j1) in the clockwise direction as many times as the number of right
half s plane poles
C : encircles the s plane point (-1+j0) in the counterclockwise direction as many times as the number of
left half s plane poles

D : encircles the s plane point (-1+j0) in the clockwise direction as many times as the number of right
half s plane poles

Q.no 52. Gain margin expressed in decibels is

A : Positive if Kg greater than 1 and negative for Kg less than 1

B : Negative if Kg greater than 1 and negative for Kg less than 1

C : Always zero

D : Infinity for Kg equal to 1

Q.no 53. Ziegler-Nichol method is used for

A : To optimise frequency response

B : To tune PID Controller

C : To design system components

D : To change system input

Q.no 54. Which unit is adopted for magnitude measurement in Bode plots?

A : Degree

B : Decimal

C : Decibel

D : Deviation

Q.no 55. PID controller affects

A : Frequency domain response of the system

B : Time domain response of the system

C : Time damain and frequency domain response of the system at a time

D : Neither Time damain nor frequency domain response of the system

Q.no 56. If a system is having damping ratio =0.7 with natural frequency wn=2rad/sec. What will
be the value of resonant frequency

A : 1.428

B : 12

C : 0.2828

D : 1.2828

Q.no 57. Cut off is the slope of log-magnitude curve

A : At the start of curve

B : At the end of curve

C : Near the cut off frequency

D : None of these
Q.no 58. Consider a feedback system with gain margin of about 30. At what point does Nyquist
plot crosses negative real axis

A : -3

B : -0.3

C : -30

D : -03

Q.no 59. Transfer function of potentiometer is

A : Vo(S) / Vin(S)

B : Vin(S) / Vo(S)

C : Vo(S) / Θ(S)

D : Θ(S) / Vo(S)

Q.no 60. The main requirement of two potentiometers when acted as error detector is

A : Two potentiometers should be identical

B : Two potentiometers may be different

C : Power rating must be same but potentiometer resistance may be different

D : No any specific condition

A.C. servomotor is basically a

(a) universal motor


(b) single phase induction motor
(c) two phase induction motor
(d) three phase induction motor

Ans: c

Question Which of the following is the nonlinearity caused by servomotor?


A Static friction
B Backlash
C Saturation
D None of the above
Answer C

An automatic toaster is a ______ loop control system.

(a) open
(b) closed
(c) partially closed
(d) any of the above

Ans: a

0 type system has

a) Zero steady state error


b) Small steady state error
c) High gain constant
d) Higher error with high K

Ans: (b)

The transfer function is applicable to which of the following?


a) Linear and time-in variant systems
b) Linear and time-variant systems
c) Linear systems
d) Nonlinear systems
Answer A

To decrease time of the servomechanism

a) Decreases inertia of the system


b) Increases inertia of the system
c) Increases damping of the system
d) Decreases torque of the servomotor

Ans: (a)

Q. If steady state error for type 1 system for unit ramp input is kept constant, then constant output is

a) Distance
b) Velocity
c) Acceleration
d) Power

Ans: (b)

Q. Servomechanism is called a proportional error device when output of the system is function of

a) Error
b) Error and its first derivative
c) First derivative of error
d) None of these

Ans: (a)

Q. For type 2 system, position error arises at steady state when input is

a)Ramp
b)Step displacement
c)Constant acceleration
d)None of these

Ans: (c)

Q. Which of the following motor is suitable for servomechanism?

a) A.c. series motor


b) 1 ɸ induction motor
c) 2 ɸ induction motor
d) 3 ɸ induction motor

Ans: (c)

Q. To decrease the number of system

a) First integrator and then differentiator is inserted


b) First differentiator and then integrator is inserted
c) Only differentiator is inserted in the forward path
d) Only integrator is inserted in the forward path

Ans: (c)

Q.If two blocks having gains A and B respectively are in series connection, find the resultant gain using block diagram reduction
technique?
A+B
A*B
A-B
A/B
ANS=B

On which of the following factors does the sensitivity of a closed loop system to gain changes and load disturbances depend ?
A. Frequency
B. Loop gain
C. Forward gain
D. All of the above
Answer: Option D

In signal flow graph input node is node having only------------


incoming branches.

outgoing branches.

both 1 and 2.
none of the above.
ANS=B

Q.When writing a transfer function which of the following loops are not valid loops?
Self loops at input node.
Self loops at output node.
Both 1 and 2.
Neither 1 nor 2.
ANS=A

Which of the following is/are the characteristics of negative feedback control system?
a) Low sensitivity to parameter variations
b)Reduction in gain at the expense of better stability
c)Rejection of disturbance signals
d)all of the above
ans=d

If two blocks having gains A and B respectively are in parallel connection, find the resultant gain using block diagram reduction
technique?
A+B
A*B
A/B
2(A+B)
ANS=A

In signal flow graph output node is node having only------------


incoming branches
outgoing branches
both 1 and 2
none of the above
ANS=A

Question Which of the following should be done to make an unstable system stable ?
A The gain of the system should be decreased
B The gain of the system should be increased
C The number of poles to the loop transfer function should be increased
D The number of zeros to the loop transfer function should be increased
Answer B

The type of a control system is obtained from -------------- transfer function?


open loop
closed loop
both open loop and closed loop
none of above.
ANS=A

Q.In signal flow graph, the product of all ______gains while going through a forward path is known as 'Path gain'.

a. Branch
b. Path
c. Node
d. Loop

ANSWER: Branch

Q.The value of variables at each node is _________the algebraic sum of all signals arriving at that node.
a. Less than
b. Equal to
c. Greater than
d. None of the above
ANSWER: Equal to

Two loops are said to be non-touching only if no common ______exists between them.
a. Loop
b. Feedback path
c. Branch
d. Node
ANSWER: Node

Associative law for summing point is applicable only to those summing points which are __________connected to each other.
a. Directly
b. Indirectly
c. Orthogonally
d. Diagonally
Answer Explanation
ANSWER: Directly
Q.In a signal flow graph method, how is an overall transfer function of a system obtained?
a. Poisson's equation
b. Block diagram reduction rules
c. Mason's equation
d. Lagrange's equation
ANSWER: Mason's equation

Where are the dummy nodes added in the branch with unity gain?
At input & output nodes
Between chain nodes
Both a and b
None of the above
ANS=A

Effect of feedback on sensitivity is minimum in:


a) Open loop control system
b) Closed loop control system
c) None of the mentioned
d) Both of the mentioned
Answer: b

In case of type-1 system steady state acceleration is


(A) Unity
(B) Infinity
(C) Zero
(D) 10
Answer: Option B

The type 0 system has _______ at the origin.


(A) No pole
(B) Net pole
(C) Simple pole
(D) Two poles
Answer: Option A

In closed loop control system, with positive value of feedback gain the overall gain of the system will
(A) Decrease
(B) Increase
(C) Be unaffected
(D) Any of the above
Answer: Option A

Regenerative feedback implies feedback with


A. oscillations
B. step input
C. negative sign
D. positive sign
Answer: Option D

In case of type-1 system steady state acceleration is


(A) Unity
(B) Infinity
(C) Zero
(D) 10
Answer: Option B

The type 1 system has ______ at the origin.


(a) no pole
(b) net pole
(c) simple pole
(d) two poles
Ans: c

The type 2 system has at the origin.


(a) no net pole
(b) net pole
(c) simple pole
(d) two poles
Ans: d

The position and velocity errors of a type-2 system are


(a) constant, constant
(b) constant, infinity
(c) zero, constant
(d) zero, zero
Ans: c

In case of type-1 system steady state acceleration is


(a) unity
(b) infinity
(c) zero
(d) 10
Ans: b

If a type 0 system is subjected to step input, what is its effect on steady state error?
a. It increases continuously
b. It remains constant
c. It decreases monotonically
d. It gets subjected to another input

For which systems are the signal flow graphs applicable?


a. Causal
b. Invertible
c. Linear time invariant system
d. Dynamic
ANSWER: Linear time invariant system

Two loops are said to be non-touching only if no common ______exists between them.
a. Loop
b. Feedback path
c. Branch
d. Node
ANSWER: Node

Which among the following is represented by a parabolic input signal?


a. Position
b. Force
c. Velocity
d. Acceleration
ANSWER: (d) Acceleration

Which among the following represents an illustration of closed loop system?


a. Automatic washing machine
b. Automatic electric iron
c. Bread toaster
d. Electric hand drier
ANSWER: (b) Automatic electric iron

In a signal flow graph, nodes are represented by small _____


a. Circles
b. Squares
c. Arrows
d. Pointers
ANSWER: (a) Circles

Which type of node comprises incoming as well as outgoing branches?


a. Source node
b. Sink node
c. Chain node
d. Main node
ANSWER: (c) Chain node

Where are the dummy nodes added in the branch with unity gain?
a. At input & output nodes
b. Between chain nodes
c. Both a and b
d. None of the above
ANSWER: (a) At input & output nodes

By which of the following the system response can be tested better ?


A. Ramp input signal
B. Sinusoidal input signal
C. Unit impulse input signal
D. Exponentially decaying signal
Answer: Option C

In case of type-1 system steady state acceleration is


A. unity
B. infinity
C. zero
D. 10
Answer: Option B

Velocity error constant of a system is measured when the input to the system is unit _______ function.
A. parabolic
B. ramp
C. impulse
D. step
Answer: Option B

For drawing root locus, the angle of asymptote yields the direction along which _________branches approach to infinity.
a. p + z
b. p - z
c. p / z
d. p x z
ANSWER: p - z
According to the property of impulse test signal, what is the value of an impulse at t = 0?
a. Zero
b. Unity
c. Infinite
d. Unpredictable
ANSWER: (c) Infinite

What is the value of parabolic input in Laplace domain?


a. 1
b. A/s
c. A/s2
d. A/s3
ANSWER: (d) A/s3

By equating the denominator of transfer function to zero, which among the following will be obtained?
a. Poles
b. Zeros
c. Both a and b
d. None of the above
ANSWER: (a) Poles

Consider that the pole is located at origin and its laplace representation is 1/s. What would be the nature of pole response?
a. Rising exponential
b. Decaying exponential
c. Sinusoidal
d. Constant value
ANSWER: Constant value

For the transfer function given below, where does the zero of the system lie?
G(s) = 5s -1 / s2 + 5s + 4
a. s = -1 & s = -1/4
b. s = -4 & s = -1
c. s = 1/5
d. s = -1/5
ANSWER: (c) s = 1/5

What is the type of closed loop system for the plant transfer function G(s) = k/s²(1+Ts) and with unity feedback?
a. 1
b. 3
c. 2
d.0
ANS=c

If a pole is located at origin, how does it get represented on the magnitude plot?
a. -10 log (ω) dB
b. -20 log (ω) dB
c. -40 log (ω) dB
d. -60 log (ω) dB
ANSWER: -20 log (ω) dB

The steady state error due to ramp input for a type two system is equal to
a. zero.
b.infinite.
c.non zero number.
d.constant.
ans=a

The frequency at which the phase of the system acquires ____ is known as 'Phase crossover frequency'.
a. 90°
b. -90°
c. 180°
d. -180°
ANSWER: -180°

According to the principle of log-scales, if the ratio between two points is same, then the two points get ________equally.
a. United
b. Separated
c. Multiplexed
d. Mixed
ANSWER: Separated

Which plots in frequency domain represent the two separate plots of magnitude and phase against frequency in logarithmic value?
a. Polar plots
b. Bode plots
c. Nyquist plots
d. All of the above
ANSWER: Bode plots

Phase margin of a system is used to specify which of the following ?


A. Frequency response
B. Absolute stability
C. Relative stability
D. Time response
Answer: Option C

In block diagram representation, what do the lines connecting the blocks, known as?
a. Branches
b. Nodes
c. Datums
d. Sources
ANSWER:(a) Branches

In Routh array, if zero is found in the first column, then by which term it needs to be replaced?
a. δ
b. η
c. σ
d. ε
ANSWER: ε

What is the order of a closed loop control system for a plant with a transfer function equal to G(s) = k/s²(1+Ts) and with unity feedback
A. 3
B. 2
C. 1
D. 0
ANS=3

In accordance to relative stability, the settling time exhibits inversely proportional nature to ________parts of roots
a. Real positive
b. Real negative
c. Imaginary positive
d. Imaginary negative
ANSWER: (b) Real negative

Routh Hurwitz criterion gives:


a) Number of roots in the right half of the s-plane
b) Value of the roots
c) Number of roots in the left half of the s-plane
d) Number of roots in the top half of the s-plane
Answer: a

Which of the test signals are best utilized by the stability analysis
a) Impulse
b) Step
c) Ramp
d) Parabolic
Answer: a

A second order control system is defined by the following equation: 4 d²c(t)/dt² + 8 dc(t)/dt + 16 c(t) = 16 u(t) The damping ratio and
natural frequency for this system are respectively
A. 0.25 and 2 rad/s
B. 0.25 and 4 rad/s
C. 0.50 and 2 rad/s
D. 0.50 and 4 rad/s
ANS-C

Routh Hurwitz criterion gives:


a) Number of roots in the right half of the s-plane
b) Value of the roots
c) Number of roots in the left half of the s-plane
d) Number of roots in the top half of the s-plane
Answer: a

What is the number of root locus segments which do not terminate on zeros?
A. The number of poles
B. The number of zeros
C. The difference between the number of holes and the number of zeros
D. The sum of the number of poles and the number of zeros
✔ View Answer
C. The difference between the number of holes and the number of zeros

Frequency response mean


a) Transient response of a system to a sinusoidal input
b) Steady state response of a system to a sinusoidal input
c) Oscillatory response of a system to a sinusoidal input
d) None of these
Ans: (b)

Number of root-locus segment which do not terminate on the zeros is equal to

a) Number of poles
b) Number of zeros
c) Sum of poles and zeros
d) Difference of poles and zeros
Ans: (d)

Bode plot approach is applied to


a) Minimum phase network
b) Non minimum phase network
c) Any network
d) None of these
Ans: (a)

The type of transfer function used in Bode plot is


a) G(s)
b) G(j)
c) G(jw)
d) G(js)
Ans: (a)

90. In a root locus plot, increase in 𝓴 will


a) Increase overshoot of the response
b) Decrease overshoot of the response
c) Not change overshoot of the response
d) None of these
Ans: (a)

91. Bode analysis method can be applied


a) If transfer function has no poles and zeros on R.H. of s-plane
b) If transfer function has no poles on R.H. of s-plane
c) If transfer function has no zero on R.H. of s-plane
d) To all transfer functions
Ans: (a)

92. In a root locus plot, increase in 𝓴 will


a) Result in decrease in the damped and undamped natural frequencies
b) Result in increase in the damped and undamped natural frequencies
c) Not change the damped and undamped natural frequencies
d) None of these
Ans: (b)

93. Cut off is the slope of log-magnitude curve


a) At the start of curve
b) At the end of curve
c) Near the cut off frequency
d) None of these
Ans: (c)

94. Gain margin expressed in decibels is


a) Positive if Kg greater than 1 and negative for Kg less than 1
b) Negative if Kg greater than 1 and negative for Kg less than 1
c) Always zero
d) Infinity for Kg equal to 1
Ans: (a)

95. Bandwidth gives an indication of


a) Characteristic equation of the system
b) Speed of response of a control system
c) Transfer function of the control system
d) Transients in the syste
Ans: (b)

96. Nyquist stability criterion requires polar plot of


a) Characteristic equation
b) Closed loop transfer function
c) Open loop transfer function
d) None of these
Ans: (c)

The bode plot is applicable to.......network.


A. Maximum phase
B. Minimum phase
C. All phase
D. None of the above
✔ View Answer
B. Minimum phase

The bode plot is a plot relating log w with magnitude in decible and........
A. Phase angle
B. 900
C. 1800
D. None of the above
✔ View Answer
A. Phase angle

The polar plot and nicholas plot can be had from bode plot and
A. Same angle
B. Vice versa
C. Different angle
D. Any of the abov
✔ View Answer
B. Vice versa

Cut off frequency is the frequency at which magnitude of closed loop frequency response is
a) 1 db below its zero frequency
b) 2 db below its zero frequency
c) 3 db below its zero frequency
d) 4 db below its zero frequency
Ans: (c)

In Nyquist criterion roots of the characteristic equation are given by


a) Zeros of open loop transfer function
b) Zeros of closed loop transfer function
c) Poles of closed loop transfer function
d) Poles of open loop transfer function
Ans: (c)

Transfer founction, when the bode diagram is plotted should be of the form
a) (1+T)
b) (1+S)
c) (Ts)
d) (1+Ts)
Ans: (d)

101. For relative stability of the system which of the following is sufficient?
a) Gain margin
b) Phase margin
c) Both (a) and (b)
d) None of these
Ans: (c)

Slope in Bode plot is expressed as


a) – 6 db/decade
b) – 6 db/octave
c) – 7 db/octave
d) – 8 db/octave
Ans: (b)

By adding a pole at s = 0, Nyquist plot of the system will


a) Shift 90° clockwise
b) Shift 90° anticlockwise
c) Shift 180°
d) Not change at all
Ans: (a)

Polar plots for+ve and –ve frequencies


a) Are always symmetrical
b) Can never be symmetrical
c) May be symmetrical
d) None of these
Ans: (a)

When the system gain is doubled the gain margin becomes?


a.2 times
b.1/2 times
c.Remains unaffected
d. None of the above
ans=b

If a pole is added to a system it causes


a) Lag compensation
b) Lead compensation
c) Lead-lag compensation
d) None of these
Ans: (b)
Which of the following is the definition of proportional band of a controller ?
A. The range of air output as measured variable varies from maximum to minimum
B. The range of measured variables from set value
C. The range of measured variables through which the air output changes from maximum to minimum
D. Any of the above
E. None of the above
Answer: Option C

In P-D controller, the derivative action plays a significant role in increasing _______ of response.
a. Time
b. Distance
c. Speed
d. Volume
ANSWER: Speed
In order to increase the damping of a badly underdamped system which of following compensators may be used ?
A. Phase-lead
B. Phase-lag
C. Both (A) and (B)
D. Either (A) and (B)
E. None of the above
Answer: Option A

A property of phase lead compensation is that the


A. overshoot is increased.
B.overshoot is increased.
C.bandwidth of closed loop system is reduced.
D.rise time of closed loop system is reduced.
gain margin is reduced.
ANS-C

The reciprocal of proportional band is called:


(A) Reset
(B) Percent
(C) Minutes per repeat
(D) Gain
(E) Rate
Answer : D

What is the relationship between the steady-state error, gain and the tendency of oscillations when the controller is supposed to be under
the proportional action?
a. Steady-state error increases with an increase in gain and oscillation tendency
b. Steady-state error decreases with the decrease in gain and oscillation tendency
c. Steady-state error decreases with an increase in gain and oscillation tendency
d. Steady-state error increases with the decrease in gain and oscillation tendency
ANSWER: c. Steady-state error decreases with an increase in gain and oscillation tendency

A condition where integral control action drives the output of a controller into saturation is called:
(A) self-bias
(B) wind-up
(C) repeat
(D) noise
(E) offset
Answer : B

Question In an open loop system


A Output control the input signal
B Output has no control over input signal
C Some other variable control the input signal
D Neither output nor any other variable has any effect on input
Answer D

Question The transfer function is applicable to which of the following?


A Linear and time-in variant systems
B Linear and time-variant systems
C Linear systems
D Nonlinear systems
Answer A

Question What is the full form of SCADA?


A Supervisory Control and Document Acquisition
B Supervisory Control and Data Acquisition
C Supervisory Column and Data Assessment
D Supervisory Column and Data Assessment
Answer B

Question DCS is a __________________


A Distributed Control System
B Data Control System
C Data Column System
D Distributed Column System
Answer A

Question What is SCADA?


A Software
B Process
C System
D Hardware
Answer B

Question Which of the following are the disadvantages of a closed loop control system?
A Reduces the overall gain.
B Complex and costly.
C Oscillatory response.
D All of the above.
Answer D

Id 7
Question If feedback is introduced in the system the transient response
A Does not very
B Decays very fast
C Decays slowly
D Dies off
Answer B

Question If two blocks having gains A and B respectively are in series connection, find the resultant gain using
block diagram reduction technique?
A A+B
B A*B
C A-B
D A/B
Answer B

Question In signal flow graph input node is node having only------------


A incoming branches.
B outgoing branches.
C both 1 and 2.
D none of the above.
Answer B

Question Which of the following is/are the example of actuator?


A Pneumatic motor
B Hydraulic motor
C All
D Electric motor
Answer C

Question In signal flow graph, the product of all ______gains while going through a forward path is known as 'Path
gain'.
A Branch
B Path
C Node
D Loop
Answer A

Question Two loops are said to be non-touching only if no common ______exists between them.
A Loop
B Feedback path
C Branch
D Node
Answer D

Question While shifting a take-off point after the summing point, which among the following should be added?
A Summing point in series with take-off point
B Summing point in parallel with take-off point
C Block of reciprocal transfer function
D Block of inverse transfer function

Answer A
Question Name test signals used in time response analysis?
A All of B, C, D.
B Unit step.
C Unit ramp.
D Impulse
Answer A

Question By equating the denominator of transfer function to zero, which among the following will be obtained?
A Poles
B Zeros
C Both a and b
D None of the above
Answer A

Question According to the property of impulse test signal, what is the value of an impulse at t = 0?
A Zero
B Unity
C Infinite
D Unpredictable
Answer C

Question If a system is subjected to step input, which type of static error coefficient performs the function of
controlling steady state error?
A Position
B Velocity
C Acceleration
D Retardation
Answer A

Question In second order system, which among the following remains independent of gain (k)?
A Open loop poles
B Closed loop poles
C Both a and b
D None of the above
Answer A

Question For the transfer function given below, where does the zero of the system lie?

G(s) = 5s -1 / s2 + 5s + 4
A s = -1 & s = -1/4
B s = -4 & s = -1
C s = 1/5
D s = -1/5
Answer C

Question If a type 1 system is subjected to parabolic input, what will be the value of steady state error?
A 0
B 100
C Constant K
D INFINITE
Answer D

Question On which factor does the steady state error of the system depend?
A Order
B Type
C Size
D Prototype
Answer B

Question What is the value of parabolic input in Laplace domain?


A 1
B A/s
C A/s2
D A/s3
Answer d
Question Laplace transform of unit impulse signal is :
A A/s
B A
C 1
D 1/s
Answer
Marks
Unit Ignore_this_row

Id 24
Question Which among the following represents an illustration of closed loop system?
A Automatic washing machine
B Automatic electric iron
C Bread toaster
D Electric hand drier
Answer B

Question How is an output represented in the control systems?


A r(t)
B c(t)
C x(t)
D y(t)
Answer B

Question Root locus specifies the movement of closed loop poles especially when the gain of system ________
A Remains constant
B Exhibit variations
C Gives zero feedback
D Gives infinite poles
Answer B

Question If poles are added to the system, where will the system tend to shift the root locus?
A To the left of an imaginary axis
B To the right of an imaginary axis
C At the center
D No shifting takes place
Answer B
Marks
Unit Ignore_this_row

uestion What should be the nature of root locus about the real axis?
A Assymetric
B Symmetric
C Exponential
D Decaying
Answer

Question At which frequency does the magnitude of the system becomes zero dB?
A Resonant frequency
B Cut-off frequency
C Gain crossover frequency
D Phase crossover frequency
Answer C

Question If a pole is located at origin, how does it get represented on the magnitude plot?
A -10 log (ω) dB
B -20 log (ω) dB
C -40 log (ω) dB
D -60 log (ω) dB
Answer B

Question Routh Hurwitz criterion gives:


A Number of roots in the right half of the s-plane
B Value of the roots
C Number of roots in the left half of the s-plane
D Number of roots in the top half of the s-plane
Answer A

Question Consider the following statement regarding Routh Hurwitz criterion:


A It gives absolute stability
B It gives gain and phase margin
C It gives the number of roots lying in RHS of the s-plane
D It gives gain, phase margin and number of roots lying in RHS of the s-plane
Answer D

Question Which unit is adopted for magnitude measurement in Bode plots?


A Degree
B Decimal
C Decibel
D Deviation
Answer C

Question If a pole is added to a system it causes


A Lag compensation
B Lead compensation
C Lead-lag compensation
D None of these
Answer B
Question 0 type system has
A Zero steady state error
B Small steady state error
C High gain constant
D Higher error with high K
Answer B
Marks
Question If gain of the critically damped system is increased, the system will behave as
A Under damped
B Over damped
C Critically damped
D Oscillatory
Answer A

Question System generally preferred is


A Under damped
B Critically damped
C Over damped
D Oscillatory
Answer A
Question In control system non-Linearity caused by gear trains is
A Backlash
B Dead space
C Coulomb friction
D saturation
Answer A
Question If steady state error for type 1 system for unit ramp input is kept constant, then constant output is
A Distance
B Velocity
C Acceleration
D Power
Answer B
Marks
Question A system is called absolutely stable is any oscillations set up in the system are
A Damped out
B Self-sustaining and tend to last indefinitely
C Negative peaked only
D None of these
Answer A
Question Best method to determine stability and transient response of the system is
A Bode plot
B Signal flow graph
C Nyquist plot
D Root locus
Answer C
Marks
Question If poles of system are lying on the imaginary axis in s-plane, the system will be
A Unstable
B Marginally stable
C Conditionally stable
D Unstable
Answer B
Question Which system conveniently see the impact of poles and zeros on phase and gain margin?
A Root locus
B Nyquist plot
C Routh-Hurwitz criterion
D Bode plot
Answer D
Marks
Question Slope in Bode plot is expressed as
A – 6 db/decade
B – 6 db/octave
C – 7 db/octave
D – 8 db/octave
Answer B
Question Relation between Fourier integral and Laplace transformer is through
A Time domain
B Frequency domain
C Both (a) and (b)
D None of these
Answer C
Question For Nyquist contour, the size of radius is _______
A 25
B 0
C 1
D ∞
Answer D
Question According to Nyquist stability criterion, where should be the position of all zeros of q(s) corresponding to
s-plane?
A On left half
B At the center
C On right half
D Random
Answer A
Question If 'ξ' approaches to zero, the peak resonance would ________
A Also be zero
B Be unity
C Tend to infinity
D Become equal to peak overshoot
Answer C
Question Which among the following is a disadvantage of modern control theory?
A Implementation of optimal design
B Transfer function can also be defined for different initial conditions
C Analysis of all systems take place
D Necessity of computational work
Answer D
Question In P-D controller, the derivative action plays a significant role in increasing _______ of response.
A Time
B Distance
C Speed
D Volume
Answer C

1.

The position and velocity errors of a type-2 system are


A. constant, constant

B. constant, infinity

C. zero, constant

D. zero, zero

Answer & Solution Discuss in Board Save for Later

Answer & Solution

Answer: Option C
No explanation is given for this question Let's Discuss on Board
2.

With feed back _____ reduces.


A. system stability

B. system gain

C. system stability and gain

D. none of the above

Answer & Solution Discuss in Board Save for Later

Answer & Solution

Answer: Option B
No explanation is given for this question Let's Discuss on Board
3.

Is a closed loop system.


A. Auto-pilot for an aircraft
B. Direct current generator

C. Car starter

D. Electric switch

Answer & Solution Discuss in Board Save for Later

Answer & Solution

Answer: Option A
No explanation is given for this question Let's Discuss on Board
4.

The effect of error damping is to


A. provide larger settling lime

B. delay the response

C. reduce steady state error

D. any of the above

E. none of the above

Answer & Solution Discuss in Board Save for Later

Answer & Solution

Answer: Option C
No explanation is given for this question Let's Discuss on Board
5.

The band width, in a feedback amplifier.


A. remains unaffected

B. decreases by the same amount as the gain increase

C. increases by the sane saaaajajt as the gain decrease

D. decreases by the same amount as the gain decrease

Answer & Solution Discuss in Board Save for Later

Answer & Solution

Answer: Option C

6.

In an open loop control system


A. Output is independent of control input

B. Output is dependent on control input


C. Only system parameters have effect on the control output

D. None of the above

Answer & Solution Discuss in Board Save for Later

Answer & Solution

Answer: Option A
No explanation is given for this question Let's Discuss on Board
7.

Due to which of the following reasons excessive bond width in control


systems should be avoided ?
A. It leads to slow speed of response

B. It leads to low relative stability

C. Noise is proportional to band width

D. None of the above

Answer & Solution Discuss in Board Save for Later

Answer & Solution

Answer: Option C
No explanation is given for this question Let's Discuss on Board
8.

Hydraulic torque transmission system is analog of


A. amplidyne set

B. resistance-capacitance parallel circuit

C. motor-generator set

D. any of the above

Answer & Solution Discuss in Board Save for Later

Answer & Solution

Answer: Option C
No explanation is given for this question Let's Discuss on Board
9.

If a step function is applied to the input of a system and the output remains
below a certain level for all the time, the system is
A. not necessarily stable

B. stable

C. unstable

D. always unstable
E. any of the above

Answer & Solution Discuss in Board Save for Later

Answer & Solution

Answer: Option A
No explanation is given for this question Let's Discuss on Board
10.

On which of the following factors does the sensitivity of a closed loop


system to gain changes and load disturbances depend ?
A. Frequency

B. Loop gain

C. Forward gain

D. All of the above

Answer & Solution Discuss in Board Save for Later

Answer & Solution

Answer: Option D

11.

By which of the following the control action is determined when a man walks
along a path?
A. Brain

B. Hands

C. Legs

D. Eyes

Answer & Solution Discuss in Board Save for Later

Answer & Solution

Answer: Option D
No explanation is given for this question Let's Discuss on Board
12.

In a control system integral error compensation _______ steady state error


A. increases

B. minimizes

C. does not have any effect on

D. any of the above


Answer & Solution Discuss in Board Save for Later

Answer & Solution

Answer: Option B
No explanation is given for this question Let's Discuss on Board
13.

The type-2 system has ________ at the origin.


A. No net pole

B. Net pole

C. Simple pole

D. Two poles

Answer & Solution Discuss in Board Save for Later

Answer & Solution

Answer: Option D
No explanation is given for this question Let's Discuss on Board
14.

Which of the following should be done to make an unstable system stable ?


A. The gain of the system should be decreased

B. The gain of the system should be increased

C. The number of poles to the loop transfer function should be increased

D. The number of zeros to the loop transfer function should be increased

Answer & Solution Discuss in Board Save for Later

Answer & Solution

Answer: Option B
No explanation is given for this question Let's Discuss on Board
15.

With feedback _____ increases.


A. system stability

B. sensitivity

C. gain

D. effects of disturbing signals

Answer & Solution Discuss in Board Save for Later

Answer & Solution

Answer: Option A
16.

Which of the following is an electromagnetically device ?


A. Induction relay

B. Thermocouple

C. LVDT

D. Any of the above

E. None of the above

Answer & Solution Discuss in Board Save for Later

Answer & Solution

Answer: Option C
No explanation is given for this question Let's Discuss on Board
17.

Which of the following can be measured by LVDT?


A. Displacement

B. Velocity

C. Acceleration

D. Any of the above

Answer & Solution Discuss in Board Save for Later

Answer & Solution

Answer: Option D
No explanation is given for this question Let's Discuss on Board
18.

The frequency and time domain are related through which of the following?
A. Laplace Transform and Fourier Integral

B. Laplace Transform

C. Fourier Integral

D. Either (B) or (C)

Answer & Solution Discuss in Board Save for Later

Answer & Solution

Answer: Option A
No explanation is given for this question Let's Discuss on Board
19.

Addition of zeros in transfer function causes which of the following?


A. Lead-compensation

B. Lag-compensation

C. Lead-lag compensation

D. None of the above

Answer & Solution Discuss in Board Save for Later

Answer & Solution

Answer: Option B
No explanation is given for this question Let's Discuss on Board
20.

In force-voltage analogy, velocity is analogous to


A. current

B. charge

C. inductance

D. capacitance

Answer & Solution Discuss in Board Save for Later

Answer & Solution

Answer: Option A

21.

Which of the following statements is correct for any closed loop system ?
A. All the co-efficients can have zero value

B. All the co-efficients are always non-zero

C. Only one of the static error co-efficients has a finite non-zero value

D. None of the above

Answer & Solution Discuss in Board Save for Later

Answer & Solution

Answer: Option C
No explanation is given for this question Let's Discuss on Board
22.

In a system zero initial condition means that


A. The system is at rest and no energy is stored in any of its components

B. The system is working with zero stored energy

C. The system is working with zero reference signal

D. None of the above

Answer & Solution Discuss in Board Save for Later

Answer & Solution

Answer: Option A
No explanation is given for this question Let's Discuss on Board
23.

Any externally introduced signal affecting the controlled output is called a


A. feedback

B. stimulus

C. signal

D. gain control

Answer & Solution Discuss in Board Save for Later

Answer & Solution

Answer: Option B
No explanation is given for this question Let's Discuss on Board
24.

Technique gives quick transient and stability response


A. Root locus

B. Bode

C. Nyquist

D. Nichols

Answer & Solution Discuss in Board Save for Later

Answer & Solution

Answer: Option A
No explanation is given for this question Let's Discuss on Board
25.

A control system in which the control action is somehow dependent on the


output is known as
A. Closed loop system

B. Semiclosed loop system

C. Open system

D. None of the above

Answer & Solution Discuss in Board Save for Later

Answer & Solution

Answer: Option A

26.

Increases the steady state accuracy.


A. Integrator

B. Differentiator

C. Phase lead compensator

D. Phase lag compensator

Answer & Solution Discuss in Board Save for Later

Answer & Solution

Answer: Option A
No explanation is given for this question Let's Discuss on Board
27.

If the gain of the critical damped system is increased it will behave as


A. oscillatory

B. critically damped

C. overdamped

D. underdamped

E. none of the above

Answer & Solution Discuss in Board Save for Later

Answer & Solution

Answer: Option D
No explanation is given for this question Let's Discuss on Board
28.

In an automatic control system which of the following elements is not used ?


A. Error detector

B. Final control element

C. Sensor

D. Oscillator

Answer & Solution Discuss in Board Save for Later

Answer & Solution

Answer: Option D
No explanation is given for this question Let's Discuss on Board
29.

A car is raining at a constant speed of 50 km/h, which of the following is the


feedback element for the driver ?
A. Clutch

B. Eyes

C. Needle of the speedometer

D. Steering wheel

E. None of the above

Answer & Solution Discuss in Board Save for Later

Answer & Solution

Answer: Option C
No explanation is given for this question Let's Discuss on Board
30.

Which of the following can be measured by the use of a tachogenerator ?


A. Acceleration

B. Speed

C. Speed and acceleration

D. Displacement

E. None of the above

Answer & Solution Discuss in Board Save for Later

Answer & Solution

Answer: Option B
31.

In a system low friction coefficient facilitates


A. Reduced velocity lag error

B. Increased velocity lag error

C. Increased speed of response

D. Reduced time constant of the system

Answer & Solution Discuss in Board Save for Later

Answer & Solution

Answer: Option A
No explanation is given for this question Let's Discuss on Board
32.

As a result of introduction of negative feedback which of the following will


not decrease?
A. Band width

B. Overall gain

C. Distortion

D. Instability

Answer & Solution Discuss in Board Save for Later

Answer & Solution

Answer: Option A
No explanation is given for this question Let's Discuss on Board
33.

an be extended to systems which are time-varying ?


A. Bode-Nyquist stability methods

B. Transfer functions

C. Root locus design

D. State model representatives

Answer & Solution Discuss in Board Save for Later

Answer & Solution

Answer: Option D
No explanation is given for this question Let's Discuss on Board
34.

The term backlash is associated with


A. servomotors

B. induction relays

C. gear trains

D. any of the above

Answer & Solution Discuss in Board Save for Later

Answer & Solution

Answer: Option C
No explanation is given for this question Let's Discuss on Board
35.

A closed loop system is distinguished from open loop system by which of


the following ?
A. Servomechanism

B. Feedback

C. Output pattern

D. Input pattern

Answer & Solution Discuss in Board Save for Later

Answer & Solution

Answer: Option B

36.

Transfer function of a system is used to calculate which of the following ?


A. The order of the system

B. The time constant

C. The output for any given input

D. The steady state gain

Answer & Solution Discuss in Board Save for Later

Answer & Solution

Answer: Option C
No explanation is given for this question Let's Discuss on Board
37.

The second derivative input signals modify which of the following ?


A. The time constant of the system

B. Damping of the system

C. The gain of the system

D. The time constant and suppress the oscillations

E. None of the above

Answer & Solution Discuss in Board Save for Later

Answer & Solution

Answer: Option D
No explanation is given for this question Let's Discuss on Board
38.

In a stable control system backlash can cause which of the following ?


A. Underdamping

B. Overdamping

C. Poor stability at reduced values of open loop gain

D. Low-level oscillations

Answer & Solution Discuss in Board Save for Later

Answer & Solution

Answer: Option D
No explanation is given for this question Let's Discuss on Board
39.

The transfer function is applicable to which of the following ?


A. Linear and time-in variant systems

B. Linear and time-variant systems

C. Linear systems

D. Non-linear systems

E. None of the above

Answer & Solution Discuss in Board Save for Later

Answer & Solution

Answer: Option A
No explanation is given for this question Let's Discuss on Board
40.

Phase margin of a system is used to specify which of the following ?


A. Frequency response

B. Absolute stability

C. Relative stability

D. Time response

Answer & Solution Discuss in Board Save for Later

Answer & Solution

Answer: Option C

41.

In a stable control system saturation can cause which of the following ?


A. Low-level oscillations

B. High-level oscillations

C. Conditional stability

D. Overdamping

Answer & Solution Discuss in Board Save for Later

Answer & Solution

Answer: Option C
No explanation is given for this question Let's Discuss on Board
42.

Pressure error can be measured by which of the following ?


A. Differential bellows and straingauge

B. Selsyn

C. Strain gauge

D. Strain gauge and potentiometer

Answer & Solution Discuss in Board Save for Later

Answer & Solution

Answer: Option A
No explanation is given for this question Let's Discuss on Board
43.

Is the reference input minus the primary feedback.


A. Manipulated variable

B. Zero sequence

C. Actuating signal

D. Primary feedback

Answer & Solution Discuss in Board Save for Later

Answer & Solution

Answer: Option C
No explanation is given for this question Let's Discuss on Board
44.

The transfer function technique is considered as inadequate under which of


the following conditions ?
A. Systems having complexities and non-linearities

B. Systems having stability problems

C. Systems having multiple input disturbances

D. All of the above

Answer & Solution Discuss in Board Save for Later

Answer & Solution

Answer: Option D
No explanation is given for this question Let's Discuss on Board
45.

A control system with excessive noise, is likely to suffer from


A. saturation in amplifying stages

B. loss of gain

C. vibrations

D. oscillations

Answer & Solution Discuss in Board Save for Later

Answer & Solution

Answer: Option A
46.

In open loop system


A. the control action depends on the size of the system

B. the control action depends on system variables

C. the control action depends on the input signal

D. the control action is independent of the output

Answer & Solution Discuss in Board Save for Later

Answer & Solution

Answer: Option D
No explanation is given for this question Let's Discuss on Board
47.

Velocity error constant of a system is measured when the input to the


system is unit _______ function.
A. parabolic

B. ramp

C. impulse

D. step

Answer & Solution Discuss in Board Save for Later

Answer & Solution

Answer: Option B
No explanation is given for this question Let's Discuss on Board
48.

The transient response, with feedback system,


A. rises slowly

B. rises quickly

C. decays slowly

D. decays quickly

Answer & Solution Discuss in Board Save for Later

Answer & Solution

Answer: Option D
No explanation is given for this question Let's Discuss on Board
49.

The initial response when tne output is not equal to input is called
A. Transient response

B. Error response

C. Dynamic response

D. Either of the above

Answer & Solution Discuss in Board Save for Later

Answer & Solution

Answer: Option A
No explanation is given for this question Let's Discuss on Board
50.

Spring constant in force-voltage analogy is analogous to


A. capacitance

B. reciprocal of capacitance

C. current

D. resistance

Answer & Solution Discuss in Board Save for Later

Answer & Solution

Answer: Option B

51.

An automatic toaster is a ______ loop control system.


A. open

B. closed

C. partially closed

D. any of the above

Answer & Solution Discuss in Board Save for Later

Answer & Solution

Answer: Option A
No explanation is given for this question Let's Discuss on Board
52.

Is an open loop control system.


A. Ward Leonard control

B. Field controlled D.C. motor

C. Stroboscope

D. Metadyne

Answer & Solution Discuss in Board Save for Later

Answer & Solution

Answer: Option B
No explanation is given for this question Let's Discuss on Board
53.

For open control system which of the following statements is incorrect ?


A. Less expensive

B. Recalibration is not required for maintaining the required quality of the output

C. Construction is simple and maintenance easy

D. Errors are caused by disturbances

Answer & Solution Discuss in Board Save for Later

Answer & Solution

Answer: Option B
No explanation is given for this question Let's Discuss on Board
54.

A.C. servomotor is basically a


A. universal motor

B. single phase induction motor

C. two phase induction motor

D. three phase induction motor

Answer & Solution Discuss in Board Save for Later

Answer & Solution

Answer: Option C
No explanation is given for this question Let's Discuss on Board
55.

Regenerative feedback implies feedback with


A. oscillations
B. step input

C. negative sign

D. positive sign

Answer & Solution Discuss in Board Save for Later

Answer & Solution

Answer: Option D

56.

By which of the following the system response can be tested better ?


A. Ramp input signal

B. Sinusoidal input signal

C. Unit impulse input signal

D. Exponentially decaying signal

Answer & Solution Discuss in Board Save for Later

Answer & Solution

Answer: Option C
No explanation is given for this question Let's Discuss on Board
57.

Zero initial condition for a system means


A. input reference signal is zero

B. zero stored energy

C. ne initial movement of moving parts

D. system is at rest and no energy is stored in any of its components

Answer & Solution Discuss in Board Save for Later

Answer & Solution

Answer: Option D
No explanation is given for this question Let's Discuss on Board
58.

In closed loop control system, with positive value of feedback gain the
overall gain of the system will
A. decrease

B. increase

C. be unaffected
D. any of the above

Answer & Solution Discuss in Board Save for Later

Answer & Solution

Answer: Option A
No explanation is given for this question Let's Discuss on Board
59.

Is not a final control element.


A. Control valve

B. Potentiometer

C. Electropneumatic converter

D. Servomotor

Answer & Solution Discuss in Board Save for Later

Answer & Solution

Answer: Option B
No explanation is given for this question Let's Discuss on Board
60.

Signal will become zero when the feedback signal and reference signs are
equal.
A. Input

B. Actuating

C. Feedback

D. Reference

Answer & Solution Discuss in Board Save for Later

Answer & Solution

Answer: Option B

61.

Which of the following is the definition of proportional band of a controller ?


A. The range of air output as measured variable varies from maximum to minimum

B. The range of measured variables from set value

C. The range of measured variables through which the air output changes from maximum to minimum

D. Any of the above

E. None of the above


Answer & Solution Discuss in Board Save for Later

Answer & Solution

Answer: Option C
No explanation is given for this question Let's Discuss on Board
62.

has tendency to oscillate.


A. Open loop system

B. Closed loop system

C. Both (A) and (B)

D. Neither (A) nor (B)

Answer & Solution Discuss in Board Save for Later

Answer & Solution

Answer: Option B
No explanation is given for this question Let's Discuss on Board
63.

An amplidyne can give which of the following characteristics ?


A. Constant current

B. Constant voltage

C. Constant current as well as constant voltage

D. Constant current, constant voltage and constant power

E. None of the above

Answer & Solution Discuss in Board Save for Later

Answer & Solution

Answer: Option D
No explanation is given for this question Let's Discuss on Board
64.

In a control system the output of the controller is given to


A. Final control element

B. Amplifier

C. Comparator

D. Sensor

Answer & Solution Discuss in Board Save for Later


Answer & Solution

Answer: Option A
No explanation is given for this question Let's Discuss on Board
65.

technique is not applicable to nonlinear system ?


A. Nyquist Criterion

B. Quasi linearization

C. Functional analysis

D. Phase-plane representation

Answer & Solution Discuss in Board Save for Later

Answer & Solution

Answer: Option A

66.

A phase lag lead network introduces in the output


A. lag at all frequencies

B. lag at high frequencies and lead at low frequencies

C. lag at low frequencies and lead at high frequencies

D. none of the above

Answer & Solution Discuss in Board Save for Later

Answer & Solution

Answer: Option C
No explanation is given for this question Let's Discuss on Board
67.

The capacitance, in force-current analogy, is analogous to


A. momentum

B. velocity

C. displacement

D. mass

Answer & Solution Discuss in Board Save for Later

Answer & Solution

Answer: Option D
No explanation is given for this question Let's Discuss on Board
68.

The viscous friction co-efficient, in force-voltage analogy, is analogous to


A. charge

B. resistance

C. reciprocal of inductance

D. reciprocal of conductance

E. none of the above

Answer & Solution Discuss in Board Save for Later

Answer & Solution

Answer: Option B
No explanation is given for this question Let's Discuss on Board
69.

Mass, in force-voltage analogy, is analogous to


A. charge

B. current

C. inductance

D. resistance

Answer & Solution Discuss in Board Save for Later

Answer & Solution

Answer: Option C
No explanation is given for this question Let's Discuss on Board
70.

A differentiator is usually not a part of a control system because it


A. reduces damping

B. reduces the gain margin

C. increases input noise

D. increases error

Answer & Solution Discuss in Board Save for Later

Answer & Solution

Answer: Option C
1.

A control system working under unknown random actions is called


A. Computer control system

B. Digital data system

C. Stochastic control system

D. Adaptive control system

Answer & Solution Discuss in Board Save for Later

Answer & Solution

Answer: Option C
No explanation is given for this question Let's Discuss on Board
2.

From which of the following transfer function can be obtained ?


A. Signal flow graph

B. Analogous table

C. Output-input ratio

D. Standard block system

E. None of the above

Answer & Solution Discuss in Board Save for Later

Answer & Solution

Answer: Option A
No explanation is given for this question Let's Discuss on Board
3.

Which of the following devices is used for conversion of co-ordinates ?


A. Microsyn

B. Selsyn

C. Synchro-resolver

D. Synchro-transformer

Answer & Solution Discuss in Board Save for Later

Answer & Solution

Answer: Option C
No explanation is given for this question Let's Discuss on Board
4.

Which of the following is an open loop control system ?


A. Field controlled D.C. motor

B. Ward leonard control

C. Metadyne

D. Stroboscope

Answer & Solution Discuss in Board Save for Later

Answer & Solution

Answer: Option A
No explanation is given for this question Let's Discuss on Board
5.

Which of the following is exhibited by Root locus diagrams ?


A. The poles of the transfer function for a set of parameter values

B. The bandwidth of the system

C. The response of a system to a step input

D. The frequency response of a system

E. None of the above

Answer & Solution Discuss in Board Save for Later

Answer & Solution

Answer: Option A

6.

Which of the following devices are commonly used as error detectors in


instruments ?
A. Vernistats

B. Microsyns

C. Resolvers

D. Any of the above

Answer & Solution Discuss in Board Save for Later

Answer & Solution

Answer: Option D
No explanation is given for this question Let's Discuss on Board
7.

Which of the following statements is not necessarily correct for open control
system ?
A. Input command is the sole factor responsible for providing the control action

B. Presence of non-linearities causes malfunctioning

C. Less expensive

D. Generally free from problems of non-linearities

Answer & Solution Discuss in Board Save for Later

Answer & Solution

Answer: Option B
No explanation is given for this question Let's Discuss on Board
8.

The on-off controller is a _____ system.


A. digital

B. linear

C. non-linear

D. discontinuous

Answer & Solution Discuss in Board Save for Later

Answer & Solution

Answer: Option C
No explanation is given for this question Let's Discuss on Board
9.

A controller, essentially, is a
A. sensor

B. clipper

C. comparator

D. amplifier

Answer & Solution Discuss in Board Save for Later

Answer & Solution

Answer: Option C
No explanation is given for this question Let's Discuss on Board
10.

The phase lag produced by transportation relays


A. is independent of frequency

C. increases linearly with frequency

D. decreases linearly with frequency

Answer & Solution Discuss in Board Save for Later

Answer & Solution

Answer: Option C

11.

Directly converts temperature into voltage.


A. Thermocouple

B. Potentiometer

C. Gear train

D. LVDT

E. None of the above

Answer & Solution Discuss in Board Save for Later

Answer & Solution

Answer: Option A
No explanation is given for this question Let's Discuss on Board
12.

Which of the following statements is correct for a system with gain margin
close to unity or a phase margin close to zero ?
A. The system is relatively stable

B. The system is highly stable

C. The system is highly oscillatory

D. None of the above

Answer & Solution Discuss in Board Save for Later

Answer & Solution

Answer: Option C
No explanation is given for this question Let's Discuss on Board
13.

In pneumatic control systems the control valve used as final control element
converts
A. Pressure signal to electric signal

B. Pressure signal to position change

C. Electric signal to pressure signal

D. Position change to pressure signal

Answer & Solution Discuss in Board Save for Later

Answer & Solution

Answer: Option B
No explanation is given for this question Let's Discuss on Board
14.

When the initial conditions of a system are specified to be zero it implies that
the system is
A. At rest without any energy stored in it

B. Working normally with reference input

C. Working normally with zero reference input

D. At rest but stores energy

Answer & Solution Discuss in Board Save for Later

Answer & Solution

Answer: Option D
No explanation is given for this question Let's Discuss on Board
15.

In order to increase the damping of a badly underdamped system which of


following compensators may be used ?
A. Phase-lead

B. Phase-lag

C. Both (A) and (B)

D. Either (A) and (B)

E. None of the above

Answer & Solution Discuss in Board Save for Later

Answer & Solution

Answer: Option A
16.

Static error coefficients are used as a measure of the effectiveness of


closed loop systems for specified ________ input signal.
A. acceleration

B. velocity

C. position

D. all of the above

Answer & Solution Discuss in Board Save for Later

Answer & Solution

Answer: Option D
No explanation is given for this question Let's Discuss on Board
17.

A.C. servomotor resembles


A. two phase induction motor

B. Three phase induction motor

C. direct current series motor

D. universal motor

Answer & Solution Discuss in Board Save for Later

Answer & Solution

Answer: Option A
No explanation is given for this question Let's Discuss on Board
18.

The temperature, under thermal and electrical system analogy, is


considered analogous to
A. voltage

B. current

C. capacitance

D. charge

E. none of the above

Answer & Solution Discuss in Board Save for Later

Answer & Solution

Answer: Option A
No explanation is given for this question Let's Discuss on Board
19.

In electrical pneumatic system analogy the current is considered analogous


to
A. velocity

B. pressure

C. air flow

D. air flow rate

Answer & Solution Discuss in Board Save for Later

Answer & Solution

Answer: Option D
No explanation is given for this question Let's Discuss on Board
20.

In liquid level and electrical system analogy, voltage is considered


analogous to
A. head

B. liquid flow

C. liquid flow rate

D. none of the above

Answer & Solution Discuss in Board Save for Later

Answer & Solution

Answer: Option A

21.

In thermal-electrical analogy charge is considered analogous to


A. heat flow

B. reciprocal of heat flow

C. reciprocal of temperature

D. temperature

E. none of the above

Answer & Solution Discuss in Board Save for Later

Answer & Solution

Answer: Option D
No explanation is given for this question Let's Discuss on Board
22.

An increase in gain, in most systems, leads to


A. smaller damping ratio

B. larger damping ratio

C. constant damping ratio

D. none of the above

Answer & Solution Discuss in Board Save for Later

Answer & Solution

Answer: Option A
No explanation is given for this question Let's Discuss on Board
23.

In case of type-1 system steady state acceleration is


A. unity

B. infinity

C. zero

D. 10

Answer & Solution Discuss in Board Save for Later

Answer & Solution

Answer: Option B
No explanation is given for this question Let's Discuss on Board
24.

The transient response of a system is mainly due to


A. inertia forces

B. internal forces

C. stored energy

D. friction

Answer & Solution Discuss in Board Save for Later

Answer & Solution

Answer: Option C
No explanation is given for this question Let's Discuss on Board
25.

Which of the following is the best method for determining the stability and
transient response ?
A. Root locus

B. Bode plot

C. Nyquist plot

D. None of the above

Answer & Solution Discuss in Board Save for Later

Answer & Solution

Answer: Option A

26.

A conditionally stable system exhibits poor stability at


A. low frequencies

B. reduced values of open loop gain

C. increased values of open loop gain

D. none of the above

Answer & Solution Discuss in Board Save for Later

Answer & Solution

Answer: Option B
No explanation is given for this question Let's Discuss on Board
27.

Is a part of the human temperature control system.


A. Digestive system

B. Perspiration system

C. Ear

D. Leg movement

Answer & Solution Discuss in Board Save for Later

Answer & Solution

Answer: Option B
No explanation is given for this question Let's Discuss on Board
28.

Which of the following is the non-linearity caused by servomotor ?


A. Static friction

B. Backlash

C. Saturation

D. None of the above

Answer & Solution Discuss in Board Save for Later

Answer & Solution

Answer: Option C
No explanation is given for this question Let's Discuss on Board
29.

The output of a feedback control system must be a function of


A. reference and output

B. reference and input

C. input and feedback signal

D. output and feedback signal

Answer & Solution Discuss in Board Save for Later

Answer & Solution

Answer: Option A
No explanation is given for this question Let's Discuss on Board
30.

Which of the following is the input to a controller ?


A. Servo signal

B. Desired variable value

C. Error signal

D. Sensed signal

Answer & Solution Discuss in Board Save for Later

Answer & Solution

Answer: Option A
31.

A signal other than the reference input that tends to affect the value of
controlled variable is known as
A. Disturbance

B. Command

C. Control element

D. Reference input

Answer & Solution Discuss in Board Save for Later

Answer & Solution

Answer: Option A
No explanation is given for this question Let's Discuss on Board
32.

The first order control system, which is well designed, has a


A. small bandwidth

B. negative time constant

C. large negative transfer function pole

D. none of the above

Answer & Solution Discuss in Board Save for Later

Answer & Solution

Answer: Option C
No explanation is given for this question Let's Discuss on Board
33.

A good control system has all the following features except


A. good stability

B. slow response

C. good accuracy

D. sufficient power handling capacity

Answer & Solution Discuss in Board Save for Later

Answer & Solution

Answer: Option B

1. In an open loop control system


(a) Output is independent of control input
(b) Output is dependent on control input
(c) Only system parameters have effect on the control output
(d) None of the above

Ans: a
2. For open control system which of the following statements is incorrect ?
(a) Less expensive
(b) Recalibration is not required for maintaining the required quality of the output
(c) Construction is simple and maintenance easy
(d) Errors are caused by disturbances

Ans: b
3. A control system in which the control action is somehow dependent on the output is known
as
(a) Closed loop system
(b) Semiclosed loop system
(c) Open system
(d) None of the above
Ans: a
4. In closed loop control system, with positive value of feedback gain the overall gain of the
system will
(a) decrease
(b) increase
(c) be unaffected
(d) any of the above

Ans: a
5. Which of the following is an open loop control system ?
(a) Field controlled D.C. motor
(b) Ward leonard control
(c) Metadyne
(d) Stroboscope

Ans: a
6. Which of the following statements is not necessarily correct for open control system ?
(a) Input command is the sole factor responsible for providing the control action
(b) Presence of non-linearities causes malfunctioning
(c) Less expensive
(d) Generally free from problems of non-linearities

Ans: b
7. In open loop system
(a) the control action depends on the size of the system
(b) the control action depends on system variables
(c) the control action depends on the input signal
(d) the control action is independent of the output

Ans: d
8 . ___has tendency to oscillate.
(a) Open loop system
(b) Closed loop system
(c) Both (a) and (b)
(d) Neither (a) nor (b)

Ans: b
9. A good control system has all the following features except
(a) good stability
(b) slow response
(c) good accuracy
(d) sufficient power handling capacity

Ans: b
10. A car is moving at a constant speed of 50 km/h, which of the following is the feedback
element for the driver ?
(a) Clutch
(b) Eyes
(c) Needle of the speedometer
(d) Steering wheel
(e) None of the above

Ans: c
11. The initial response when tune output is not equal to input is called
(a) Transient response
(b) Error response
(c) Dynamic response
(d) Either of the above

Ans: a
12. A control system working under unknown random actions is called
(a) computer control system
(b) digital data system
(c) stochastic control system
(d) adaptive control system

Ans: c
13. An automatic toaster is a ______ loop control system.
(a) open
(b) closed
(c) partially closed
(d) any of the above

Ans: a
14. Any externally introduced signal affecting the controlled output is called a
(a) feedback
(b) stimulus
(c) signal
(d) gain control

Ans: b
15. A closed loop system is distinguished from open loop system by which of the following ?
(a) Servomechanism
(b) Feedback
(c) Output pattern
(d) Input pattern

Ans: b
16. ___ is a part of the human temperature control system.
(a) Digestive system
(b) Perspiration system
(c) Ear
(d) Leg movement

Ans: b
17. By which of the following the control action is determined when a man walks along a path
?
(a) Brain
(b) Hands
(c) Legs
(d) Eyes

Ans: d
18. ___ is a closed loop system.
(a) Auto-pilot for an aircraft
(6) Direct current generator
(c) Car starter
(d) Electric switch
Ans: a
19. Which of the following devices are commonly used as error detectors in instruments ?
(a) Vernistats
(b) Microsyns
(c) Resolvers
(d) Any of the above

Ans: d
20. Which of the following should be done to make an unstable system stable ?
(a) The gain of the system should be decreased
(b) The gain of the system should be increased
(c) The number of poles to the loop transfer function should be increased
(d) The number of zeros to the loop transfer function should be increased

Ans: b
21. ___ increases the steady state accuracy.
(a) Integrator
(b) Differentiator
(c) Phase lead compensator
(d) Phase lag compensator

Ans: a
22. A.C. servomotor resembles
(a) two phase induction motor
(b) Three phase induction motor
(c) direct current series motor
(d) universal motor

Ans: a
23. As a result of introduction of negative feedback which of the following will not decrease ?
(a) Band width
(b) Overall gain
(c) Distortion
(d) Instability

Ans: a
24. Regenerative feedback implies feedback with
(a) oscillations
(b) step input
(c) negative sign
(d) positive sign

Ans: d
25. The output of a feedback control system must be a function of
(a) reference and output
(b) reference and input
(e) input and feedback signal
(d) output and feedback signal

Ans: a
ALSO READ : OPEN LOOP AND CLOSED LOOP ANIMATION
26. ___ is an open loop control system.
(a) Ward Leonard control
(b) Field controlled D.C. motor
(c) Stroboscope
(d) Metadyne

Ans: b
27. A control system with excessive noise, is likely to suffer from
(a) saturation in amplifying stages
(b) loss of gain
(c) vibrations
(d) oscillations
Ans: a
28. Zero initial condition for a system means
(a) input reference signal is zero
(b) zero stored energy
(c) ne initial movement of moving parts
(d) system is at rest and no energy is stored in any of its components

Ans: d
29. Transfer function of a system is used to calculate which of the following ?
(a) The order of the system
(b) The time constant
(c) The output for any given input
(d) The steady state gain

Ans: c
30. The band width, in a feedback amplifier.
(a) remains unaffected
(b) decreases by the same amount as the gain increase
(c) increases by the same amount as the gain decrease
(d) decreases by the same amount as the gain decrease

Ans: c
31. On which of the following factors does the sensitivity of a closed loop system to gain
changes and load disturbances depend ?
(a) Frequency
(b) Loop gain
(c) Forward gain
(d) All of the above

Ans: d
32. The transient response, with feedback system,
(a) rises slowly
(b) rises quickly
(c) decays slowly
(d) decays quickly

Ans: d
33. The second derivative input signals modify which of the following ?
(a) The time constant of the system
(b) Damping of the system
(c) The gain of the system
(d) The time constant and suppress the oscillations
(e) None of the above

Ans: d
34. Which of the following statements is correct for any closed loop system ?
(a) All the co-efficients can have zero value
(6) All the co-efficients are always non-zero
(c) Only one of the static error co-efficients has a finite non-zero value
(d) None of the above

Ans: c
35. Which of the following statements is correct for a system with gain margin close to unity
or a phase margin close to zero ?
(a) The system is relatively stable
(b) The system is highly stable
(c) The system is highly oscillatory
(d) None of the above

Ans: c
36. Due to which of the following reasons excessive bond width in control systems should be
avoided ?
(a) It leads to slow speed of response
(b) It leads to low relative stability
(c) Noise is proportional to band width
(d) None of the above

Ans: c
37. In a stable control system backlash can cause which of the following ?
(a) Underdamping
(b) Overdamping
(c) Poor stability at reduced values of open loop gain
(d) Low-level oscillations

Ans: d
38. In an automatic control system which of the following elements is not used ?
(a) Error detector
(b) Final control element
(c) Sensor
(d) Oscillator

Ans: d
39. In a control system the output of the controller is given to
(a) final control element
(b) amplifier
(c) comparator
(d) sensor
(e) none of the above

Ans: a
40. A controller, essentially, is a
(a) sensor
(b) clipper
(c) comparator
(d) amplifier

Ans: c
41. Which of the following is the not the ideal input to a controller ?
(a) Servo signal
(b) Desired variable value
(c) Error signal
(d) Sensed signal

Ans: a
42. The on-off controller is a _____ system.
(a) digital
(b) linear
(c) non-linear
(d) discontinuous

Ans: d
43. The capacitance, in force-current analogy, is analogous to
(a) momentum
(b) velocity
(c) displacement
(d) mass

Ans: d
44. The temperature, under thermal and electrical system analogy, is considered analogous
to
(a) voltage
(b) current
(c) capacitance
(d) charge
(e) none of the above

Ans: a
45. In electrical-pneumatic system analogy the current is considered analogous to
(a) velocity
(b) pressure
(c) air flow
(d) air flow rate

Ans: d
46. In liquid level and electrical system analogy, voltage is considered analogous to
(a) head
(b) liquid flow
(c) liquid flow rate
(d) none of the above

Ans: a
47. The viscous friction co-efficient, in force-voltage analogy, is analogous to
(a) charge
(b) resistance
(c) reciprocal of inductance
(d) reciprocal of conductance
(e) none of the above

Ans: b
48. In force-voltage analogy, velocity is analogous to
(a) current
(b) charge
(c) inductance
(d) capacitance

Ans: a
49. In thermal-electrical analogy charge is considered analogous to
(a) heat flow
(b) reciprocal of heat flow
(c) reciprocal of temperature
(d) temperature
(e) none of the above

Ans: d
50. Mass, in force-voltage analogy, is analogous to
(a) charge
(b) current
(c) inductance
(d) resistance

Ans: c
ALSO READ : AUTOMATION INTERVIEW QUESTIONS
51. The transient response of a system is mainly due to
(a) inertia forces
(b) internal forces
(c) stored energy
(d) friction

Ans: c
52. ___ Signal will become zero when the feedback signal and reference signs are equal.
(a) Input
(b) Actuating
(c) Feedback
(d) Reference

Ans: b
53. A signal other than the reference input that tends to affect the value of controlled variable
is known as
(a) disturbance
(b) command
(c) control element
(d) reference input

Ans: a
54. The transfer function is applicable to which of the following ?
(a) Linear and time-in variant systems
(b) Linear and time-variant systems
(c) Linear systems
(d) Non-linear systems
(e) None of the above

Ans: a
55. From which of the following transfer function can be obtained ?
(a) Signal flow graph
(b) Analogous table
(c) Output-input ratio
(d) Standard block system
(e) None of the above

Ans: a
56. ___ is the reference input minus the primary feedback.
(a) Manipulated variable
(b) Zero sequence
(c) Actuating signal
(d) Primary feedback

Ans: c
57. The term backlash is associated with
(a) servomotors
(b) induction relays
(c) gear trains
(d) any of the above

Ans:
58. With feedback _____ increases.
(a) system stability
(b) sensitivity
(c) gain
(d) effects of disturbing signals

Ans: a
59. By which of the following the system response can be tested better ?
(a) Ramp input signal
(b) Sinusoidal input signal
(c) Unit impulse input signal
(d) Exponentially decaying signal

Ans: c
60. In a system zero initial condition means that
(a) The system is at rest and no energy is stored in any of its components
(b) The system is working with zero stored energy
(c) The system is working with zero reference signal

Ans: a
61. In a system low friction co-efficient facilitates
(a) reduced velocity lag error
(b) increased velocity lag error
(c) increased speed of response
(d) reduced time constant of the system

Ans: a
62. Hydraulic torque transmission system is analog of
(a) amplidyne set
(b) resistance-capacitance parallel circuit
(c) motor-generator set
(d) any of the above

Ans:
63. Spring constant in force-voltage analogy is analogous to
(a) capacitance
(b) reciprocal of capacitance
(c) current
(d) resistance

Ans: b
64. The frequency and time domain are related through which of the following?
(a) Laplace Transform and Fourier Integral
(b) Laplace Transform
(c) Fourier Integral
(d) Either (b) or (c)

Ans: a
65. An increase in gain, in most systems, leads to
(a) smaller damping ratio
(b) larger damping ratio
(c) constant damping ratio
(d) none of the above

Ans: a
66. Static error co-efficients are used as a measure of the effectiveness of closed loop
systems for specified ________ input signal.
(a) acceleration
(b) velocity
(c) position
(d) all of the above

Ans: d
67. A conditionally stable system exhibits poor stability at
(a) low frequencies
(b) reduced values of open loop gain
(c) increased values of open loop gain
(d) none of the above

Ans: b
68. The type 0 system has ______ at the origin.
(a) no pole
(b) net pole
(c) simple pole
(d) two poles
(e) none of the above

Ans: a
69. The type 1 system has ______ at the origin.
(a) no pole
(b) net pole
(c) simple pole
(d) two poles

Ans: c
70. The type 2 system has at the origin.
(a) no net pole
(b) net pole
(c) simple pole
(d) two poles

Ans: d
71. The position and velocity errors of a type-2 system are
(a) constant, constant
(b) constant, infinity
(c) zero, constant
(d) zero, zero

Ans: c
72. Velocity error constant of a system is measured when the input to the system is unit
_______ function.
(a) parabolic
(b) ramp
(c) impulse
(d) step

Ans: b
73. In case of type-1 system steady state acceleration is
(a) unity
(b) infinity
(c) zero
(d) 10

Ans: b
74. If a step function is applied to the input of a system and the output remains below a
certain level for all the time, the system is
(a) not necessarily stable
(b) stable
(c) unstable
(d) always unstable
(e) any of the above

Ans: a
75. Which of the following is the best method for determining the stability and transient
response ?
(a) Root locus
(b) Bode plot
(c) Nyquist plot
(d) None of the above

Ans: a
76. Phase margin of a system is used to specify which of the following ?
(a) Frequency response
(b) Absolute stability
(c) Relative stability
(d) Time response

Ans: c
77. Addition of zeros in transfer function causes which of the following ?
(a) Lead-compensation
(b) Lag-compensation
(c) Lead-lag compensation
(d) None of the above

Ans: b
78. ___ technique is not applicable to nonlinear system ?
(a) Nyquist Criterion
(b) Quasi linearization
(c) Functional analysis
(d) Phase-plane representation

Ans: a
79. In order to increase the damping of a badly underdamped system which of following
compensators may be used ?
(a) Phase-lead
(b) Phase-lag
(c) Both (a) and (b)
(d) Either (a) and (b)
(e) None of the above

Ans: a
80. The phase lag produced by transportation relays
(a) is independent of frequency
(b) is inverseh’proportional to frequency
(c) increases linearly with frequency
(d) decreases linearly with frequency

Ans: c
ALSO READ : PROGRAMMABLE LOGIC CONTROLLER QUESTIONS
81. In a stable control system saturation can cause which of the following ?
(a) Low-level oscillations
(b) High-level oscillations
(c) Conditional stability
(d) Overdamping

Ans: a
82. Which of the following can be measured by the use of a tacho-generator ?
(a) Acceleration
(b) Speed
(c) Speed and acceleration
(d) Displacement
(e) None of the above

Ans: b
83. ___ is not a final control element.
(a) Control valve
(b) Potentiometer
(c) Electro-pneumatic converter
(d) Servomotor

Ans: b
84. Which of the following is the definition of proportional band of a controller ?
(a) The range of air output as measured variable varies from maximum to minimum
(b) The range of measured variables from set value
(c) The range of measured variables through which the air output chan¬ges from
maximum to minimum
(d) Any of the above
(e) None of the above

Ans: c
85. In pneumatic control systems the control valve used as final control element converts
(a) pressure signal to electric signal
(b) pressure signal to position change
(c) electric signal to pressure signal
(d) position change to pressure signal
(e) none of the above

Ans: b
86. Pressure error can be measured by which of the following ?
(a) Differential bellows and straingauge
(b) Selsyn
(c) Strain gauge
(d) Strain gauge and potentiometer

Ans: a
87. Which of the following devices is used for conversion of co-ordinates ?
(a) Microsyn
(b) Selsyn
(c) Synchro-resolver
(d) Synchro-transformer
Ans: c
88. The effect of error damping is to
(a) provide larger settling lime
(b) delay the response
(c) reduce steady state error
(d) any of the above
(e) none of the above

Ans: c
89. ___ technique gives quick transient and stability response
(a) Root locus
(b) Bode
(c) Nyquist
(d) Nichols

Ans: a
90. A phase lag lead network introduces in the output
(a) lag at all frequencies
(b) lag at high frequencies and lead at low frequencies
(c) lag at low frequencies and lead at high frequencies
(d) none of the above

Ans: c
91. Which of the following is the non-linearity caused by servomotor ?
(a) Static friction
(b) Backlash
(c) Saturation
(d) None of the above

Ans: c
92. ___ can be extended to systems which are time-varying ?
(a) Bode-Nyquist stability methods
(b) Transfer functions
(c) Root locus design
(d) State model representatives

Ans: d
93. When the initial conditions of a system are specified to be zero it implies that the system
is
(a) at rest without any energy stored in it
(b) working normally with reference input
(c) working normally with zero reference input
(d) at rest but stores energy

Ans: d
94. Which of the following is electromechanical device ?
(a) Induction relay
(b) Thermocouple
(c) LVDT
(d) Any of the above
(e) None of the above

Ans: c
95. A differentiator is usually not a part of a control system because it
(a) reduces damping
(b) reduces the gain margin
(c) increases input noise
(d) increases error

Ans: c
96. If the gain of the critical damped system is increased it will behave as
(a) oscillatory
(b) critically damped
(c) overdamped
(d) underdamped
(e) none of the above

Ans: d
97. In a control system integral error compensation _______ steady state error
(a) increases
(b) minimizes
(c) does not have any effect on
(d) any of the above

Ans: b
98. With feed back _____ reduces.
(a) system stability
(6) system gain
(c) system stability and gain
(d) none of the above

Ans: b
99. An amplidyne can give which of the following characteristics ?
(a) Constant current
(b) Constant voltage
(c) Constant current as well as constant voltage
(d) Constant current, constant voltage and constant power
(e) None of the above

Ans: d
100. Which of the following can be measured byLVDT?
(a) Displacement
(b) Velocity
(c) Acceleration
(d) Any of the above

Ans: d
101. ___ directly converts temperature into voltage.
(a) Thermocouple
(b) Potentiometer
(c) Gear train
(d) LVDT
(e) None of the above

Ans: a
102. The transfer function technique is considered as inadequate under which of the following
conditions ?
(a) Systems having complexities and non-linearities
(b) Systems having stability problems
(c) Systems having multiple input dis¬turbances
(d) All of the above

Ans: d
103. Which of the following is the output of a thermocouple ?
(a) Alternating current
(b) Direct current
(c) A.C. voltage
(d) D.C. voltage
(e) None of the above

Ans: d
104. A.C. servomotor is basically a
(a) universal motor
(b) single phase induction motor
(c) two phase induction motor
(d) three phase induction motor

Ans: c
105. The first order control system, which is well designed, has a
(a) small bandwidth
(b) negative time constant
(c) large negative transfer function pole
(d) none of the above

Ans: c
106. Which of the following is exhibited by Root locus diagrams ?
(a) The poles of the transfer function for a set of parameter values
(b) The bandwidth of the system
(c) The response of a system to a step input
(d) The frequency response of a system
(e) None of the above

Ans: a

1.Transient response in the system is basically due to

a) Forces
b) Friction
c) Stored energy
d) Coupling

Ans: (c)

2. Effect of feedback on the plant is to

a) Control system transient response


b) Reduce the sensitivity to plant parameter variations
c) Both (a) and (b)
d) None of these

Ans: (c)

3. Transfer function of a system is defined as the ratio of output to input in

a) Z-transformer
b) Fourier transform
c) Laplace transform
d) All of these

Ans: (c)

4. In an open loop system

a) Output control the input signal


b) Output has no control over input signal
c) Some other variable control the input signal
d) Neither output nor any other variable has any effect on input

Ans: (d)

5. Transfer function of a system can used to study its

a) Steady state behavior


b) Transient behavior
c) Both (a) and (b)
d) None of these

Ans: (c)

6. Electrical resistance is analogous to

a) Intertia
b) Dampers
c) Spring
d) Fluid capacity

Ans: (b)

7. Automatic control system in which output is a variable is called

a) Closed loop system


b) Servomechanism
c) Automatic regulating system
d) Process control system

Ans: (d)

8. Output of the feedback control system should be a function of

a) Input
b) Reference and output
c) Feedback signal
d) None of these

Ans: (b)

9. Steady state error is always zero in response to the displacement input for

a) Type 0 system
b) Type 1 system
c) Type 2 system
d) Type (N > 1) system for N= 0, 1, 2….N

Ans: (d)

10. Relation between Fourier integral and Laplace transformer is through

a) Time domain
b) Frequency domain
c) Both (a) and (b)
d) None of these

Ans: (c)

11. At resonance peak, ratio of output to input is

a) Zero
b) Lowest
c) Highest
d) None of these

Ans: (c)

12. 0 type system has

a) Zero steady state error


b) Small steady state error
c) High gain constant
d) Higher error with high K

Ans: (b)

13. If gain of the system is zero, then the roots

a) Coincide with the poles


b) Move away from the zeros
c) Move away from the poles
d) None of these

Ans: (a)

14. Settling time is inversely proportional to product of the damping ratio and

a) Time constant
b) Maximum overshoot
c) Peak time
d) Undamped natural frequency of the roots

Ans: (b)

15. If gain of the critically damped system is increased, the system will behave as

a) Under damped
b) Over damped
c) Critically damped
d) Oscillatory

Ans: (a)

16. If gain of the system is increased, then

a) Roots move away from the zeros


b) Roots move towards the origin of the S-plot
c) Roots move away from the poles
d) None of these

Ans: (c)

17. A low value of friction coefficient

a) Minimize the velocity lag error


b) Maximize the velocity lag error
c) Minimize the time constant of the system
d) Maximize the time constant of the system
Ans: (a)

18. Physical meaning of zero initial condition is that the

a) System is at rest and stores no energy


b) System is at rest but stores energy
c) Reference input to working system is zero
d) System is working but stores no energy

Ans: (a)

19. If overshoot is excessive, then damping ratio is

a) Equal to 0.4
b) Less than 0.4
c) More than 0.4
d) Infinity

Ans: (b)

20. For a desirable transient response of a second order system damping ratio must be between

a) 0.4 and 0.8


b) 0.8 and 1.0
c) 1.0 and 1.2
d) 1.2 and 1.4

Ans: (a)

21. For second order linear system, setting time is

a) 1/4 of the time constant


b) 1/2 of the time constant
c) 4 times the time constant
d) 2 times the time constant

Ans: (c)

22. System generally preferred is

a) Under damped
b) Critically damped
c) Over damped
d) Oscillatory

Ans: (a)

23. For unity damping factor, the system will be

a) Under damped
b) Critically damped
c) Over damped
d) Oscillatory
Ans: (b)

24. Second-derivative input signal adjust

a) Time constant of the system


b) Time constant and supress the oscillations
c) Damping of the system
d) Gail of the system

Ans: (b)

25. In the derivative error compensation

a) Damping decreases and setting time increases


b) Damping increases and settling time increases
c) Damping decreases and setting time decrease
d) Damping increases and setting time decreases

Ans: (d)

26. If for second order system damping factor is less than one, then system response will be

a) Under damped
b) Over damped
c) Critically damped
d) None of these

Ans: (a)

27. With feedback system sensitivity to parameter

a) Decreases
b) Increases
c) Becomes zero
d) Becomes infinite

Ans: (a)

28. In a closed loop system, source power is modulated with

a) Error signal
b) Reference signal
c) Actuating signal
d) Feed back signal

Ans: (a)

29. With feedback system, transient response

a) Decays constantly
b) Decays slowly
c) Decays quickly
d) Rises fast
Ans: (c)

30. Non-Linearity in the servo system due to saturation is caused by

a) Servo motor
b) Gear trains
c) Relays
d) None of these

Ans: (a)

31. In control system non-Linearity caused by gear trains is

a) Backlash
b) Dead space
c) Coulomb friction
d) saturation

Ans: (a)

32. Time sharing of an expansive control system can be achieved by using a/an

a) a.c. control system


b) analog control system
c) Sampled date control system
d) None of these

Ans: (c)

33. Differential is used in synchro differential unit for generators only

a) Indicating difference of rotation angle of two synchro generators only


b) Indicating sum of rotation angle of two synchro generators only
c) Both (a) and (b)
d) None of these

Ans: (a)

34. Microsyn is the name given to

a) Potentiometer
b) Magnetic amplifier
c) Resolver
d) Rotary differential transformer

Ans: (c)

35. Laplace transformer is not applicable to non-linear system because

a) Non-linear systems are time-varying


b) Time domain analysis is easier than frequency domain analysis
c) Initial conditions are not zero in non-linear systems
d) Superposition law is not applicable to non-linear system
Ans: (d)

36. Value of i(0+) for the system whose transfer function is given by the equation I(s) = (2s+3)/((s+1)(s+3)) is

a) 0
b) 2
c) 1
d) 3

Ans: (b)

37. Liner differential transformer is an

a) Electromechanical device
b) Electrical device
c) Electromagnetic device
d) Electrostatic device

Ans: (a)

38. If transfer function of the system is 1/(TS+1), then steady state error to the unity step input is

a) 1
b) T
c) Zero
d) Infinite

Ans: (c)

39. Which of the following is not a desirable feature of a modern control system?

a) No oscillation
b) Accuracy
c) Quick response
d) Correct power level

Ans: (a)

40. Power amplification in a magnetic amplifier can be increased

a) By negative feed back


b) By positive feed back
c) With higher inductance of a.c. coil
d) None of these

Ans: (d)

41. Device used for conversion of coordinates is

a) Syschros
b) Microsyn
c) Synchro resolver
d) Synchro transformer
Ans: (c)

42. Friction coefficient is usually kept low to

a) Minimize velocity-lag error


b) Maximize velocity-lag error
c) Minimize time constant
d) Maximize speed of response

Ans: (a)

43. Most common use of the synchros is as

a) Error detector
b) Transmission of angular
c) Transmission of arithmetic data
d) For synchronization

Ans: (d)

44. To decrease time of the servomechanism

a) Decreases inertia of the system


b) Increases inertia of the system
c) Increases damping of the system
d) Decreases torque of the servomotor

Ans: (a)

45. If steady state error for type 1 system for unit ramp input is kept constant, then constant output is

a) Distance
b) Velocity
c) Acceleration
d) Power

Ans: (b)

46. Servomechanism is called a proportional error device when output of the system is function of

a) Error
b) Error and its first derivative
c) First derivative of error
d) None of these

Ans: (a)

47. For type 2 system, position error arises at steady state when input is

a) Ramp
b) Step displacement
c) Constant acceleration
d) None of these
Ans: (c)

48. Which of the following motor is suitable for servomechanism?

a) A.c. series motor


b) 1 ɸ induction motor
c) 2 ɸ induction motor
d) 3 ɸ induction motor

Ans: (c)

49. To decrease the number of system

a) First integrator and then differentiator is inserted


b) First differentiator and then integrator is inserted
c) Only differentiator is inserted in the forward path
d) Only integrator is inserted in the forward path

Ans: (c)

50. Servomechanism with step-displacement input is

a) Type 0 system
b) Type 1 system
c) Type 2 system
d) Type 3 system

Ans: (b)

51. If feedback is introduced in the system the transient response

a) Does not very


b) Decays very fast
c) Decays slowly
d) Dies off

Ans: (b)

52. Main difference between servomotor and standard motor is that

a) Servomotor has low inertia and higher starting torque


b) Servomotor has inertia low starting torque
c) Servomotor has high inertia and high starting torque
d) None of these

Ans: (b)

53. The frequency range over which response of the system is within acceptable units is called the system

a) Band width
b) Modulation frequency
c) Demodulation frequency
d) Carrier frequency
Ans: (a)

54. Self balancing instrument uses

a) D.C. servomotor
b) A.C. servomotor
c) Tachometer
d) Magnetic amplifier

Ans: (b)

55. Lead compensation in the system add

a) Zeros
b) Poles
c) Both (a) and (b)
d) None of these

Ans: (b)

56. In type 1 system steady state acceleration error is

a) 0
b) 1
c) Infinity
d) None of these

Ans: (c)

57. Lead lag compensation improve

a) Transient response of the system


b) Steady state response of the system
c) Both (a) and (b)
d) None of these

Ans: (c)

58. If poles of the system are lying on the imaginary axis in s-plane, then system will be

a) Stable
b) Marginally stable
c) Conditionally stable
d) Unstable

Ans: (b)

59. If open loop transfer function of a system is G(s) H(s) = K/(S(1+T 1 S)(1+T2 S)) then system will be

a) Unstable
b) Conditionally stable
c) Stable
d) Marginally stable
Ans: (c)

60. According to Hurwitz criterion the characteristic equation s2+ 8 s3+18 s2+16 s + 5 = 0 is

a) Unstable
b) Marginally stable
c) Conditionally stable
d) Unstable

Ans: (a)

61. A system is called absolutely stable is any oscillations set up in the system are

a) Damped out
b) Self-sustaining and tend to last indefinitely
c) Negative peaked only
d) None of these

Ans: (a)

62. To increase damping of pair of complex roots compensator used is

a) Phase lag
b) Phase lead
c) Phase lag lead
d) One with 60° lead circuit

Ans: (b)

63. Best method to determine stability and transient response of the system is

a) Bode plot
b) Signal flow graph
c) Nyquist plot
d) Root locus

Ans: (c)

64. For type 3 system, lowest frequency asymptote will have the slop of

a) 15 db/octave
b) -16 db/octave
c) 17 db/octave
d) -18 db/octave

Ans: (d)

65. If poles of system are lying on the imaginary axis in s-plane, the system will be

a) Unstable
b) Marginally stable
c) Conditionally stable
d) Unstable
Ans: (b)

66. If a pole is added to a system it causes

a) Lag compensation
b) Lead compensation
c) Lead-lag compensation
d) None of these

Ans: (b)

67. The number of pure integrations in the system transfer function determine

a) Degree of stability
b) Stability of the system
c) Transient performance of the system
d) Steady state performance

Ans: (d)

68. For steady state transient improvement, compensator used is

a) Lead compensator
b) Lag compensator
c) Lead lag compensator
d) None of these

Ans: (c)

69. Which system conveniently see the impact of poles and zeros on phase and gain margin?

a) Root locus
b) Nyquist plot
c) Routh-Hurwitz criterion
d) Bode plot

Ans: (d)

70. Which gives the information between number of poles and zero of the closed loop transfer function?

a) Routh Hurwitz criterion


b) Bode diagram
c) Root locus method
d) Nyquist plot

Ans: (d)

71. Factor which cannot be can cancelled from numerator and denominator of G(s) E(s) in

a) Bode plot
b) Nyquist plot
c) Higher frequencies
d) None of these
Ans: (c)

72. To study time delay of the system which of the following is used?

a) Nyquist plot
b) Bode plot
c) Routh Hurwitz method
d) Nicholas chart

Ans: (a)

73. Intersection of root locus branches with the imaginary axis can be determined by the use of

a) Polar plot
b) Routh’s criterion
c) Nyquist criterion
d) None of these

Ans: (b)

74. Closed loop ples are

a) Zeros of 1+G(S). H(s)


b) Zeros of G(s) H(s)
c) Poles of G(s) H(s)
d) Poles of 1 + G(s) H(s)

Ans: (a)

75. If gain is zero, then

a) Roots move away from zeros


b) Roots coincide with poles
c) Roots move away from poles
d) None of these

Ans: (b)

76. Maximum over shoot is the function of

a) Damping ratio
b) Natural frequency of oscillation
c) Both (a) and (b)
d) Damped frequency of oscillation

Ans: (b)

77. If value of gain is increased, then roots of the system will move to

a) Origin
b) Lower frequencies
c) Higher frequencies
d) None of these
Ans: (c)

78. Feed back control system are basically

a) Low pass filter


b) High pass filter
c) Band pass filter
d) Band stop filter

Ans: (a)

79. In root if 𝓴 is greater than critical value, then increasing 𝓴 will

a) Increase value of the real part of closed loop


b) Decrease value of the real part of closed loop
c) Not change value of the real part of closed loop
d) None of these

Ans: (c)

80. A liner system obeys the principle of

a) Homogeneity
b) Reciprocity
c) Superposition and homogeneity
d) None of these

Ans: (c)

81. Plot of the constant gain loci of the system is

a) Asymptote
b) Circle with centre at the origin
c) Parabola
d) Ellipse

Ans: (b)

82. If poles are more than zeros in G(S) F(S), then number of root locus segment is equal to

a) Number of poles
b) Number of zeros
c) Sum of poles and zeros
d) Difference of poles and zeros

Ans: (a)

83. In root locus technique, angle between adjacent asymptote is

a) 180°/(m + n)
b) 360°/(m + n)
c) 360°/(m - n)
d) 180°/(m - n)
Ans: (c)

84. For G(S) F(S) = (k(S+z))/(S+p), (z < p) the plot is

a) One pole on the imaginary axis


b) One zero on the right-hand side of the plane
c) One pole and one zero on the left-hand side of plane
d) 2 poles and 2 zeros on the left-hand side of plane

Ans: (c)

85. Frequency response mean

a) Transient response of a system to a sinusoidal input


b) Steady state response of a system to a sinusoidal input
c) Oscillatory response of a system to a sinusoidal input
d) None of these

Ans: (b)

86. Number of root-locus segment which do not terminate on the zeros is equal to

a) Number of poles
b) Number of zeros
c) Sum of poles and zeros
d) Difference of poles and zeros

Ans: (d)

87. Bode plot approach is applied to

a) Minimum phase network


b) Non minimum phase network
c) Any network
d) None of these

Ans: (a)

88. In a root locus plot, the increase in 𝓴 will

a) Increase damping ratio


b) Decrease damping ratio
c) Not change damping ratio
d) None of these

Ans: (b)

89. The type of transfer function used in Bode plot is

a) G(s)
b) G(j)
c) G(jw)
d) G(js)
Ans: (a)

90. In a root locus plot, increase in 𝓴 will

a) Increase overshoot of the response


b) Decrease overshoot of the response
c) Not change overshoot of the response
d) None of these

Ans: (a)

91. Bode analysis method can be applied

a) If transfer function has no poles and zeros on R.H. of s-plane


b) If transfer function has no poles on R.H. of s-plane
c) If transfer function has no zero on R.H. of s-plane
d) To all transfer functions

Ans: (a)

92. In a root locus plot, increase in 𝓴 will

a) Result in decrease in the damped and undamped natural frequencies


b) Result in increase in the damped and undamped natural frequencies

c) Not change the damped and undamped natural frequencies


d) None of these

Ans: (b)
93. Cut off is the slope of log-magnitude curve

a) At the start of curve


b) At the end of curve
c) Near the cut off frequency
d) None of these

Ans: (c)

94. Gain margin expressed in decibels is

a) Positive if Kg greater than 1 and negative for Kg less than 1


b) Negative if Kg greater than 1 and negative for Kg less than 1
c) Always zero
d) Infinity for Kg equal to 1

Ans: (a)

95. Bandwidth gives an indication of

a) Characteristic equation of the system


b) Speed of response of a control system
c) Transfer function of the control system
d) Transients in the system
Ans: (b)

96. Nyquist stability criterion requires polar plot of

a) Characteristic equation
b) Closed loop transfer function
c) Open loop transfer function
d) None of these

Ans: (c)

97. Cut off frequency is the frequency at which magnitude of closed loop frequency response is

a) 1 db below its zero frequency


b) 2 db below its zero frequency
c) 3 db below its zero frequency
d) 4 db below its zero frequency

Ans: (c)

98. In Nyquist criterion roots of the characteristic equation are given by

a) Zeros of open loop transfer function


b) Zeros of closed loop transfer function
c) Poles of closed loop transfer function
d) Poles of open loop transfer function

Ans: (c)

99. For all frequencies, a unit circle in the Nyquist plot transformer into

a) Db line of amplitude plot in Bode diagram


b) 1 db line of amplitude plot in Bode diagram
c) Either (a) and (b)
d) None of these

Ans: (c)

100. Transfer founction, when the bode diagram is plotted should be of the form

a) (1+T)
b) (1+S)
c) (Ts)
d) (1+Ts)

Ans: (d)

101. For relative stability of the system which of the following is sufficient?

a) Gain margin
b) Phase margin
c) Both (a) and (b)
d) None of these
Ans: (c)

102. Slope in Bode plot is expressed as

a) – 6 db/decade
b) – 6 db/octave
c) – 7 db/octave
d) – 8 db/octave

Ans: (b)

103. Polar plots for+ve and –ve frequencies

a) Are always symmetrical


b) Can never be symmetrical
c) May be symmetrical
d) None of these

Ans: (a)

104. Gain margin of a first or second order system is

a) Zero
b) 100
c) 1
d) Infinity

Ans: (d)

105. Frequency range over which response of the system is within acceptable limits is called system

a) Modulation frequency
b) Demodulation frequency
c) Carrier frequency
d) Band width

Ans: (d)

106. By adding a pole at s = 0, Nyquist plot of the system will

a) Shift 90° clockwise


b) Shift 90° anticlockwise
c) Shift 180°
d) Not change at all

Ans: (a)

107. A complex-conjugate pair of poles near the jw axis will produce a

a) High oscillatory mode of transient response


b) Steady state mode of response
c) Sinusoidal mode of response
d) None of these
Ans: (a)

1) Which among the following units of PLC is adopted to convey the control plan to CPU?
- Published on 09 Oct 15

a. Memory
b. Power supply unit
c. I/O interface
d. Programming software
Answer Explanation Related Ques

ANSWER: Programming software


Explanation:
No explanation is available for this question!

2) Which architectural unit/block of PLC decides the sequence of different operations to be executed by mean
instructions written in memory?
- Published on 09 Oct 15

a. Memory
b. Programming software
c. I/O interface
d. CPU
Answer Explanation Related Ques

ANSWER: CPU
Explanation:
No explanation is available for this question!

3) Which among the following is a unique model of a system?


- Published on 09 Oct 15

a. Transfer function
b. State variable
c. Both a and b
d. None of the above
Answer Explanation Related Ques

ANSWER: Transfer function


Explanation:
No explanation is available for this question!

4) Which among the following is a disadvantage of modern control theory?


- Published on 09 Oct 15

a. Implementation of optimal design


b. Transfer function can also be defined for different initial conditions
c. Analysis of all systems take place
d. Necessity of computational work
Answer Explanation Related Ques

ANSWER: Necessity of computational work


Explanation:
No explanation is available for this question!

5) According to the property of state transition method, e0 is equal to _____


- Published on 09 Oct 15

a. I
b. A
c. e-At
d. -eAt
Answer Explanation Related Ques

ANSWER: I
Explanation:
No explanation is available for this question!

6) Which mechanism in control engineering implies an ability to measure the state by taking measurements a
- Published on 09 Oct 15

a. Controllability
b. Observability
c. Differentiability
d. Adaptability
Answer Explanation Related Ques

ANSWER: Observability
Explanation:
No explanation is available for this question!

7) State model representation is possible using _________


- Published on 09 Oct 15

a. Physical variables
b. Phase variables
c. Canonical state variables
d. All of the above
Answer Explanation Related Ques

ANSWER: All of the above


Explanation:
No explanation is available for this question!

8) Which among the following constitute the state model of a system in addition to state equations?
- Published on 09 Oct 15

a. Input equations
b. Output equations
c. State trajectory
d. State vector
Answer Explanation Related Ques

ANSWER: Output equations


Explanation:
No explanation is available for this question!
9) Which principle specifies the relationship between enclosure of poles & zeros by s-plane contour and the e
origin by q(s) plane contour?
- Published on 09 Oct 15

a. Argument
b. Agreement
c. Assessment
d. Assortment
Answer Explanation Related Ques

ANSWER: Argument
Explanation:
No explanation is available for this question!

10) If a Nyquist plot of G (jω) H (jω) for a closed loop system passes through (-2, j0) point in GH plane, what w
value of gain margin of the system in dB?
- Published on 09 Oct 15

a. 0 dB
b. 2.0201 dB
c. 4 dB
d. 6.0205 dB
Answer Explanation Related Ques

ANSWER: 6.0205 dB

11) For Nyquist contour, the size of radius is _______


- Published on 09 Oct 15

a. 25
b. 0
c. 1
d. ∞
Answer Explanation Related Ques

ANSWER: ∞
Explanation:
No explanation is available for this question!

12) Consider a feedback system with gain margin of about 30. At what point does Nyquist plot crosses negati
- Published on 09 Oct 15

a. -3
b. -0.3
c. -30
d. -0.03
Answer Explanation Related Ques

ANSWER: -0.3
Explanation:
No explanation is available for this question!

13) According to Nyquist stability criterion, where should be the position of all zeros of q(s) corresponding to
- Published on 09 Oct 15

a. On left half
b. At the center
c. On right half
d. Random
Answer Explanation Related Ques

ANSWER: On left half


Explanation:
No explanation is available for this question!

14) If the system is represented by G(s) H(s) = k (s+7) / s (s +3) (s + 2), what would be its magnitude at ω = ∞?
- Published on 09 Oct 15

a. 0
b. ∞
c. 7/10
d. 21
Answer Explanation Related Ques

ANSWER: 0
Explanation:
No explanation is available for this question!

15) If the unity feedback system is given by the open loop transfer function G(s) = ks 2 / [(1 + 0.3s) (1+ 0.05s)],
the initial slope of magnitude plot?
- Published on 09 Oct 15

a. 20 dB/decade
b. 40 dB/decade
c. 60 dB/decade
d. Unpredictable
Answer Explanation Related Ques

ANSWER: 40 dB/decade
Explanation:
No explanation is available for this question!

16) If the constant 'k' is positive, then what would be its contribution on the phase plot?
- Published on 09 Oct 15

a. 0°
b. 45°
c. 90°
d. 180°
Answer Explanation Related Ques

ANSWER: 0°
Explanation:
No explanation is available for this question!

17) The system is said to be marginally stable, if gain margin is ______


- Published on 09 Oct 15
a. 0
b. 1
c. +∞
d. None of the above
Answer Explanation Related Ques

ANSWER: +∞
Explanation:
No explanation is available for this question!

18) If the phase angle at gain crossover frequency is estimated to be -105°, what will be the value of phase ma
system?
- Published on 09 Oct 15

a. 23°
b. 45°
c. 60°
d. 75°
Answer Explanation Related Ques

ANSWER: 75°
Explanation:
No explanation is available for this question!

19) At which frequency does the magnitude of the system becomes zero dB?
- Published on 09 Oct 15

a. Resonant frequency
b. Cut-off frequency
c. Gain crossover frequency
d. Phase crossover frequency
Answer Explanation Related Ques

ANSWER: Gain crossover frequency


Explanation:
No explanation is available for this question!

20) The frequency at which the phase of the system acquires ____ is known as 'Phase crossover frequency'.
- Published on 09 Oct 15

a. 90°
b. -90°
c. 180°
d. -180°
Answer Explanation Related Ques

ANSWER: -180

21) In frequency response, the resonance frequency is basically a measure of _______ of response.
- Published on 09 Oct 15

a. Speed
b. Distance
c. Angle
d. Curvature
Answer Explanation Related Ques

ANSWER: Speed
Explanation:
No explanation is available for this question!

22) If a system is said to have a damping ξ = 0.5532 with the natural frequency ωn = 2 rad/sec, what will be the
resonant frequency (ωr)?
- Published on 09 Oct 15

a. 1.2456 rad/s
b. 1.7352 rad/s
c. 2.3421 rad/s
d. 3.66 rad/s
Answer Explanation Related Ques

ANSWER: 1.2456 rad/s


Explanation:
No explanation is available for this question!

23) If the resonant peak is estimated to be '5', which among the following would be the correct value of dampi
- Published on 09 Oct 15

a. ξ = 0.3
b. ξ = 1
c. ξ = 3.2
d. ξ = 5.55
Answer Explanation Related Ques

ANSWER: ξ = 0.3
Explanation:
No explanation is available for this question!

24) If the damping of the system becomes equal to zero, which condition of the resonant frequency is likely to
- Published on 09 Oct 15

a. ωr = ωd
b. ωr > ωn
c. ωr < ωn
d. ωr = ωn
Answer Explanation Related Ques

ANSWER: ωr = ω n
Explanation:
No explanation is available for this question!

25) At which condition of 'ξ', resonant peak does not exist and its maximum value is considered to be unity al
resonant frequency?
- Published on 09 Oct 15

a. 0 < ξ < 0.707


b. ξ > 0.707
c. ξ = 0
d. ξ = 1
Answer Explanation Related Ques
ANSWER: ξ > 0.707
Explanation:
No explanation is available for this question!

26) If 'ξ' approaches to zero, the peak resonance would ________


- Published on 09 Oct 15

a. Also be zero
b. Be unity
c. Tend to infinity
d. Become equal to peak overshoot
Answer Explanation Related Ques

ANSWER: Tend to infinity


Explanation:
No explanation is available for this question!

27) For a unity feedback system with G(s) = 10 / s2, what would be the value of centroid?
- Published on 09 Oct 15

a. 0
b. 2
c. 5
d. 10
Answer Explanation Related Ques

ANSWER: 0
Explanation:
No explanation is available for this question!

28) If poles are added to the system, where will the system tend to shift the root locus?
- Published on 09 Oct 15

a. To the left of an imaginary axis


b. To the right of an imaginary axis
c. At the center
d. No shifting takes place
Answer Explanation Related Ques

ANSWER: To the right of an imaginary axis


Explanation:
No explanation is available for this question!

29) If the system is represented by characteristic equation s6 + s4 + s3 + s2 + s + 3 = 0, then the system is_____
- Published on 09 Oct 15

a. Stable
b. Unstable
c. Marginally stable
d. Unpredictable
Answer Explanation Related Ques

ANSWER: Unstable
Explanation:
No explanation is available for this question!
30) Consider the equation S3 + 3s2 + 5s + 2 = 0. How many roots are located in left half of s-plane?
- Published on 09 Oct 15

a. Zero
b. Two
c. Three
d. Four
Answer Explanation Related Ques

ANSWER: Three
31) On which factor does the steady state error of the system depend?
- Published on 09 Oct 15

a. Order

b. Type

c. Size

d. Prototype

Answer Explanation Related Ques

ANSWER: Type
Explanation:
No explanation is available for this question!

32) If a type 1 system is subjected to parabolic input, what will be the value of steady state error?
- Published on 09 Oct 15

a. 0

b. 100

c. Constant k

d. Infinite

Answer Explanation Related Ques

ANSWER: Infinite
Explanation:
No explanation is available for this question!

33) In signal flow graph, the product of all ______gains while going through a forward path is known as 'Path
- Published on 09 Oct 15

a. Branch

b. Path

c. Node

d. Loop

Answer Explanation Related Ques

ANSWER: Branch
Explanation:
No explanation is available for this question!

34) The value of variables at each node is _________the algebraic sum of all signals arriving at that node.
- Published on 09 Oct 15

a. Less than

b. Equal to

c. Greater than

d. None of the above

Answer Explanation Related Ques

ANSWER: Equal to
Explanation:
No explanation is available for this question!

35) At summing point, more than one signal can be added or _________
- Published on 09 Oct 15

a. Subtracted

b. Multiplied

c. Both a and b

d. None of the above

Answer Explanation Related Ques

ANSWER: Subtracted
Explanation:
No explanation is available for this question!

36) Match the following notations with their meanings:

A. G(s) ---------- 1) Laplace of error signal


B. H(s) ---------- 2) Laplace of output signal
C. C(s) ---------- 3) Forward transfer function
D. E(s) ---------- 4) Feedback transfer function
- Published on 09 Oct 15

a. A- 2, B- 3, C- 1, D- 4

b. A- 3, B- 4, C- 2, D- 1

c. A- 2, B- 3, C- 4, D- 1

d. A- 1, B- 2, C- 3, D- 4

Answer Explanation Related Ques

ANSWER: A- 3, B- 4, C- 2, D- 1
Explanation:
No explanation is available for this question!

37) Which among the following are the elements of rotational motion?
- Published on 09 Oct 15

a. Mass, Spring, Friction

b. Inertia, Damper, Spring

c. Work, Energy, Power

d. Force, Pressure, Viscocity

Answer Explanation Related Ques

ANSWER: Inertia, Damper, Spring


Explanation:
No explanation is available for this question!

38) If an impulse response of a system is e-5t, what would be its transfer function?
- Published on 09 Oct 15

a. 1/ s - 5

b. 1/ s + 5

c. (s+1) / (s+5)

d. (s2 - 5s)/ (s-5)

Answer Explanation Related Ques

ANSWER: 1/ s + 5
Explanation:
No explanation is available for this question!

39) What should be the nature of bandwidth for a good control system?
- Published on 09 Oct 15

a. Large

b. Small

c. Medium

d. All of the above

Answer Explanation Related Ques

ANSWER: Large
Explanation:
No explanation is available for this question!

40) Which system exhibits the initiation of corrective action only after the output gets affected?
- Published on 09 Oct 15
a. Feed forward

b. Feedback

c. Both a and b

d. None of the above

Answer Explanation Related Ques

ANSWER: Feedback

41) How many digital inputs are present in PLCs?


- Published on 09 Oct 15

a. 4
b. 8
c. 16
d. 32
Answer Explanation Related Ques

ANSWER: 16
Explanation:
No explanation is available for this question!

42) Which is the correct sequence of operational steps necessary for proper operation of an elevator (lift) con
mechanism?

1. Up switch
2. Stop switch
3. Down switch
4. Start switch
- Published on 09 Oct 15

a. 1-2-3-4
b. 2-1-4-3
c. 4-2-1-3
d. 3-1-2-4
Answer Explanation Related Ques

ANSWER: 4-2-1-3
Explanation:
No explanation is available for this question!

43) In addition to storage instructions, PLC controls __________


- Published on 09 Oct 15

a. Logic sequence timing


b. Counting
c. Arithmetic operations
d. All of the above
Answer Explanation Related Ques

ANSWER: All of the above


Explanation:
No explanation is available for this question!
44) In P-D controller, the derivative action plays a significant role in increasing _______ of response.
- Published on 09 Oct 15

a. Time
b. Distance
c. Speed
d. Volume
Answer Explanation Related Ques

ANSWER: Speed
Explanation:
No explanation is available for this question!

45) Which among the following plays a crucial role in determining the state of dynamic system?
- Published on 09 Oct 15

a. State variables
b. State vector
c. State space
d. State scalar
Answer Explanation Related Ques

ANSWER: State variables


Explanation:
No explanation is available for this question!

46) Which among the following are the interconnected units of state diagram representation?
- Published on 09 Oct 15

a. Scalars
b. Adders
c. Integrators
d. All of the above
Answer Explanation Related Ques

ANSWER: All of the above


Explanation:
No explanation is available for this question!

47) Consider the system represented by the equation given below. What would be the total phase value at ω =
200/ [s3 (s + 3) (s + 6) (s + 10)]
- Published on 09 Oct 15

a. -90°
b. -180°
c. -270°
d. -360°
Answer Explanation Related Ques

ANSWER: -270°
Explanation:
No explanation is available for this question!
48) Due to an addition of pole at origin, the polar plot gets shifted by ___ at ω = 0?
- Published on 09 Oct 15

a. -45°
b. -60°
c. -90°
d. -180°
Answer Explanation Related Ques

ANSWER: -90°
Explanation:
No explanation is available for this question!

49) If a pole is located at origin, how does it get represented on the magnitude plot?
- Published on 09 Oct 15

a. -10 log (ω) dB


b. -20 log (ω) dB
c. -40 log (ω) dB
d. -60 log (ω) dB
Answer Explanation Related Ques

ANSWER: -20 log (ω) dB


Explanation:
No explanation is available for this question!

50) According to the principle of log-scales, if the ratio between two points is same, then the two points get _
- Published on 09 Oct 15

a. United
b. Separated
c. Multiplexed
d. Mixed
Answer Explanation Related Ques

ANSWER: Separated

51) How is the sinusoidal transfer function obtained from the system transfer function in frequency domain?
- Published on 09 Oct 15

a. Replacement of 'jω' by 's'


b. Replacement of 's' by 'ω'
c. Replacement of 's' by 'jω'
d. Replacement of 'ω' by 's'
Answer Explanation Related Ques

ANSWER: Replacement of 's' by 'jω'


Explanation:
No explanation is available for this question!

52) Which plots in frequency domain represent the two separate plots of magnitude and phase against freque
logarithmic value?
- Published on 09 Oct 15

a. Polar plots
b. Bode plots
c. Nyquist plots
d. All of the above
Answer Explanation Related Ques

ANSWER: Bode plots


Explanation:
No explanation is available for this question!

53) If the system is specified by open loop transfer function G(s)H(s) = k / s(s+3) (s + 2), how many root loci pr
infinity?
- Published on 09 Oct 15

a. 2
b. 3
c. 5
d. 6
Answer Explanation Related Ques

ANSWER: 3
Explanation:
No explanation is available for this question!

54) What should be the nature of root locus about the real axis?
- Published on 09 Oct 15

a. Assymetric
b. Symmetric
c. Exponential
d. Decaying
Answer Explanation Related Ques

ANSWER: Symmetric
Explanation:
No explanation is available for this question!

55) Which point on root locus specifies the meeting or collision of two poles?
- Published on 09 Oct 15

a. Centroid
b. Break away point
c. Stability point
d. Anti-break point
Answer Explanation Related Ques

ANSWER: Break away point


Explanation:
No explanation is available for this question!

56) For drawing root locus, the angle of asymptote yields the direction along which _________branches appro
- Published on 09 Oct 15

a. p + z
b. p - z
c. p / z
d. p x z
Answer Explanation Related Ques

ANSWER: p - z
Explanation:
No explanation is available for this question!

57) In a second order system, if the damping ratio is greater than equal to '1', then what would be the nature o
- Published on 09 Oct 15

a. Imaginary
b. Real and equal
c. Real but not equal
d. Complex conjugate
Answer Explanation Related Ques

ANSWER: Real but not equal


Explanation:
No explanation is available for this question!

58) In Routh array, if zero is found in the first column, then by which term it needs to be replaced?
- Published on 09 Oct 15

a. δ
b. η
c. σ
d. ε
Answer Explanation Related Ques

ANSWER: ε
Explanation:
No explanation is available for this question!

59) In accordance to relative stability, the settling time exhibits inversely proportional nature to ________part
- Published on 09 Oct 15

a. Real positive
b. Real negative
c. Imaginary positive
d. Imaginary negative
Answer Explanation Related Ques

ANSWER: Real negative


Explanation:
No explanation is available for this question!

60) Consider that the pole is located at origin and its laplace representation is 1/s. What would be the nature o
response?
- Published on 09 Oct 15

a. Rising exponential
b. Decaying exponential
c. Sinusoidal
d. Constant value
Answer Explanation Related Ques
ANSWER: Constant value

61) If a type 0 system is subjected to step input, what is its effect on steady state error?
- Published on 09 Oct 15

a. It increases continuously
b. It remains constant
c. It decreases monotonically
d. It gets subjected to another input
Answer Explanation Related Ques

ANSWER: It remains constant


Explanation:
No explanation is available for this question!

62) Type 0 systems are unsuitable ___________


- Published on 09 Oct 15

a. For ramp inputs


b. If the input is parabolic in nature
c. Both a and b
d. None of the above
Answer Explanation Related Ques

ANSWER: Both a and b


Explanation:
No explanation is available for this question!

63) Which among the following is represented by a parabolic input signal?


- Published on 09 Oct 15

a. Position
b. Force
c. Velocity
d. Acceleration
Answer Explanation Related Ques

ANSWER: Acceleration
Explanation:
No explanation is available for this question!

64) In time domain system, which response has its existence even after an extinction of transient response?
- Published on 09 Oct 15

a. Step response
b. Impulse response
c. Steady state response
d. All of the above
Answer Explanation Related Ques

ANSWER: Steady state response


Explanation:
No explanation is available for this question!
65) Two loops are said to be non-touching only if no common ______exists between them.
- Published on 09 Oct 15

a. Loop
b. Feedback path
c. Branch
d. Node
Answer Explanation Related Ques

ANSWER: Node
Explanation:
No explanation is available for this question!

66) For which systems are the signal flow graphs applicable?
- Published on 09 Oct 15

a. Causal
b. Invertible
c. Linear time invariant system
d. Dynamic
Answer Explanation Related Ques

ANSWER: Linear time invariant system


Explanation:
No explanation is available for this question!

67) While solving signal flow graph using Mason's gain equation, what does the second letter in two subscrip
stand for?
- Published on 09 Oct 15

a. Serial number of loop


b. Parallel number of loop
c. Number of touching loops
d. Number of non-touching loops
Answer Explanation Related Ques

ANSWER: Number of non-touching loops


Explanation:
No explanation is available for this question!

68) In a signal flow graph method, how is an overall transfer function of a system obtained?
- Published on 09 Oct 15

a. Poisson's equation
b. Block diagram reduction rules
c. Mason's equation
d. Lagrange's equation
Answer Explanation Related Ques

ANSWER: Mason's equation


Explanation:
No explanation is available for this question!

69) In block diagram representation, what do the lines connecting the blocks, known as?
- Published on 09 Oct 15

a. Branches
b. Nodes
c. Datums
d. Sources
Answer Explanation Related Ques

ANSWER: Branches
Explanation:
No explanation is available for this question!

70) For the elimination of feedback loops, the derivation based on transfer function of ______ loop is used.
- Published on 09 Oct 15

a. Open
b. Closed
c. Both a and b
d. None of the above
Answer Explanation Related Ques

ANSWER: Closed
71) Associative law for summing point is applicable only to those summing points which are __________conn
other.
- Published on 09 Oct 15

a. Directly

b. Indirectly

c. Orthogonally

d. Diagonally

Answer Explanation Related Ques

ANSWER: Directly
Explanation:
No explanation is available for this question!

72) If finite number of blocks are connected in series or cascade configuration, then how are the blocks comb
algebraically?
- Published on 09 Oct 15

a. By addition

b. By multiplication

c. By differentiation

d. By integration

Answer Explanation Related Ques

ANSWER: By multiplication
Explanation:
No explanation is available for this question!
73) The fundamental function of a tachometer is the conversion of angular _______ into voltage
- Published on 09 Oct 15

a. Velocity

b. Displacement

c. Acceleration

d. Current

Answer Explanation Related Ques

ANSWER: Velocity
Explanation:
No explanation is available for this question!

74) Laplace transform of an impulse response is regarded as _____function of the system


- Published on 09 Oct 15

a. Analytic

b. Parabolic

c. Transfer

d. Hypothetical

Answer Explanation Related Ques

ANSWER: Transfer
Explanation:
No explanation is available for this question!

75) If a signal is passed through an integrator, it _____the amplitude of noise signal.


- Published on 09 Oct 15

a. Enhances

b. Reduces

c. Stabilizes

d. Factorizes

Answer Explanation Related Ques

ANSWER: Reduces
Explanation:
No explanation is available for this question!

76) For the transfer function given below, where does the zero of the system lie?

G(s) = 5s -1 / s2 + 5s + 4
- Published on 09 Oct 15

a. s = -1 & s = -1/4
b. s = -4 & s = -1

c. s = 1/5

d. s = -1/5

Answer Explanation Related Ques

ANSWER: s = 1/5
Explanation:
No explanation is available for this question!

77) A good control system should be sensitive to __________


- Published on 09 Oct 15

a. Internal disturbances

b. Environmental parameters

c. Parametric variations

d. Input signals (except noise)

Answer Explanation Related Ques

ANSWER: Input signals (except noise)


Explanation:
No explanation is available for this question!

78) What is the value of steady state error in closed loop control systems?
- Published on 09 Oct 15

a. Zero

b. Unity

c. Infinity

d. Unpredictable

Answer Explanation Related Ques

ANSWER: Zero
Explanation:
No explanation is available for this question!

79) Into which energy signal does the position sensor convert the measured position of servomotor in servom
- Published on 09 Oct 15

a. Mechanical

b. Electrical

c. Thermal

d. Light

Answer Explanation Related Ques

ANSWER: Electrical
Explanation:
No explanation is available for this question!

80) Which among the following controls the speed of D.C. motor?
- Published on 09 Oct 15

a. Galvanometer

b. Gauss meter

c. Potentiometer

d. Tachometer

Answer Explanation Related Ques

ANSWER: Tachometer
81) In P-I controller, what does an integral of a function compute?
- Published on 07 Oct 15

a. Density of curve

b. Area under the curve

c. Volume over the curve

d. Circumference of curve

Answer Explanation Related Ques

ANSWER: Area under the curve


Explanation:
No explanation is available for this question!

82) Which controller has the potential to eliminate/overcome the drawback of offset in proportional controller
- Published on 07 Oct 15

a. P-I

b. P-D

c. Both a and b

d. None of the above

Answer Explanation Related Ques

ANSWER: P-I
Explanation:
No explanation is available for this question!

83) Which time is responsible for introducing an error in the temperature regulation of applications associate
controllers?
- Published on 07 Oct 15

a. Rise time

b. Dead time
c. Switching time

d. Decay time

Answer Explanation Related Ques

ANSWER: Dead time


Explanation:
No explanation is available for this question!

84) If an error signal e(t) of an ON-OFF controller is found to be greater than zero, what would be its output?
- Published on 07 Oct 15

a. 10%

b. 50%

c. 80%

d. 100%

Answer Explanation Related Ques

ANSWER: 100%
Explanation:
No explanation is available for this question!

85) State space analysis is applicable even if the initial conditions are _____
- Published on 07 Oct 15

a. Zero

b. Non-zero

c. Equal

d. Not equal

Answer Explanation Related Ques

ANSWER: Non-zero
Explanation:
No explanation is available for this question!

86) Conventional control theory is applicable to ______ systems


- Published on 07 Oct 15

a. SISO

b. MIMO

c. Time varying

d. Non-linear

Answer Explanation Related Ques

ANSWER: SISO
Explanation:
No explanation is available for this question!

87) In polar plots, if a pole is added at the origin, what would be the value of the magnitude at Ω = 0?
- Published on 07 Oct 15

a. Zero

b. Infinity

c. Unity

d. Unpredictable

Answer Explanation Related Ques

ANSWER: Infinity
Explanation:
No explanation is available for this question!

88) In polar plots, what does each and every point represent w.r.t magnitude and angle?
- Published on 07 Oct 15

a. Scalar

b. Vector

c. Phasor

d. Differentiator

Answer Explanation Related Ques

ANSWER: Phasor
Explanation:
No explanation is available for this question!

89) In an octave frequency band, the ratio of f2 / f1 is equivalent to ________


- Published on 07 Oct 15

a. 2

b. 4

c. 8

d. 10

Answer Explanation Related Ques

ANSWER: 2
Explanation:
No explanation is available for this question!

90) Which unit is adopted for magnitude measurement in Bode plots?


- Published on 07 Oct 15
a. Degree

b. Decimal

c. Decibel

d. Deviation

Answer Explanation Related Ques

ANSWER: Decibel
91) If a linear system is subjected to an input r(t) = Asin(ωt), what output will be generated?
- Published on 07 Oct 15

a. c(t) = B sin (ωt + Φ)

b. c(t) = B cos (ωt + Φ)

c. c(t) = B tan (ωt + Φ)

d. c(t) = B cot (ωt + Φ)

Answer Explanation Related Ques

ANSWER: c(t) = B sin (ωt + Φ)


Explanation:
No explanation is available for this question!

92) The magnitude & phase relationship between _________input and the steady state output is called as freq
- Published on 07 Oct 15

a. Step

b. Ramp

c. Sinusoidal

d. Parabolic

Answer Explanation Related Ques

ANSWER: Sinusoidal
Explanation:
No explanation is available for this question!

93) While specifying the angle and magnitude conditions, angles are added whereas magnitudes get _______
- Published on 07 Oct 15

a. Subtracted

b. Multiplied

c. Divided

d. All of the above

Answer Explanation Related Ques

ANSWER: Multiplied
Explanation:
No explanation is available for this question!
94) Which condition is used to verify the existence of a particular point on the root locus?
- Published on 07 Oct 15

a. Amplitude

b. Frequency

c. Magnitude

d. Angle

Answer Explanation Related Ques

ANSWER: Angle
Explanation:
No explanation is available for this question!

95) Root locus specifies the movement of closed loop poles especially when the gain of system ________
- Published on 07 Oct 15

a. Remains constant

b. Exhibit variations

c. Gives zero feedback

d. Gives infinite poles

Answer Explanation Related Ques

ANSWER: Exhibit variations


Explanation:
No explanation is available for this question!

96) In second order system, which among the following remains independent of gain (k)?
- Published on 07 Oct 15

a. Open loop poles

b. Closed loop poles

c. Both a and b

d. None of the above

Answer Explanation Related Ques

ANSWER: Open loop poles


Explanation:
No explanation is available for this question!

97) If a pole is located at s = -5 in left-hand plane (LHP), how will it be represented in Laplace domain?
- Published on 07 Oct 15

a. 1/ s + 5
b. 1/ s - 5

c. s/ s + 5

d. s/ s - 5

Answer Explanation Related Ques

ANSWER: 1/ s + 5
Explanation:
No explanation is available for this question!

98) Which among the following are solely responsible in determining the speed of response of control system
- Published on 07 Oct 15

a. Poles

b. Zeros

c. Speed of input

d. All of the above

Answer Explanation Related Ques

ANSWER: Poles
Explanation:
No explanation is available for this question!

99) If the complex conjugate poles are located at RHP, what would be the nature of corresponding impulse re
- Published on 07 Oct 15

a. Exponential

b. Damping oscillations

c. Increasing amplifier

d. Constant amplitude oscillations

Answer Explanation Related Ques

ANSWER: Increasing amplifier


Explanation:
No explanation is available for this question!

100) On the basis of an output response, into how many parts can the s-plane be divided?
- Published on 07 Oct 15

a. 2

b. 3

c. 4

d. 6

Answer Explanation Related Ques

ANSWER: 3
101) If a system is subjected to step input, which type of static error coefficient performs the function of contr
state error?
- Published on 07 Oct 15

a. Position

b. Velocity

c. Acceleration

d. Retardation

Answer Explanation Related Ques

ANSWER: Position
Explanation:
No explanation is available for this question!

102) Which among the following is/are an/the illustration/s of a sinusoidal input?
- Published on 07 Oct 15

a. Setting the temperature of an air conditioner

b. Input given to an elevator

c. Checking the quality of speakers of music system

d. All of the above

Answer Explanation Related Ques

ANSWER: Checking the quality of speakers of music system


Explanation:
No explanation is available for this question!

103) What is the value of parabolic input in Laplace domain?


- Published on 07 Oct 15

a. 1

b. A/s

c. A/s2

d. A/s3

Answer Explanation Related Ques

ANSWER: A/s3
Explanation:
No explanation is available for this question!

104) According to the property of impulse test signal, what is the value of an impulse at t = 0?
- Published on 07 Oct 15

a. Zero

b. Unity
c. Infinite

d. Unpredictable

Answer Explanation Related Ques

ANSWER: Infinite
Explanation:
No explanation is available for this question!

105) Where are the dummy nodes added in the branch with unity gain?
- Published on 07 Oct 15

a. At input & output nodes

b. Between chain nodes

c. Both a and b

d. None of the above

Answer Explanation Related Ques

ANSWER: At input & output nodes


Explanation:
No explanation is available for this question!

106) Which type of node comprises incoming as well as outgoing branches?


- Published on 07 Oct 15

a. Source node

b. Sink node

c. Chain node

d. Main node

Answer Explanation Related Ques

ANSWER: Chain node


Explanation:
No explanation is available for this question!

107) According to signal flow graph, which among the following represents the relationship between nodes b
between them?
- Published on 07 Oct 15

a. Branch

b. Self-loop

c. Semi-node

d. Mesh

Answer Explanation Related Ques

ANSWER: Branch
Explanation:
No explanation is available for this question!

108) In a signal flow graph, nodes are represented by small _____


- Published on 07 Oct 15

a. Circles

b. Squares

c. Arrows

d. Pointers

Answer Explanation Related Ques

ANSWER: Circles
Explanation:
No explanation is available for this question!

109) Consider the assertions related to block diagram. Which among them represents the precise condition?

A. Block diagram is used for analysis & design of control system.


B. Block diagram also provides the information regarding the physical construction of the system.
- Published on 07 Oct 15

a. A is true, B is false

b. A is false, B is true

c. Both A & B are true

d. Both A & B are false

Answer Explanation Related Ques

ANSWER: A is true, B is false


Explanation:
No explanation is available for this question!

110) While shifting a take-off point after the summing point, which among the following should be added?
- Published on 07 Oct 15

a. Summing point in series with take-off point

b. Summing point in parallel with take-off point

c. Block of reciprocal transfer function

d. Block of inverse transfer function

Answer Explanation Related Ques

ANSWER: Summing point in series with take-off point

111) In a parallel combination, the direction of flow of signals through blocks in parallel must resemble to the
- Published on 07 Oct 15

a. Forward
b. Feedback
c. Opposite
d. Diagonal
Answer Explanation Related Ques

ANSWER: Forward
Explanation:
No explanation is available for this question!

112) The output signal is fed back at the input side from the _________point
- Published on 07 Oct 15

a. Summing
b. Differential
c. Take-off
d. All of the above
Answer Explanation Related Ques

ANSWER: Take-off
Explanation:
No explanation is available for this question!

113) By equating the denominator of transfer function to zero, which among the following will be obtained?
- Published on 07 Oct 15

a. Poles
b. Zeros
c. Both a and b
d. None of the above
Answer Explanation Related Ques

ANSWER: Poles
Explanation:
No explanation is available for this question!

114) Basically, poles of transfer function are the laplace transform variable values which causes the transfer f
become ___________
- Published on 07 Oct 15

a. Zero
b. Unity
c. Infinite
d. Average value
Answer Explanation Related Ques

ANSWER: Infinite
Explanation:
No explanation is available for this question!

115) The output is said to be zero state response because ______conditions are made equal to zero.
- Published on 07 Oct 15
a. Initial
b. Final
c. Steady state
d. Impulse response
Answer Explanation Related Ques

ANSWER: Initial
Explanation:
No explanation is available for this question!

116) How is an output represented in the control systems?


- Published on 07 Oct 15

a. r(t)
b. c(t)
c. x(t)
d. y(t)
Answer Explanation Related Ques

ANSWER: c(t)
Explanation:
No explanation is available for this question!

117) Which among the following represents an illustration of closed loop system?
- Published on 07 Oct 15

a. Automatic washing machine


b. Automatic electric iron
c. Bread toaster
d. Electric hand drier
Answer Explanation Related Ques

ANSWER: Automatic electric iron


Explanation:
No explanation is available for this question!

118) Which notation represents the feedback path in closed loop system representation?
- Published on 07 Oct 15

a. b(t)
b. c(t)
c. e(t)
d. r(t)
Answer Explanation Related Ques

ANSWER: b(t)
Explanation:
No explanation is available for this question!

119) Which among the following is not an advantage of an open loop system?
- Published on 07 Oct 15

a. Simplicity in construction & design


b. Easy maintenance
c. Rare problems of stability
d. Requirement of system recalibration from time to time
Answer Explanation Related Ques

ANSWER: Requirement of system recalibration from time to time


Explanation:
No explanation is available for this question!

120) Which terminology deals with the excitation or stimulus applied to the system from an external source fo
generation of an output?
- Published on 07 Oct 15

a. Input signal
b. Output signal
c. Error signal
d. Feedback signal
Answer Explanation Related Ques

ANSWER: Input signal

1. A control system working under unknown random actions is called

(A) Computer control system


(B) Digital data system
(C) Stochastic control system
(D) Adaptive control system
Correct Answer

2. By which of the following the control action is determined when a man walks along a path?
(A) Brain
(B) Hands
(C) Legs
(D) Eyes
Correct Answer

3. As a result of introduction of negative feedback which of the following will not decrease?
(A) Band width
(B) Overall gain
(C) Distortion
(D) Instability
Correct Answer

4. In a control system the output of the controller is given to

(A) Final control element


(B) Amplifier
(C) Comparator
(D) Sensor
Correct Answer
5. A signal other than the reference input that tends to affect the value of controlled variable is known as

(A) Disturbance
(B) Command
(C) Control element
(D) Reference input
Correct Answer

6. In a system low friction coefficient facilitates

(A) Reduced velocity lag error


(B) Increased velocity lag error
(C) Increased speed of response
(D) Reduced time constant of the system
Correct Answer

7. The type-2 system has ________ at the origin.


(A) No net pole
(B) Net pole
(C) Simple pole
(D) Two poles
Correct Answer

8. Addition of zeros in transfer function causes which of the following?

(A) Lead-compensation
(B) Lag-compensation
(C) Lead-lag compensation
(D) None of the above
Correct Answer

9. In pneumatic control systems the control valve used as final control element converts

(A) Pressure signal to electric signal


(B) Pressure signal to position change
(C) Electric signal to pressure signal
(D) Position change to pressure signal
Correct Answer

10. When the initial conditions of a system are specified to be zero it implies that the system is

(A) At rest without any energy stored in it


(B) Working normally with reference input
(C) Working normally with zero reference input
(D) At rest but stores energy
Correct Answer
View All Answers
01. Answer: Option C 02. Answer: Option D 03. Answer: Option A 04. Answer: Option A 05. Answer: Option A 06.
Answer: Option A 07. Answer: Option D 08. Answer: Option B 09. Answer: Option B 10. Answer: Option D

1. An amplidyne can give which of the following characteristics?


(A) Constant current
(B) Constant voltage
(C) Constant current as well as constant voltage
(D) Constant current, constant voltage and constant power
Correct Answer

2. A.C. servomotor is basically a


(A) Universal motor
(B) Single phase induction motor
(C) Two phase induction motor
(D) Three phase induction motor
Correct Answer

3. In an open loop control system


(A) Output is independent of control input
(B) Output is dependent on control input
(C) Only system parameters have effect on the control output
(D) None of the above
Correct Answer

4. A car is running at a constant speed of 50 km/h, which of the following is the feedback element for the driver?
(A) Clutch
(B) Eyes
(C) Needle of the speedometer
(D) Steering wheel
Correct Answer

5. A.C. servomotor resembles


(A) Two phase induction motor
(B) Three phase induction motor
(C) Direct current series motor
(D) Universal motor
Correct Answer

6. Which of the following statements is correct for a system with gain margin close to unity or a phase margin close to zero?
(A) The system is relatively stable
(B) The system is highly stable
(C) The system is highly oscillatory
(D) None of the above
Correct Answer
7. In force-voltage analogy, velocity is analogous to
(A) Current
(B) Charge
(C) Inductance
(D) Capacitance
Correct Answer

8. By which of the following the system response can be tested better?


(A) Ramp input signal
(B) Sinusoidal input signal
(C) Unit impulse input signal
(D) Exponentially decaying signal
Correct Answer

9. The position and velocity errors of a type-2 system are


(A) Constant, constant
(B) Constant, infinity
(C) Zero, constant
(D) Zero, zero
Correct Answer

10. The phase lag produced by transportation relays


(A) Is independent of frequency
(B) Is inversely proportional to frequency
(C) Increases linearly with frequency
(D) Decreases linearly with frequency
Correct Answer

View All Answers


01. Answer: Option D 02. Answer: Option C 03. Answer: Option A 04. Answer: Option C 05. Answer: Option A 06.
Answer: Option C 07. Answer: Option A 08. Answer: Option C 09. Answer: Option C 10. Answer: Option C

1. Which of the following is the definition of proportional band of a controller?


(A) The range of air output as measured variable varies from maximum to minimum
(B) The range of measured variables from set value
(C) The range of measured variables through which the air output changes from maximum to
minimum
(D) Any of the above
Correct Answer

2. Which of the following is the nonlinearity caused by servomotor?


(A) Static friction
(B) Backlash
(C) Saturation
(D) None of the above
Correct Answer
3. _________ directly converts temperature into voltage.
(A) Thermocouple
(B) Potentiometer
(C) Gear train
(D) LVDT
Correct Answer

4. Which of the following is exhibited by Root locus diagrams?


(A) The poles of the transfer function for a set of parameter values
(B) The bandwidth of the system
(C) The response of a system to a step input
(D) The frequency response of a system
Correct Answer

5. Regenerative feedback implies feedback with


(A) Oscillations
(B) Step input
(C) Negative sign
(D) Positive sign
Correct Answer

6. For open control system which of the following statements is incorrect?


(A) Less expensive
(B) Recalibration is not required for maintaining the required quality of the output
(C) Construction is simple and maintenance easy
(D) Errors are caused by disturbances
Correct Answer

7. In a system zero initial condition means that


(A) The system is at rest and no energy is stored in any of its components
(B) The system is working with zero stored energy
(C) The system is working with zero reference signals
(D) None of the above
Correct Answer

8. Which of the following is the best method for determining the stability and transient response?

(A) Root locus


(B) Bode plot
(C) Nyquist plot
(D) None of the above
Correct Answer

9. Technique gives quick transient and stability response


(A) Root locus
(B) Bode
(C) Nyquist
(D) Nichols
Correct Answer

10. The first order control system, which is well designed, has a
(A) Small bandwidth
(B) Negative time constant
(C) Large negative transfer function pole
(D) None of the above
Correct Answer

View All Answers


01. Answer: Option C 02. Answer: Option C 03. Answer: Option A 04. Answer: Option A 05. Answer: Option D 06.
Answer: Option B 07. Answer: Option A 08. Answer: Option A 09. Answer: Option A 10. Answer: Option C

1. If the gain of the critical damped system is increased it will behave as


(A) Oscillatory
(B) Critically damped
(C) Over-damped
(D) Under-damped
Correct Answer

2. The capacitance, in force-current analogy, is analogous to

(A) Momentum
(B) Velocity
(C) Displacement
(D) Mass
Correct Answer

3. The transient response, with feedback system,


(A) Rises slowly
(B) Rises quickly
(C) Decays slowly
(D) Decays quickly
Correct Answer

4. ________ is an open loop control system.


(A) Ward Leonard control
(B) Field controlled D.C. motor
(C) Stroboscope
(D) Metadyne
Correct Answer

5. Is a part of the human temperature control system?


(A) Digestive system
(B) Perspiration system
(C) Ear
(D) Leg movement
Correct Answer

6. The initial response when the output is not equal to input is called
(A) Transient response
(B) Error response
(C) Dynamic response
(D) Either of the above
Correct Answer

7. A control system in which the control action is somehow dependent on the output is known as
(A) Closed loop system
(B) Semi-closed loop system
(C) Open system
(D) None of the above
Correct Answer

8. Which of the following statements is correct for any closed loop system?
(A) All the coefficients can have zero value
(B) All the coefficients are always nonzero
(C) Only one of the static error coefficients has a finite nonzero value
(D) None of the above
Correct Answer

9. Mass, in force-voltage analogy, is analogous to


(A) Charge
(B) Current
(C) Inductance
(D) Resistance
Correct Answer

10. An increase in gain, in most systems, leads to


(A) Smaller damping ratio
(B) Larger damping ratio
(C) Constant damping ratio
(D) None of the above
Correct Answer

View All Answers


01. Answer: Option D 02. Answer: Option D 03. Answer: Option D 04. Answer: Option B 05. Answer: Option B 06.
Answer: Option A 07. Answer: Option A 08. Answer: Option C 09. Answer: Option C 10. Answer: Option A

1. Phase margin of a system is used to specify which of the following?


(A) Frequency response
(B) Absolute stability
(C) Relative stability
(D) Time response
Correct Answer

2. The type 0 system has _______ at the origin.


(A) No pole
(B) Net pole
(C) Simple pole
(D) Two poles
Correct Answer

3. ________ is not a final control element.


(A) Control valve
(B) Potentiometer
(C) Electro-pneumatic converter
(D) Servomotor
Correct Answer

4. Which of the following is an electromagnetically device?


(A) Induction relay
(B) Thermocouple
(C) LVDT
(D) Any of the above
Correct Answer

5. The transfer function technique is considered as inadequate under which of the following conditions?
(A) Systems having complexities and nonlinearities
(B) Systems having stability problems
(C) Systems having multiple input disturbances
(D) All of the above
Correct Answer

6. In closed loop control system, with positive value of feedback gain the overall gain of the system will
(A) Decrease
(B) Increase
(C) Be unaffected
(D) Any of the above
Correct Answer

7. An automatic toaster is a ________ loop control system.


(A) Open
(B) Closed
(C) Partially closed
(D) Any of the above
Correct Answer

8. Which of the following should be done to make an unstable system stable?


(A) The gain of the system should be decreased
(B) The gain of the system should be increased
(C) The number of poles to the loop transfer function should be increased
(D) The number of zeros to the loop transfer function should be increased
Correct Answer

9. The output of a feedback control system must be a function of


(A) Reference and output
(B) Reference and input
(C) Input and feedback signal
(D) Output and feedback signal
Correct Answer

10. Due to which of the following reasons excessive bond width in control systems should be avoided?
(A) It leads to slow speed of response
(B) It leads to low relative stability
(C) Noise is proportional to band width
(D) None of the above
Correct Answer

View All Answers


01. Answer: Option C 02. Answer: Option A 03. Answer: Option B 04. Answer: Option C 05. Answer: Option D 06.
Answer: Option A 07. Answer: Option A 08. Answer: Option B 09. Answer: Option A 10. Answer: Option C

1. In electrical-pneumatic system analogy the current is considered analogous to


(A) Velocity
(B) Pressure
(C) Air flow
(D) Air flow rate
Correct Answer

2. Signal will become zero when the feedback signal and reference signs are equal.
(A) Input
(B) Actuating
(C) Feedback
(D) Reference
Correct Answer

3. With feedback _______ increases.


(A) System stability
(B) Sensitivity
(C) Gain
(D) Effects of disturbing signals
Correct Answer

4. Velocity error constant of a system is measured when the input to the system is unit ________ function.
(A) Parabolic
(B) Ramp
(C) Impulse
(D) Step
Correct Answer

5. Which of the following can be measured by the use of a tachogenerator?


(A) Acceleration
(B) Speed
(C) Speed and acceleration
(D) Displacement
Correct Answer

6. A differentiator is usually not a part of a control system because it


(A) Reduces damping
(B) Reduces the gain margin
(C) Increases input noise
(D) Increases error
Correct Answer

7. Which of the following is an open loop control system?


(A) Field controlled D.C. motor
(B) Ward Leonard control
(C) Metadyne
(D) Stroboscope
Correct Answer

8. ________ Increases the steady state accuracy.


(A) Integrator
(B) Differentiator
(C) Phase lead compensator
(D) Phase lag compensator
Correct Answer

9. In a stable control system backlash can cause which of the following?


(A) Under-damping
(B) Over-damping
(C) Poor stability at reduced values of open loop gain
(D) Low-level oscillations
Correct Answer
10. The transient response of a system is mainly due to
(A) Inertia forces
(B) Internal forces
(C) Stored energy
(D) Friction
Correct Answer

View All Answers


01. Answer: Option D 02. Answer: Option B 03. Answer: Option A 04. Answer: Option B 05. Answer: Option B 06.
Answer: Option C 07. Answer: Option A 08. Answer: Option A 09. Answer: Option D 10. Answer: Option C

1. The frequency and time domain are related through which of the following?
(A) Laplace Transform and Fourier Integral
(B) Laplace Transform
(C) Fourier Integral
(D) Either (B) or (C)
Correct Answer

2. In case of type-1 system steady state acceleration is


(A) Unity
(B) Infinity
(C) Zero
(D) 10
Correct Answer

3. Which of the following statements is not necessarily correct for open control system?
(A) Input command is the sole factor responsible for providing the control action
(B) Presence of nonlinearities causes malfunctioning
(C) Less expensive
(D) Generally free from problems of nonlinearities
Correct Answer

4. Zero initial condition for a system means


(A) Input reference signal is zero
(B) Zero stored energy
(C) Net initial movement of moving parts
(D) System is at rest and no energy is stored in any of its components
Correct Answer

5. A controller, essentially, is a
(A) Sensor
(B) Clipper
(C) Comparator
(D) Amplifier
Correct Answer
6. The transfer function is applicable to which of the following?
(A) Linear and time-in variant systems
(B) Linear and time-variant systems
(C) Linear systems
(D) Nonlinear systems
Correct Answer

7. A conditionally stable system exhibits poor stability at


(A) Low frequencies
(B) Reduced values of open loop gain
(C) Increased values of open loop gain
(D) None of the above
Correct Answer

8. Which of the following devices is used for conversion of coordinates?


(A) Microsyn
(B) Selsyn
(C) Synchro-resolver
(D) Synchro-transformer
Correct Answer

9. With feedback ________ reduces.


(A) System stability
(B) System gain
(C) System stability and gain
(D) None of the above
Correct Answer

10. In open loop system


(A) The control action depends on the size of the system
(B) The control action depends on system variables
(C) The control action depends on the input signal
(D) The control action is independent of the output
Correct Answer

View All Answers


01. Answer: Option A 02. Answer: Option B 03. Answer: Option B 04. Answer: Option D 05. Answer: Option C 06.
Answer: Option A 07. Answer: Option B 08. Answer: Option C 09. Answer: Option B 10. Answer: Option D

1. A closed loop system is distinguished from open loop system by which of the following?
(A) Servomechanism
(B) Feedback
(C) Output pattern
(D) Input pattern
Correct Answer
2. The band width, in a feedback amplifier.
(A) Remains unaffected
(B) Decreases by the same amount as the gain increase
(C) Increases by the same amount as the gain decrease
(D) Decreases by the same amount as the gain decrease
Correct Answer

3. In liquid level and electrical system analogy, voltage is considered analogous to


(A) Head
(B) Liquid flow
(C) Liquid flow rate
(D) None of the above
Correct Answer

4. _______ is the reference input minus the primary feedback.


(A) Manipulated variable
(B) Zero sequence
(C) Actuating signal
(D) Primary feedback
Correct Answer

5. If a step function is applied to the input of a system and the output remains below a certain level for all the time, the system
is
(A) Not necessarily stable
(B) Stable
(C) Unstable
(D) Always unstable
Correct Answer

6. The effect of error damping is to


(A) Provide larger settling lime
(B) Delay the response
(C) Reduce steady state error
(D) Any of the above
Correct Answer

7. Which of the following can be measured by LVDT?


(A) Displacement
(B) Velocity
(C) Acceleration
(D) Any of the above
Correct Answer

8. Any externally introduced signal affecting the controlled output is called a


(A) Feedback
(B) Stimulus
(C) Signal
(D) Gain control
Correct Answer

9. Transfer function of a system is used to calculate which of the following?


(A) The order of the system
(B) The time constant
(C) The output for any given input
(D) The steady state gain
Correct Answer

10. Has tendency to oscillate.


(A) Open loop system
(B) Closed loop system
(C) Both (A) and (B)
(D) Neither (A) nor (B)
Correct Answer

View All Answers


01. Answer: Option B 02. Answer: Option C 03. Answer: Option A 04. Answer: Option C 05. Answer: Option A 06.
Answer: Option C 07. Answer: Option D 08. Answer: Option B 09. Answer: Option C 10. Answer: Option B

1. A phase lag lead network introduces in the output


(A) Lag at all frequencies
(B) Lag at high frequencies and lead at low frequencies
(C) Lag at low frequencies and lead at high frequencies
(D) None of the above
Correct Answer

2. The type-1 system has ________ at the origin.


(A) No pole
(B) Net pole
(C) Simple pole
(D) Two poles
Correct Answer

3. Spring constant in force-voltage analogy is analogous to


(A) Capacitance
(B) Reciprocal of capacitance
(C) Current
(D) Resistance
Correct Answer
4. The temperature, under thermal and electrical system analogy, is considered analogous to
(A) Voltage
(B) Current
(C) Capacitance
(D) Charge
Correct Answer

5. The second derivative input signals modify which of the following?


(A) The time constant of the system
(B) Damping of the system
(C) The gain of the system
(D) The time constant and suppress the oscillations
Correct Answer

6. ________ is a closed loop system.


(A) Autopilot for an aircraft
(B) Direct current generator
(C) Car starter
(D) Electric switch
Correct Answer

7. A good control system has all the following features except


(A) Good stability
(B) Slow response
(C) Good accuracy
(D) Sufficient power handling capacity
Correct Answer

8. Technique is not applicable to nonlinear system?


(A) Nyquist Criterion
(B) Quasi linearization
(C) Functional analysis
(D) Phase-plane representation
Correct Answer

9. In a control system integral error compensation _________ steady state error


(A) Increases
(B) Minimizes
(C) Does not have any effect on
(D) Any of the above
Correct Answer

10. In thermal-electrical analogy charge is considered analogous to


(A) Heat flow
(B) Reciprocal of heat flow
(C) Reciprocal of temperature
(D) Temperature
Correct Answer

View All Answers


01. Answer: Option C 02. Answer: Option C 03. Answer: Option B 04. Answer: Option A 05. Answer: Option D 06.
Answer: Option A 07. Answer: Option B 08. Answer: Option A 09. Answer: Option B 10. Answer: Option D

1. Which of the following devices are commonly used as error detectors in instruments?
(A) Vernistats
(B) Microsyns
(C) Resolvers
(D) Any of the above
Correct Answer

2. A control system with excessive noise, is likely to suffer from


(A) Saturation in amplifying stages
(B) Loss of gain
(C) Vibrations
(D) Oscillations
Correct Answer

3. On which of the following factors does the sensitivity of a closed loop system to gain changes and load disturbances depend?
(A) Frequency
(B) Loop gain
(C) Forward gain
(D) All of the above
Correct Answer

4. In an automatic control system which of the following elements is not used?


(A) Error detector
(B) Final control element
(C) Sensor
(D) Oscillator
Correct Answer

5. The viscous friction coefficient, in force-voltage analogy, is analogous to


(A) Charge
(B) Resistance
(C) Reciprocal of inductance
(D) Reciprocal of conductance
Correct Answer

6. From which of the following transfer function can be obtained?


(A) Signal flow graph
(B) Analogous table
(C) Output-input ratio
(D) Standard block system
Correct Answer

7. Static error coefficients are used as a measure of the effectiveness of closed loop systems for specified ________ input signal.
(A) Acceleration
(B) Velocity
(C) Position
(D) All of the above
Correct Answer

8. In order to increase the damping of a badly under-damped system which of following compensators may be used?
(A) Phase-lead
(B) Phase-lag
(C) Both (A) and (B)
(D) Either (A) and (B)
Correct Answer

9. ________ can be extended to systems which are time-varying?


(A) Bode-Nyquist stability methods
(B) Transfer functions
(C) Root locus design
(D) State model representatives
Correct Answer

10. Which of the following is the output of a thermocouple?


(A) Alternating current
(B) Direct current
(C) A.C. voltage
(D) D.C. voltage
Correct Answer

View All Answers


01. Answer: Option D 02. Answer: Option A 03. Answer: Option D 04. Answer: Option D 05. Answer: Option B 06.
Answer: Option A 07. Answer: Option D 08. Answer: Option A 09. Answer: Option D 10. Answer: Option D

You might also like